Você está na página 1de 173

SAT Practice Test #1

IMPORTANT REMINDERS

A No. 2 pencil is required for the test. Sharing any questions with anyone
Do not use a mechanical pencil or pen. is a violation of Test Security
and Fairness policies and may result
in your scores being canceled.

This cover is representative of what you’ll see on test day.

THIS TEST BOOK MUST NOT BE TAKEN FROM THE ROOM. UNAUTHORIZED
REPRODUCTION OR USE OF ANY PART OF THIS TEST BOOK IS PROHIBITED.
© 2015 The College Board. College Board, SAT, and the acorn logo are registered trademarks of the College Board.
2 2
Writing and Language Test
35 M I NU TES, 4 4 QUESTIONS

Turn to Section 2 of your answer sheet to answer the questions in this section.

DIRECTIONS

Each passage below is accompanied by a number of questions. For some questions, you
will consider how the passage might be revised to improve the expression of ideas. For
other questions, you will consider how the passage might be edited to correct errors in
sentence structure, usage, or punctuation. A passage or a question may be accompanied by
one or more graphics (such as a table or graph) that you will consider as you make revising
and editing decisions.

Some questions will direct you to an underlined portion of a passage. Other questions will
direct you to a location in a passage or ask you to think about the passage as a whole.

After reading each passage, choose the answer to each question that most effectively
improves the quality of writing in the passage or that makes the passage conform to the
conventions of standard written English. Many questions include a “NO CHANGE” option.
Choose that option if you think the best choice is to leave the relevant portion of the
passage as it is.

Questions 1-11 are based on the following passage.


...................................................................................................................................

Whey to Go
Greek yogurt—a strained form of cultured
yogurt—has grown enormously in popularity in the
United States since it was first introduced in the country
in the late 1980s.
From 2011 to 2012 alone, sales of Greek yogurt in the
US increased by 50 percent. The resulting increase in
Greek yogurt production has forced those involved in the
business to address the detrimental effects that the
yogurt-making process may be having on the
environment. Fortunately, farmers and others in the

Unauthorized copying or reuse of any part of this page is illegal.


18 CO NTI N U E
2 2
Greek yogurt business have found many methods of

...............................................................................................................................................................................................................................................................................................................
1
controlling and eliminating most environmental threats. A) NO CHANGE
B) defeat
Given these solutions as well as the many health benefits
C) outperform
of the food, the advantages of Greek yogurt 1 outdo the
D) outweigh
potential drawbacks of its production.
[1] The main environmental problem caused by the 2
production of Greek yogurt is the creation of acid whey Which choice provides the most relevant detail?
as a by-product. [2] Because it requires up to four times A) NO CHANGE
B) supplement and convert it into gas to use as fuel
more milk to make than conventional yogurt does, Greek in electricity production.
yogurt produces larger amounts of acid whey, which is C) supplement, while sweet whey is more desirable
as a food additive for humans.
difficult to dispose of. [3] To address the problem of
D) supplement, which provides an important
disposal, farmers have found a number of uses for acid element of their diet.
whey. [4] They can add it to livestock feed as a protein
2 supplement, and people can make their own 3
A) NO CHANGE
Greek-style yogurt at home by straining regular yogurt.
B) can pollute waterway’s,
[5] If it is improperly introduced into the environment, C) could have polluted waterways,
acid-whey runoff 3 can pollute waterways, depleting D) has polluted waterway’s,

the oxygen content of streams and rivers as it
decomposes. [6] Yogurt manufacturers, food 4
A) NO CHANGE
4 scientists; and government officials are also
B) scientists: and
working together to develop additional solutions for
C) scientists, and
reusing whey. 5 D) scientists, and,

5
To make this paragraph most logical, sentence 5
should be placed
A) where it is now.
B) after sentence 1.
C) after sentence 2.
D) after sentence 3.

Unauthorized copying or reuse of any part of this page is illegal.


19 CO NTI N U E
2 2
6 Though these conservation methods can be

...............................................................................................................................................................................................................................................................................................................
6
costly and time-consuming, they are well worth the The writer is considering deleting the underlined
sentence. Should the writer do this?
effort. Nutritionists consider Greek yogurt to be a healthy
A) Yes, because it does not provide a transition
food: it is an excellent source of calcium and protein, from the previous paragraph.
serves 7 to be a digestive aid, and 8 it contains few B) Yes, because it fails to support the main
argument of the passage as introduced in the
calories in its unsweetened low- and non-fat forms. first paragraph.
Greek yogurt is slightly lower in sugar and carbohydrates C) No, because it continues the explanation of how
acid whey can be disposed of safely.
than conventional yogurt is. 9 Also, because it is more D) No, because it sets up the argument in the
concentrated, Greek yogurt contains slightly more paragraph for the benefits of Greek yogurt.

protein per serving, thereby helping people stay


7
A) NO CHANGE
B) as
C) like
D) for

8
A) NO CHANGE
B) containing
C) contains
D) will contain

9
A) NO CHANGE
B) In other words,
C) Therefore,
D) For instance,

Unauthorized copying or reuse of any part of this page is illegal.


20 CO NTI N U E
2 2
10 satiated for longer periods of time. These health

...............................................................................................................................................................................................................................................................................................................
10
benefits have prompted Greek yogurt’s recent surge in A) NO CHANGE
B) fulfilled
popularity. In fact, Greek yogurt can be found in an
C) complacent
increasing number of products such as snack food and
D) sufficient
frozen desserts. Because consumers reap the nutritional
benefits of Greek yogurt and support those who make 11
and sell 11 it, therefore farmers and businesses should A) NO CHANGE
B) it, farmers
continue finding safe and effective methods of producing
C) it, so farmers
the food.
D) it: farmers

Unauthorized copying or reuse of any part of this page is illegal.


21 CO NTI N U E
2 2
Questions 12-22 are based on the following passage

...............................................................................................................................
12
and supplementary material.
Which choice most accurately and effectively
represents the information in the graph?
Dark Snow A) NO CHANGE
Most of Greenland’s interior is covered by a thick B) to 12 degrees Fahrenheit.
C) to their lowest point on December 13.
layer of ice and compressed snow known as the
D) to 10 degrees Fahrenheit and stay there for
Greenland Ice Sheet. The size of the ice sheet fluctuates months.
seasonally: in summer, average daily high temperatures
in Greenland can rise to slightly above 50 degrees
Fahrenheit, partially melting the ice; in the winter, the
sheet thickens as additional snow falls, and average daily
low temperatures can drop 12 to as low as 20 degrees.

Average Daily High and Low Temperatures Recorded


at Nuuk Weather Station, Greenland (1961—1990)

Mar 5 Jun 10 Sep 8 Dec 13


60
Temperature (°F)

50 45° 45°
40
30 35° 38° 26°
20°
20
19°
10
12°
0
Jan Feb Mar Apr May Jun Jul Aug Sep Oct Nov Dec
average daily high
average daily low

Adapted from WMO. ©2014 by World Meteorological Organization.

Unauthorized copying or reuse of any part of this page is illegal.


22 CO NTI N U E
2 2
Typically, the ice sheet begins to show evidence of

...............................................................................................................................................................................................................................................................................................................
13
thawing in late 13 summer. This follows several weeks of Which choice most effectively combines the two
sentences at the underlined portion?
higher temperatures. 14 For example, in the summer of
A) summer, following
2012, virtually the entire Greenland Ice Sheet underwent B) summer, and this thawing follows
thawing at or near its surface by mid-July, the earliest C) summer, and such thawing follows
date on record. Most scientists looking for the causes of D) summer and this evidence follows

the Great Melt of 2012 have focused exclusively on rising


14
temperatures. The summer of 2012 was the warmest in
A) NO CHANGE
170 years, records show. But Jason 15 Box, an associate B) However,
professor of geology at Ohio State believes that another C) As such,
factor added to the early 16 thaw; the “dark snow” D) Moreover,

problem.
15
A) NO CHANGE
B) Box an associate professor of geology at
Ohio State,
C) Box, an associate professor of geology at
Ohio State,
D) Box, an associate professor of geology, at
Ohio State

16
A) NO CHANGE
B) thaw; and it was
C) thaw:
D) thaw: being

Unauthorized copying or reuse of any part of this page is illegal.


23 CO NTI N U E
2 2
According to Box, a leading Greenland expert,

...............................................................................................................................................................................................................................................................................................................
17
tundra fires in 2012 from as far away as North America A) NO CHANGE
B) soot
produced great amounts of soot, some 17 of it drifted
C) of which
over Greenland in giant plumes of smoke and then
D) DELETE the underlined portion.
18 fell as particles onto the ice sheet. Scientists have long
known that soot particles facilitate melting by darkening 18
snow and ice, limiting 19 it’s ability to reflect the Sun’s A) NO CHANGE
rays. As Box explains, “Soot is an extremely powerful B) falls
C) will fall
light absorber. It settles over the ice and captures the
D) had fallen
Sun’s heat.” The result is a self-reinforcing cycle. As the
ice melts, the land and water under the ice become 19
exposed, and since land and water are darker than snow, A) NO CHANGE
the surface absorbs even more heat, which 20 is related B) its
C) there
to the rising temperatures.
D) their

20
Which choice best completes the description of a
self-reinforcing cycle?
A) NO CHANGE
B) raises the surface temperature.
C) begins to cool at a certain point.
D) leads to additional melting.

Unauthorized copying or reuse of any part of this page is illegal.


24 CO NTI N U E
2 2

...............................................................................................................................................................................................................................................................................................................
[1] Box’s research is important because the fires of 21
2012 may not be a one-time phenomenon. [2] According A) NO CHANGE
B) itself,
to scientists, rising Arctic temperatures are making
C) itself, with damage and
northern latitudes greener and thus more fire prone. D) itself possibly,
[3] The pattern Box observed in 2012 may repeat
21 itself again, with harmful effects on the Arctic 22
ecosystem. [4] Box is currently organizing an expedition To make this paragraph most logical, sentence 4
should be placed
to gather this crucial information. [5] The next step for
A) where it is now.
Box and his team is to travel to Greenland to perform B) after sentence 1.
direct sampling of the ice in order to determine just how C) after sentence 2.
much the soot is contributing to the melting of the ice D) after sentence 5.

sheet. [6] Members of the public will be able to track his


team’s progress—and even help fund the
expedition—through a website Box has created. 22

Unauthorized copying or reuse of any part of this page is illegal.


25 CO NTI N U E
2 2
Questions 23-33 are based on the following passage.

...............................................................................................................................................................................................................................................................................................................
23
A) NO CHANGE
Coworking: A Creative Solution B) was promptly worn
C) promptly wore
When I left my office job as a website developer at a
D) wore
small company for a position that allowed me to work
full-time from home, I thought I had it made: I gleefully
24
traded in my suits and dress shoes for sweatpants and A) NO CHANGE
slippers, my frantic early-morning bagged lunch packing B) colleagues were important for sharing ideas.

for a leisurely midday trip to my refrigerator. The novelty C) ideas couldn’t be shared with colleagues.
D) I missed having colleagues nearby to consult.
of this comfortable work-from-home life, however,
23 soon got worn off quickly. Within a month, I found
25
myself feeling isolated despite having frequent email and A) NO CHANGE
instant messaging contact with my colleagues. Having B) about
become frustrated trying to solve difficult problems, C) upon
D) for
24 no colleagues were nearby to share ideas. It was
during this time that I read an article 25 into coworking
spaces.

Unauthorized copying or reuse of any part of this page is illegal.


26 CO NTI N U E
2 2

...............................................................................................................................................................................................................................................................................................................
The article, published by Forbes magazine, explained 26
that coworking spaces are designated locations that, for a A) NO CHANGE
B) equipment, such as:
fee, individuals can use to conduct their work. The spaces
C) equipment such as:
are usually stocked with standard office 26 equipment, D) equipment, such as,
such as photocopiers, printers, and fax machines. 27 In
these locations, however, the spaces often include small 27
meeting areas and larger rooms for hosting presentations. A) NO CHANGE
B) In addition to equipment,
28 The cost of launching a new coworking business in
C) For these reasons,
the United States is estimated to be approximately
D) Likewise,
$58,000.

28
The writer is considering deleting the underlined
sentence. Should the sentence be kept or deleted?
A) Kept, because it provides a detail that supports
the main topic of the paragraph.
B) Kept, because it sets up the main topic of the
paragraph that follows.
C) Deleted, because it blurs the paragraph’s main
focus with a loosely related detail.
D) Deleted, because it repeats information that has
been provided in an earlier paragraph.

Unauthorized copying or reuse of any part of this page is illegal.


27 CO NTI N U E
2 2
What most caught my interest, though, was a

...............................................................................................................................................................................................................................................................................................................
29
quotation from someone who described coworking At this point, the writer wants to add specific
information that supports the main topic of the
spaces as “melting pots of creativity.” The article refers to paragraph.
a 2012 survey in which 29 64 percent of respondents Perceived Effect of Coworking on Business Skills
noted that coworking spaces prevented them from
positive impact negative impact
completing tasks in a given time. The article goes on to
suggest that the most valuable resources provided by ideas relating 74%
to business 2%
coworking spaces are actually the people 30 whom use
creativity 71%
them. 3%

ability to focus 68%


12%
completing tasks 64%
in a given time 8%

standard of work 62%


3%
Adapted from “The 3rd Global Coworking Survey.” ©2013 by
Deskmag.

Which choice most effectively completes the


sentence with relevant and accurate information
based on the graph above?
A) NO CHANGE
B) 71 percent of respondents indicated that using a
coworking space increased their creativity.
C) respondents credited coworking spaces with
giving them 74 percent of their ideas relating to
business.
D) respondents revealed that their ability to focus
on their work improved by 12 percent in a
coworking space.

30
A) NO CHANGE
B) whom uses
C) who uses
D) who use

Unauthorized copying or reuse of any part of this page is illegal.


28 CO NTI N U E
2 2

...............................................................................................................................................................................................................................................................................................................
[1] Thus, even though I already had all the 31
equipment I needed in my home office, I decided to try The writer wants to add the following sentence to the
paragraph.
using a coworking space in my city. [2] Because I was
After filling out a simple registration form and
specifically interested in coworking’s reported benefits taking a quick tour of the facility, I took a seat at
related to creativity, I chose a facility that offered a bright, a table and got right to work on my laptop.
The best placement for the sentence is immediately
open work area where I wouldn’t be isolated.
A) before sentence 1.
[3] Throughout the morning, more people appeared.
B) after sentence 1.
[4] Periods of quiet, during which everyone worked C) after sentence 2.
independently, were broken up occasionally with lively D) after sentence 3.
conversation. 31
I liked the experience so much that I now go to the 32
A) NO CHANGE
coworking space a few times a week. Over time, I’ve
B) colleagues;
gotten to know several of my coworking 32 colleagues: C) colleagues,
another website developer, a graphic designer, a freelance D) colleagues
writer, and several mobile app coders. Even those of us
who work in disparate fields are able to 33 share advice 33
A) NO CHANGE
and help each other brainstorm. In fact, it’s the diversity
B) give some wisdom
of their talents and experiences that makes my coworking C) proclaim our opinions
colleagues so valuable. D) opine

Unauthorized copying or reuse of any part of this page is illegal.


29 CO NTI N U E
2 2
Questions 34-44 are based on the following passage.

...............................................................................................................................................................................................................................................................................................................
34
A) NO CHANGE
The Consolations of Philosophy B) For example,
C) In contrast,
Long viewed by many as the stereotypical useless
D) Nevertheless,
major, philosophy is now being seen by many students
and prospective employers as in fact a very useful and
35
practical major, offering students a host of transferable A) NO CHANGE
skills with relevance to the modern workplace. 34 In B) speaking in a more pragmatic way,
broad terms, philosophy is the study of meaning and the C) speaking in a way more pragmatically,
D) in a more pragmatic-speaking way,
values underlying thought and behavior. But 35 more
pragmatically, the discipline encourages students to
36
analyze complex material, question conventional beliefs, A) NO CHANGE
and express thoughts in a concise manner. B) teaches
Because philosophy 36 teaching students not what C) to teach
D) and teaching
to think but how to think, the age-old discipline offers
consistently useful tools for academic and professional
37
achievement. 37 A 1994 survey concluded that only
Which choice most effectively sets up the
18 percent of American colleges required at least one information that follows?
A) Consequently, philosophy students have been
philosophy course. 38 Therefore, between 1992 and
receiving an increasing number of job offers.
1996, more than 400 independent philosophy B) Therefore, because of the evidence, colleges
increased their offerings in philosophy.
departments were eliminated from institutions.
C) Notwithstanding the attractiveness of this course
of study, students have resisted majoring in
philosophy.
D) However, despite its many utilitarian benefits,
colleges have not always supported the study of
philosophy.

38
A) NO CHANGE
B) Thus,
C) Moreover,
D) However,

Unauthorized copying or reuse of any part of this page is illegal.


30 CO NTI N U E
2 2

...............................................................................................................................................................................................................................................................................................................
More recently, colleges have recognized the 39
practicality and increasing popularity of studying Which choice most effectively combines the
sentences at the underlined portion?
philosophy and have markedly increased the number of
A) writing as
philosophy programs offered. By 2008 there were B) writing, and these results can be
817 programs, up from 765 a decade before. In addition, C) writing, which can also be
the number of four-year graduates in philosophy has D) writing when the results are

grown 46 percent in a decade. Also, studies have found


40
that those students who major in philosophy often do
A) NO CHANGE
better than students from other majors in both verbal B) have scored
reasoning and analytical 39 writing. These results can be C) scores
measured by standardized test scores. On the Graduate D) scoring

Record Examination (GRE), for example, students


41
intending to study philosophy in graduate school 40 has
A) NO CHANGE
scored higher than students in all but four other majors. B) students majoring
These days, many 41 student’s majoring in C) students major
philosophy have no intention of becoming philosophers; D) student’s majors

instead they plan to apply those skills to other disciplines.


42
Law and business specifically benefit from the
At this point, the writer is considering adding the
complicated theoretical issues raised in the study of following sentence.
philosophy, but philosophy can be just as useful in The ancient Greek philosopher Plato, for example,
engineering or any field requiring complex analytic skills. wrote many of his works in the form of dialogues.
Should the writer make this addition here?
42 That these skills are transferable across professions
A) Yes, because it reinforces the passage’s main
point about the employability of philosophy
majors.
B) Yes, because it acknowledges a common
counterargument to the passage’s central claim.
C) No, because it blurs the paragraph’s focus by
introducing a new idea that goes unexplained.
D) No, because it undermines the passage’s claim
about the employability of philosophy majors.

Unauthorized copying or reuse of any part of this page is illegal.


31 CO NTI N U E
2 2
43 which makes them especially beneficial to

.....................................................................................................................................................................................................................................................
43
twenty-first-century students. Because today’s students A) NO CHANGE
B) that
can expect to hold multiple jobs—some of which may not
C) and
even exist yet—during 44 our lifetime, studying
D) DELETE the underlined portion.
philosophy allows them to be flexible and adaptable.
High demand, advanced exam scores, and varied 44
professional skills all argue for maintaining and A) NO CHANGE

enhancing philosophy courses and majors within B) one’s


C) his or her
academic institutions.
D) their

STOP
If you finish before time is called, you may check your work on this section only.
Do not turn to any other section.

Unauthorized copying or reuse of any part of this page is illegal.


32
SAT Practice Test #1: Worksheets

ANSWER KEY

Reading Test Answers Writing and Language Test Answers

1 B 12 B 23 D 34 C 45 D 1 D 12 B 23 D 34 A
2 B 13 D 24 D 35 B 46 C 2 B 13 A 24 D 35 A
3 C 14 A 25 C 36 D 47 B 3 A 14 B 25 B 36 B
4 A 15 A 26 B 37 C 48 A 4 C 15 C 26 A 37 D
5 C 16 C 27 D 38 C 49 D 5 C 16 C 27 B 38 C
6 D 17 C 28 C 39 B 50 B 6 D 17 C 28 C 39 A
7 D 18 D 29 A 40 C 51 D 7 B 18 A 29 B 40 B
8 B 19 A 30 A 41 B 52 A 8 C 19 D 30 D 41 B
9 C 20 B 31 D 42 B 9 A 20 D 31 C 42 C
10 B 21 A 32 B 43 A 10 A 21 B 32 A 43 D

11 A 22 B 33 A 44 A 11 B 22 D 33 A 44 D

READING TEST
WRITING AND
RAW SCORE
LANGUAGE TEST
(NUMBER OF
RAW SCORE
CORRECT ANSWERS)
(NUMBER OF
CORRECT ANSWERS)

Math Test Math Test


No Calculator Answers Calculator Answers

1 D 11 D 1 B 11 A 21 C 31 Any number between 4-6, inclusive


2 A 12 D 2 C 12 C 22 B 32 107

3 C 13 B 3 D 13 C
23 B 33 5/8 or 0.625
4 B 14 A 4 C 14 C 24 A 34 96

5 C 15 D 5 D 15 A
25 D 35 6

6 A 16 2 6 D 16 C
26 B 36 3

7 B 17 1600 7 C 17 B
27 C 37 1.02

8 C 18 7 8 D 18 A
28 C 38 6.11

9 B 19 4/5 or 0.8 9 A 19 B
29 D
10 A 20 100 10 B 20 D
30 D

MATH TEST MATH TEST


NO CALCULATOR CALCULATOR
RAW SCORE RAW SCORE
(NUMBER OF (NUMBER OF
CORRECT ANSWERS) CORRECT ANSWERS)

SAT Practice Test #1 Created 8/4/2015 6


SAT Practice Test #2
®

IMPORTANT REMINDERS

A No. 2 pencil is required for the test. Sharing any questions with anyone
Do not use a mechanical pencil or pen. is a violation of Test Security
and Fairness policies and may result
in your scores being canceled.

This cover is representative of what you’ll see on test day.

THIS TEST BOOK MUST NOT BE TAKEN FROM THE ROOM. UNAUTHORIZED
REPRODUCTION OR USE OF ANY PART OF THIS TEST BOOK IS PROHIBITED.
© 2015 The College Board. College Board, SAT, and the acorn logo are registered trademarks of the College Board.
2 2
Writing and Language Test
35 M I NU TES, 4 4 QUESTIONS

Turn to Section 2 of your answer sheet to answer the questions in this section.

DIRECTIONS

Each passage below is accompanied by a number of questions. For some questions, you
will consider how the passage might be revised to improve the expression of ideas. For
other questions, you will consider how the passage might be edited to correct errors in
sentence structure, usage, or punctuation. A passage or a question may be accompanied by
one or more graphics (such as a table or graph) that you will consider as you make revising
and editing decisions.

Some questions will direct you to an underlined portion of a passage. Other questions will
direct you to a location in a passage or ask you to think about the passage as a whole.

After reading each passage, choose the answer to each question that most effectively
improves the quality of writing in the passage or that makes the passage conform to the
conventions of standard written English. Many questions include a “NO CHANGE” option.
Choose that option if you think the best choice is to leave the relevant portion of the
passage as it is.

Questions 1-11 are based on the following passage.


...................................................................................................................................

1
A) NO CHANGE
Librarians Help Navigate in the Digital Age B) reductions
In recent years, public libraries in the United States C) deducting
D) deducts
have experienced 1 reducing in their operating funds
due to cuts imposed at the federal, state, and local
2
government levels. 2 However, library staffing has been A) NO CHANGE
cut by almost four percent since 2008, and the demand B) Consequently,
for librarians continues to decrease, even though half of C) Nevertheless,
D) Previously,
public libraries report that they have an insufficient
number of staff to meet their patrons’ needs.
Employment in all job sectors in the United States is
projected to grow by fourteen percent over the next

Unauthorized copying or reuse of any part of this page is illegal. 18 CO NTI N U E


2 2
decade, yet the expected growth rate for librarians is

...............................................................................................................................................................................................................................................................................................................
3
predicted to be only seven percent, or half of the overall A) NO CHANGE
B) have
rate. This trend, combined with the increasing
C) which have
accessibility of information via the Internet, 3 has led
D) which has
some to claim that librarianship is in decline as a
profession. As public libraries adapt to rapid 4
technological advances in information distribution, At this point, the writer is considering adding the
following information.
librarians’ roles are actually expanding.
—e-books, audio and video materials, and online
The share of library materials that is in nonprint journals—
formats 4 is increasing steadily; in 2010, at least Should the writer make this addition here?
18.5 million e-books were available 5 for them to A) Yes, because it provides specific examples of the
materials discussed in the sentence.
circulate. As a result, librarians must now be proficient
B) Yes, because it illustrates the reason for the
curators of electronic information, compiling, increase mentioned later in the sentence.
6 catalog, and updating these collections. But perhaps C) No, because it interrupts the flow of the sentence
by supplying irrelevant information.
even more importantly, librarians function as first D) No, because it weakens the focus of the passage
responders for their communities’ computer needs. Since by discussing a subject other than librarians.

5
A) NO CHANGE
B) to be circulated by them.
C) for their circulating.
D) for circulation.

6
A) NO CHANGE
B) librarians cataloging,
C) to catalog,
D) cataloging,

Unauthorized copying or reuse of any part of this page is illegal. 19 CO NTI N U E


2 2
one of the fastest growing library services is public access

...............................................................................................................................................................................................................................................................................................................
7
computer use, there is great demand for computer Which choice most effectively combines the
underlined sentences?
instruction. 7 In fact, librarians’ training now includes
A) In fact, librarians’ training now includes courses
courses on research and Internet search methods. Many on research and Internet search methods; many
librarians teach classes in Internet navigation,
of whom teach classes in Internet navigation, database database and software use, and digital
and software use, and digital information literacy. While information literacy is taught by them.
B) In fact, many librarians, whose training now
these classes are particularly helpful to young students includes courses on research and Internet search
developing basic research skills, 8 but adult patrons can methods, teach classes in Internet navigation,
database and software use, and digital
also benefit from librarian assistance in that they can acquire information literacy.
job-relevant computer skills. 9 Free to all who utilize C) Training now includes courses on research and
Internet search methods; many librarians, in
their services, public libraries and librarians are especially fact, are teaching classes in Internet navigation,
database and software use, and digital
valuable, because they offer free resources that may be information literacy.
difficult to find elsewhere, such as help with online job D) Including courses on research and Internet
search methods in their training is, in fact,
why many librarians teach classes in Internet
navigation, database and software use, and
digital information literacy.

8
A) NO CHANGE
B) and
C) for
D) DELETE the underlined portion.

9
Which choice most effectively sets up the examples
given at the end of the sentence?
A) NO CHANGE
B) During periods of economic recession,
C) Although their value cannot be measured,
D) When it comes to the free services libraries
provide,

Unauthorized copying or reuse of any part of this page is illegal. 20 CO NTI N U E


2 2
searches as well as résumé and job material development.

...............................................................................................................................................................................................................................................................................................................
10
An overwhelming number of public libraries also report A) NO CHANGE
B) legal issues,
that they provide help with electronic government
C) concerns related to law courts,
resources related to income taxes, 10 law troubles, and
D) matters for the law courts,
retirement programs.
In sum, the Internet does not replace the need for 11
librarians, and librarians are hardly obsolete. 11 Like Which choice most clearly ends the passage with a
restatement of the writer’s primary claim?
books, librarians have been around for a long time, but
A) NO CHANGE
the Internet is extremely useful for many types of B) Although their roles have diminished
research. significantly, librarians will continue to be
employed by public libraries for the foreseeable
future.
C) The growth of electronic information has led to a
diversification of librarians’ skills and services,
positioning them as savvy resource specialists for
patrons.
D) However, given their extensive training and
skills, librarians who have been displaced by
budget cuts have many other possible avenues
of employment.

Unauthorized copying or reuse of any part of this page is illegal. 21 CO NTI N U E


2 2
Questions 12-22 are based on the following passage.

...............................................................................................................................................................................................................................................................................................................
12
A) NO CHANGE
Tiny Exhibit, Big Impact B) For instance,

—1— C) However,
D) Similarly,
The first time I visited the Art Institute of Chicago,
I expected to be impressed by its famous large paintings.
13
12 On one hand, I couldn’t wait to view 13 painter, A) NO CHANGE
Georges Seurat’s, 10-foot-wide A Sunday Afternoon on B) painter, Georges Seurat’s
the Island of La Grande Jatte in its full size. It took me by C) painter Georges Seurat’s,
D) painter Georges Seurat’s
surprise, then, when my favorite exhibit at the museum
was one of 14 it’s tiniest; the Thorne Miniature Rooms.
14
A) NO CHANGE
B) its tiniest;
C) its tiniest:
D) it’s tiniest,

Unauthorized copying or reuse of any part of this page is illegal. 22 CO NTI N U E


2 2
—2—

...............................................................................................................................................................................................................................................................................................................
15
Viewing the exhibit, I was amazed by the intricate At this point, the writer is considering adding the
following sentence.
details of some of the more ornately decorated rooms.
Some scholars argue that the excesses of
I marveled at a replica of a salon (a formal living room) King Louis XV’s reign contributed
significantly to the conditions that resulted
dating back to the reign of French king Louis XV.
in the French Revolution.
15 Built into the dark paneled walls are bookshelves Should the writer make this addition here?
stocked with leather-bound volumes. The couch and A) Yes, because it provides historical context for the
Thorne Miniature Rooms exhibit.
chairs, in keeping with the style of the time, are
B) Yes, because it explains why salons are often
characterized by elegantly curved arms and 16 legs, they ornately decorated.
are covered in luxurious velvet. A dime-sized portrait of a C) No, because it interrupts the paragraph’s
description of the miniature salon.
French aristocratic woman hangs in a golden frame.
D) No, because it implies that the interior designer
—3— of the salon had political motivations.
This exhibit showcases sixty-eight miniature rooms
inserted into a wall at eye level. Each furnished room 16
A) NO CHANGE
consists of three walls; the fourth wall is a glass pane
B) legs, the couch and chairs
through which museumgoers observe. The rooms and
C) legs and
their furnishings were painstakingly created to scale at D) legs,
1/12th their actual size, so that one inch in the exhibit
correlates with one foot in real life. A couch, for example, 17

is seven inches long, and 17 that is based on a Which choice gives a second supporting example
that is most similar to the example already in the
seven-foot-long couch. Each room represents a sentence?
distinctive style of European, American, or Asian interior A) NO CHANGE
B) a tea cup is about a quarter of an inch.
design from the thirteenth to twentieth centuries.
C) there are even tiny cushions on some.
D) household items are also on this scale.

Unauthorized copying or reuse of any part of this page is illegal. 23 CO NTI N U E


2 2
—4—

...............................................................................................................................................................................................................................................................................................................
18
The plainer rooms are more sparsely 18 furnished. Which choice most effectively combines the
sentences at the underlined portion?
Their architectural features, furnishings, and decorations
A) furnished by their
are just as true to the periods they represent. One of my B) furnished, but their
favorite rooms in the whole exhibit, in fact, is an 1885 C) furnished: their
summer kitchen. The room is simple but spacious, with a D) furnished, whereas

small sink and counter along one wall, a cast-iron wood


19
stove and some hanging pots and pans against another
Which choice most closely matches the stylistic
wall, and 19 a small table under a window of the third pattern established earlier in the sentence?
wall. Aside from a few simple wooden chairs placed near A) NO CHANGE
the edges of the room, the floor is open and obviously B) a small table is under the third wall’s window.
C) the third wall has a window and small table.
well worn.
D) the third wall has a small table against it and a
window.

Unauthorized copying or reuse of any part of this page is illegal. 24 CO NTI N U E


2 2
—5—

...............................................................................................................................................................................................................................................................................................................
20
As I walked through the exhibit, I overheard a A) NO CHANGE
B) visitors remarking,
20 visitors’ remark, “You know, that grandfather clock
C) visitor remarked,
actually runs. Its glass door swings open, and the clock
D) visitor remark,
can be wound up.” 21 Dotted with pin-sized knobs,
another visitor noticed my fascination with a tiny writing 21
desk and its drawers. “All of those little drawers pull out. A) NO CHANGE
B) Another visitor, dotted with pin-sized knobs,
And you see that hutch? Can you believe it has a secret
noticed my fascination with a tiny writing desk
compartment?” Given the exquisite craftsmanship and and its drawers.
C) Another visitor dotted with pin-sized knobs
level of detail I’d already seen, I certainly could.
noticed my fascination with a tiny writing desk
and its drawers.
D) Another visitor noticed my fascination with a
Question 22 asks about the previous passage as a tiny writing desk and its drawers, dotted with
whole. pin-sized knobs.

Think about the previous passage as a whole as you


answer question 22.
22
To make the passage most logical, paragraph 2
should be placed
A) where it is now.
B) after paragraph 3.
C) after paragraph 4.
D) after paragraph 5.

Unauthorized copying or reuse of any part of this page is illegal. 25 CO NTI N U E


2 2
Questions 23-33 are based on the following passage

...............................................................................................................................................................................................................................................................................................................
23
and supplementary material.
A) NO CHANGE
B) living along the West Coast of North America,
Environmentalist Otters they help
C) that live along the West Coast of North America
It has long been known that the sea otters 23 living
and help to
along the West Coast of North America help keep kelp D) that live along the West Coast of North America,
where they help
forests in their habitat healthy and vital. They do this by
feeding on sea urchins and other herbivorous
24
invertebrates that graze voraciously on kelp. With sea
Which choice offers an accurate interpretation of the
otters to keep the population of sea urchins in check, kelp data in the chart?
forests can flourish. In fact, 24 two years or less of sea A) NO CHANGE
B) even two years or less of sea otter presence can
otters can completely eliminate sea urchins in a coastal reduce the sea urchin threat
area (see chart). C) kelp density increases proportionally as sea
urchin density increases
Effects of Sea Otter Presence on Kelp D) even after sea otters were present for ten years or
and Sea Urchin Density in Coastal Areas more, kelp density was still lower than sea urchin
60 density
(number per square meter)

50 sea urchins
kelp
40 25
Density

A) NO CHANGE
30
B) however,
20 C) hence,
10 D) likewise,
0
no otters otters present otters present
present for 2 years for 10 years
or less or more
Coastal area
Adapted from David O. Duggins, “Kelp Beds and Sea Otters: An
Experimental Approach.” ©1980 by the Ecological Society of America.

Without sea otters present, 25 nevertheless, kelp forests


run the danger of becoming barren stretches of coastal
wasteland known as urchin barrens.

Unauthorized copying or reuse of any part of this page is illegal. 26 CO NTI N U E


2 2
[1] What was less well-known, until recently at least,

...............................................................................................................................................................................................................................................................................................................
26
was how this relationship among sea otters, sea urchins, At this point, the writer is considering adding the
following information.
and kelp forests might help fight global warming. [2] The
since the start of the Industrial Revolution,
amount of carbon dioxide in the atmosphere has resulting in a rise in global temperatures
increased 40 percent 26 . [3] A recent study by two Should the writer make this addition here?
professors at the University of California, Santa Cruz, A) Yes, because it establishes the relationship
between the level of carbon dioxide in the
Chris Wilmers and James Estes, 27 suggests, that kelp atmosphere and global warming.
forests protected by sea otters can absorb as much as B) Yes, because it explains the key role sea otters,
sea urchins, and kelp forests play in combating
twelve times the amount of carbon dioxide from the global warming.
atmosphere as those where sea urchins are allowed to C) No, because it contradicts the claim made in the
previous paragraph that sea otters help keep kelp
28 devour the kelp. [4] Like 29 their terrestrial plant
forests healthy.
cousins, kelp removes carbon dioxide from the D) No, because it mentions the Industrial
Revolution, blurring the focus of the paragraph.
atmosphere, turning it into sugar fuel through
photosynthesis, and releases oxygen back into the air.
27
A) NO CHANGE
B) suggests—that
C) suggests, “that
D) suggests that

28
A) NO CHANGE
B) dispatch
C) overindulge on
D) dispose of

29
A) NO CHANGE
B) they’re
C) its
D) it’s

Unauthorized copying or reuse of any part of this page is illegal. 27 CO NTI N U E


2 2
[5] Scientists knew this but did not recognize 30 how

...............................................................................................................................................................................................................................................................................................................
30
large a role they played in helping kelp forests to A) NO CHANGE
B) how large a role that it played
significantly decrease the amount of carbon dioxide in
C) how large a role sea otters played
the atmosphere. [6] Far from making no difference to the
D) that they played such a large role
ecosystem, the presence of otters was found to increase
the carbon storage of kelp forests by 4.4 to 8.7 megatons 31
annually, offsetting the amount of carbon dioxide Where is the most logical place in this paragraph to
add the following sentence?
emitted by three million to six million passenger cars
What Wilmers and Estes discovered in their
each year. 31 study, therefore, surprised them.
Wilmers and Estes caution, however, that 32 having A) After sentence 1
more otters will not automatically solve the problem of B) After sentence 3
C) After sentence 4
higher levels of carbon dioxide in the air. But they suggest
D) After sentence 5
that the presence of otters provides a good model of how
carbon can be sequestered, 33 or removed; from the
32
atmosphere through the management of animal A) NO CHANGE
populations. If ecologists can better understand what B) increasing the otter population
C) the otters multiplying
kinds of impacts animals might have on the environment,
D) having more otters than other locations
Wilmers contends, “there might be opportunities for
win-win conservation scenarios, whereby animal species
33
are protected or enhanced, and carbon gets sequestered.” A) NO CHANGE
B) or removed from,
C) or, removed from,
D) or removed, from

Unauthorized copying or reuse of any part of this page is illegal. 28 CO NTI N U E


2 2
Questions 34-44 are based on the following passage.

...............................................................................................................................................................................................................................................................................................................
34
A) NO CHANGE
A Quick Fix in a Throwaway Culture B) from which
C) so that
Planned obsolescence, a practice 34 at which
D) whereby
products are designed to have a limited period of
35 usefulness, has been a cornerstone of manufacturing
35
strategy for the past 80 years. This approach increases A) NO CHANGE
sales, but it also stands in 36 austere contrast to a time B) usefulness—
when goods were produced to be durable. Planned C) usefulness;
D) usefulness
obsolescence wastes materials as well as energy in making
and shipping new products. It also reinforces the belief
36
that it is easier to replace goods than to mend them, as A) NO CHANGE
repair shops are rare and 37 repair methods are often B) egregious
specialized. In 2009, an enterprising movement, the C) unmitigated
D) stark
Repair Café, challenged this widely accepted belief.

37
Which choice provides information that best
supports the claim made by this sentence?
A) NO CHANGE
B) obsolete goods can become collectible items.
C) no one knows whether something will fall into
disrepair again.
D) new designs often have “bugs” that must be
worked out.

Unauthorized copying or reuse of any part of this page is illegal. 29 CO NTI N U E


2 2
[1] More like a 38 fair then an actual café, the first

...............................................................................................................................................................................................................................................................................................................
38
Repair Café took place in Amsterdam, the Netherlands. A) NO CHANGE
B) fair than
[2] It was the brainchild of former journalist Martine
C) fare than
Postma, 39 wanting to take a practical stand in a
D) fair, then
throwaway culture. [3] Her goals were
40 straightforward, however: reduce waste, maintain 39
and perpetuate knowledge and skills, and strengthen A) NO CHANGE

community. [4] Participants bring all manner of B) whom wants


C) who wanted
damaged articles—clothing, appliances, furniture, and
D) she wanted
more—to be repaired by a staff of volunteer specialists
including tailors, electricians, and carpenters. [5] Since 40
the inaugural Repair Café, others have been hosted in A) NO CHANGE
theater foyers, community centers, hotels, and B) straightforward, therefore:
C) straightforward, nonetheless:
auditoriums. [6] While 41 they await for service, patrons
D) straightforward:
can enjoy coffee and snacks and mingle with their
neighbors in need. 42 41
A) NO CHANGE
B) awaiting
C) they waited
D) waiting

42
To make this paragraph most logical, sentence 5
should be placed
A) where it is now.
B) before sentence 1.
C) after sentence 3.
D) after sentence 6.

Unauthorized copying or reuse of any part of this page is illegal. 30 CO NTI N U E


2 2
Though only about 3 percent of the Netherlands’

..............................................................................................................................................................................................................................................................
43
municipal waste ends up in landfills, Repair Cafés still At this point, the writer is considering adding the
following sentence.
raise awareness about what may otherwise be mindless
As the number of corporate and service-based
acts of waste by providing a venue for people to share and jobs has increased, the need for people who work
learn valuable skills that are in danger of being lost. 43 It with their hands has diminished.
Should the writer make this addition here?
is easy to classify old but fixable items as “junk” in an era
A) Yes, because it provides an example of specific
that places great emphasis on the next big thing. In repair skills being lost.
helping people consider how the goods they use on a B) Yes, because it elaborates on the statistic about
the Netherlands’ municipal waste.
daily basis work and are made, Repair Cafés restore a
C) No, because it blurs the paragraph’s focus by
sense of relationship between human beings and material introducing a topic that is not further explained.
goods. D) No, because it contradicts the claims made in the
rest of the paragraph.
Though the concept remained a local trend at first,
international Repair Cafés, all affiliated with the Dutch
44
Repair Café via its website, have since arisen in France, A) NO CHANGE
Germany, South Africa, the United States, and other B) in addition.
C) likewise.
countries 44 on top of that. The original provides a
D) DELETE the underlined portion, and end the
central source for start-up tips and tools, as well as sentence with a period.
marketing advice to new Repair Cafés. As a result, the
Repair Café has become a global network united by
common ideals. Ironically, innovators are now looking
back to old ways of doing things and applying them in
today’s cities in an effort to transform the way people
relate to and think about the goods they consume.

STOP
If you finish before time is called, you may check your work on this section only.
Do not turn to any other section.

Unauthorized copying or reuse of any part of this page is illegal. 31


SAT Practice Test #2: Worksheets

ANSWER KEY

Reading Test Answers Writing and Language Test Answers

1 A 12 D 23 A 34 A 45 D 1 B 12 B 23 A 34 D
2 B 13 A 24 B 35 D 46 C 2 B 13 D 24 B 35 A
3 C 14 B 25 C 36 B 47 A 3 A 14 C 25 B 36 D
4 A 15 C 26 B 37 B 48 B 4 A 15 C 26 A 37 A
5 D 16 A 27 D 38 D 49 D 5 D 16 C 27 D 38 B
6 B 17 C 28 D 39 C 50 D 6 D 17 B 28 A 39 C
7 D 18 C 29 D 40 A 51 D 7 B 18 B 29 C 40 D
8 D 19 A 30 B 41 B 52 A 8 D 19 A 30 C 41 D
9 B 20 B 31 C 42 D 9 B 20 D 31 D 42 C
10 D 21 C 32 B 43 C 10 B 21 D 32 B 43 C

11 D 22 C 33 B 44 B 11 C 22 B 33 D 44 D

READING TEST
WRITING AND
RAW SCORE
LANGUAGE TEST
(NUMBER OF
RAW SCORE
CORRECT ANSWERS)
(NUMBER OF
CORRECT ANSWERS)

Math Test Math Test


No Calculator Answers Calculator Answers

1 C 11 C 1 C 11 B 21 D 31 14

2 B 12 B 2 B 12 D
22 B 32 7

3 A 13 D 3 A 13 D
23 A 33 11

4 A 14 A 4 C 14 C
24 A 34 105

5 C 15 D 5 C 15 A
25 A 35 15

6 D 16 3, 6, or 9
6 B 16 B
26 D 36 32

7 A 17 19
7 D 17 C
27 D 37 3284

8 C 18 12
8 D 18 C
28 B 38 7500

9 B 19 6
9 A 19 B
29 B
10 C 20 1/4 or .25
10 B 20 C
30 A

MATH TEST MATH TEST


NO CALCULATOR CALCULATOR
RAW SCORE RAW SCORE
(NUMBER OF (NUMBER OF
CORRECT ANSWERS) CORRECT ANSWERS)

SAT Practice Test #2 Created 8/27/15 6


SAT Practice Test #3
®

IMPORTANT REMINDERS

A No. 2 pencil is required for the test. Sharing any questions with anyone
Do not use a mechanical pencil or pen. is a violation of Test Security
and Fairness policies and may result
in your scores being canceled.

This cover is representative of what you’ll see on test day.

THIS TEST BOOK MUST NOT BE TAKEN FROM THE ROOM. UNAUTHORIZED
REPRODUCTION OR USE OF ANY PART OF THIS TEST BOOK IS PROHIBITED.
© 2015 The College Board. College Board, SAT, and the acorn logo are registered trademarks of the College Board.
2 2
Writing and Language Test
35 M I NU TES, 4 4 QUESTIONS

Turn to Section 2 of your answer sheet to answer the questions in this section.

DIRECTIONS

Each passage below is accompanied by a number of questions. For some questions, you
will consider how the passage might be revised to improve the expression of ideas. For
other questions, you will consider how the passage might be edited to correct errors in
sentence structure, usage, or punctuation. A passage or a question may be accompanied by
one or more graphics (such as a table or graph) that you will consider as you make revising
and editing decisions.

Some questions will direct you to an underlined portion of a passage. Other questions will
direct you to a location in a passage or ask you to think about the passage as a whole.

After reading each passage, choose the answer to each question that most effectively
improves the quality of writing in the passage or that makes the passage conform to the
conventions of standard written English. Many questions include a “NO CHANGE” option.
Choose that option if you think the best choice is to leave the relevant portion of the
passage as it is.

Questions 1-11 are based on the following passage.


...................................................................................................................................

1
A) NO CHANGE
Shed Some Light on the Workplace B) healthy, and more

Studies have shown that employees are happier, C) healthier, and they are
D) healthier, being more
1 healthier, and more productive when they work in an
environment 2 in which temperatures are carefully
2
controlled. New buildings may be designed with these
Which choice provides the most appropriate
studies in mind, but many older buildings were not, introduction to the passage?
A) NO CHANGE
resulting in spaces that often depend primarily on
B) that affords them adequate amounts of natural
artificial lighting. While employers may balk at the light.
expense of reconfiguring such buildings to increase the C) that is thoroughly sealed to prevent energy loss.
D) in which they feel comfortable asking managers
amount of natural light, the investment has been shown
for special accommodations.
to be well worth it in the long run—for both employees
and employers.

Unauthorized copying or reuse of any part of this page is illegal. 16 CO NTI N U E


2 2

...............................................................................................................................................................................................................................................................................................................
For one thing, lack of exposure to natural light has a 3
significant impact on employees’ health. A study At this point, the writer is considering adding the
following sentence.
conducted in 2013 by Northwestern University in
Workers in offices with windows sleep an
Chicago showed that inadequate natural light could average of 46 minutes more per night than
workers in offices without windows.
result in eye strain, headaches, and fatigue, as well as
Should the writer make this addition here?
interference with the body’s circadian rhythms. 3
A) Yes, because it supplies quantitative data that
Circadian rhythms, which are controlled by the will be examined in the rest of the paragraph.
4 bodies biological clocks, influence body temperature, B) Yes, because it explains the nature of the bodily
functions referred to in the next sentence.
hormone release, cycles of sleep and wakefulness, and
C) No, because it interrupts the discussion of
other bodily functions. Disruptions of circadian rhythms circadian rhythms.
have been linked to sleep disorders, diabetes, depression, D) No, because it does not take into account
whether workers were exposed to sunlight
and bipolar disorder. Like any other health problems, outside the office.
these ailments can increase employee absenteeism,
which, in turn, 5 is costly for employers. Employees 4
A) NO CHANGE
who feel less than 100 percent and are sleep deprived are
B) bodies’ biological clocks’,
also less prone to work at their maximal productivity.
C) body’s biological clocks,
One company in California 6 gained a huge boost in its D) body’s biological clock’s,
employees’ morale when it moved from an artificially lit
distribution facility to one with natural illumination. 5
A) NO CHANGE
B) are
C) is being
D) have been

6
Which choice best supports the statement made in
the previous sentence?
A) NO CHANGE
B) saw a 5 percent increase in productivity
C) saved a great deal on its operational costs
D) invested large amounts of time and capital

Unauthorized copying or reuse of any part of this page is illegal. 17 CO NTI N U E


2 2

...............................................................................................................................................................................................................................................................................................................
7 Artificial light sources are also costly aside from 7
lowering worker productivity. They typically constitute In context, which choice best combines the
underlined sentences?
anywhere from 25 to 50 percent of a building’s energy
A) Aside from lowering worker productivity,
use. When a plant in Seattle, Washington, was redesigned artificial light sources are also costly, typically
constituting anywhere from 25 to 50 percent of a
for more natural light, the company was able to enjoy building’s energy use.
annual electricity cost reductions of $500,000 8 each B) The cost of artificial light sources, aside from
lowering worker productivity, typically
year. constitutes anywhere from 25 to 50 percent of a
building’s energy use.
C) Typically constituting 25 to 50 percent of a
building’s energy use, artificial light sources
lower worker productivity and are costly.
D) Artificial lights, which lower worker productivity
and are costly, typically constitute anywhere
from 25 to 50 percent of a building’s energy use.

8
A) NO CHANGE
B) every year.
C) per year.
D) DELETE the underlined portion and end the
sentence with a period.

Unauthorized copying or reuse of any part of this page is illegal. 18 CO NTI N U E


2 2

...............................................................................................................................................................................................................................................................................................................
Among the possibilities to reconfigure a building’s 9
lighting is the installation of full-pane windows to allow A) NO CHANGE
B) Nevertheless,
the greatest degree of sunlight to reach office interiors.
C) Alternatively,
9 Thus, businesses can install light tubes, 10 these are D) Finally,
pipes placed in workplace roofs to capture and funnel
sunlight down into a building’s interior. Glass walls and 10
dividers can also be used to replace solid walls as a means A) NO CHANGE
B) they are
11 through distributing natural light more freely.
C) which are
Considering the enormous costs of artificial lighting, D) those being
both in terms of money and productivity, investment in
such improvements should be a natural choice for 11
businesses. A) NO CHANGE
B) of
C) from
D) DELETE the underlined portion.

Unauthorized copying or reuse of any part of this page is illegal. 19 CO NTI N U E


2 2
Questions 12-22 are based on the following passage.

...............................................................................................................................................................................................................................................................................................................
12
A) NO CHANGE
Transforming the American West Through Food and B) himself or herself
Hospitality C) their selves
Just as travelers taking road trips today may need to D) oneself
take a break for food at a rest area along the highway,
settlers traversing the American West by train in the 13

mid-1800s often found 12 themselves in need of Which choice provides the most logical introduction
to the sentence?
refreshment. However, food available on rail lines was A) NO CHANGE
generally of terrible quality. 13 Despite having worked B) He had lived in New York and New Orleans, so

for railroad companies, Fred Harvey, an English-born C) To capitalize on the demand for good food,
D) DELETE the underlined portion.
14 entrepreneur. He decided to open his own restaurant
business to serve rail customers. Beginning in the 1870s,
14
he opened dozens of restaurants in rail stations and A) NO CHANGE
dining cars. These Harvey Houses, which constituted the B) entrepreneur:
first restaurant chain in the United States, 15 was unique C) entrepreneur; he
D) entrepreneur,
for its high standards of service and quality. The menu
was modeled after those of fine restaurants, so the food
15
was leagues beyond the 16 sinister fare travelers were A) NO CHANGE
accustomed to receiving in transit. B) were unique for their
C) was unique for their
D) were unique for its

16
Which choice best maintains the tone established in
the passage?
A) NO CHANGE
B) surly
C) abysmal
D) icky

Unauthorized copying or reuse of any part of this page is illegal. 20 CO NTI N U E


2 2

...............................................................................................................................................................................................................................................................................................................
His restaurants were immediately successful, but 17
Harvey was not content to follow conventional business The writer is considering deleting the previous
sentence. Should the writer make this change?
practices. 17 Although women did not traditionally
A) Yes, because it introduces information that is
work in restaurants in the nineteenth century, Harvey irrelevant at this point in the passage.
decided to try employing women as waitstaff. In 1883, he B) Yes, because it does not logically follow from the
previous paragraph.
placed an advertisement seeking educated, C) No, because it provides a logical introduction to
well-mannered, articulate young women between the the paragraph.
D) No, because it provides a specific example in
ages of 18 and 30. 18 Response to the advertisement was support of arguments made elsewhere in the
overwhelming, even tremendous, and Harvey soon passage.

replaced the male servers at his restaurants with women.


18
Those who were hired as “Harvey Girls” joined an elite
A) NO CHANGE
group of workers, who were expected to complete a
B) Response to the advertisement was
30-day training program and follow a strict code of rules overwhelming,
C) Overwhelming, even tremendous, was the
for conduct and curfews. In the workplace, the women
response to the advertisement,
donned identical black-and-white uniforms and carried D) There was an overwhelming, even tremendous,
response to the advertisement,
out their duties with precision. Not only were such
regulations meant to ensure the efficiency of the business
19
and the safety of the workers, 19 but also helped to raise
A) NO CHANGE
people’s generally low opinion of the restaurant industry. B) but also helping
C) also helping
D) but they also helped

Unauthorized copying or reuse of any part of this page is illegal. 21 CO NTI N U E


2 2
In return for the servers’ work, the position paid quite

...............................................................................................................................................................................................................................................................................................................
20
well for the time: $17.50 a month, plus tips, meals, room Which choice most logically follows the previous
sentence?
and board, laundry service, and travel expenses. 20
A) The growth of Harvey’s business coincided with
For as long as Harvey Houses served rail travelers the expansion of the Santa Fe Railway, which
served large sections of the American West.
through the mid-twentieth century, working there was a
B) Harvey would end up opening dozens of
steady and lucrative position for women. Living restaurants and dining cars, plus 15 hotels, over
his lucrative career.
independently and demonstrating an intense work
C) These benefits enabled the Harvey Girls to save
21 ethic; the Harvey Girls became known as a money and build new and exciting lives for
themselves in the so-called Wild West.
transformative force in the American 22 West.
D) The compensation was considered excellent at
Advancing the roles of women in the restaurant industry the time, though it may not seem like much
money by today’s standards.
and the American workforce as a whole, the Harvey Girls
raised the standards for restaurants and blazed a trail in
21
the fast-changing landscape of the western territories. A) NO CHANGE
B) ethic:
C) ethic, and
D) ethic,

22
The writer is considering revising the underlined
portion of the sentence to read:
West, inspiring books, documentaries, and even
a musical.
Should the writer add this information here?
A) Yes, because it provides examples of the Harvey
Girls’ influence.
B) Yes, because it serves as a transitional point in
the paragraph.
C) No, because it should be placed earlier in the
passage.
D) No, because it contradicts the main claim of the
passage.

Unauthorized copying or reuse of any part of this page is illegal. 22 CO NTI N U E


2 2

...............................................................................................................................................................................................................................................................................................................
Questions 23-33 are based on the following passage 23
and supplementary material.
Which choice most effectively combines the
underlined sentences?
How Do You Like Those Apples? A) When applied to apples, 1-MCP lengthens
storage life by three to four times, allowing
Marketed as SmartFresh, the chemical 1-MCP producers to sell their apples in the off-season,
(1-methylcyclopropene) has been used by fruit growers months after the apples have been harvested.
B) Producers are allowed to sell their apples
since 2002 in the United States and elsewhere to preserve months after they have been harvested—in the
the crispness and lengthen the storage life of apples and off-season—because 1-MCP, when applied to
apples, lengthens their storage life by three to
other fruit, which often must travel long distances before four times.
being eaten by consumers. 23 1-MCP lengthens storage C) 1-MCP lengthens storage life, when applied
to apples, by three to four times, allowing
life by three to four times when applied to apples. This producers to sell their apples months after the
apples have been harvested in the off-season.
extended life allows producers to sell their apples in the
D) Months after apples have been harvested,
off-season, months after the apples have been harvested. producers are allowed to sell their apples, in the
off-season, because 1-MCP lengthens storage life
And at a cost of about one cent per pound of apples, when applied to apples by three to four times.
1-MCP is a highly cost-effective treatment. However,
1-MCP is not a panacea for fruit producers or sellers:
there are problems and limitations associated with its use.

Unauthorized copying or reuse of any part of this page is illegal. 23 CO NTI N U E


2 2

...............................................................................................................................................................................................................................................................................................................
[1] 1-MCP works by limiting a fruit’s production of 24
ethylene, 24 it is a chemical that causes fruit to ripen and A) NO CHANGE
B) being
eventually rot. [2] While 1-MCP keeps apples 25 tight
C) that is
and crisp for months, it also limits 26 their scent D) DELETE the underlined portion.
production. [3] This may not be much of a problem with
certain kinds of apples that are not naturally very 25
fragrant, such as Granny Smith, but for apples that are A) NO CHANGE
B) firm
prized for their fruity fragrance, such as McIntosh, this
C) stiff
can be a problem with consumers, 27 that will reject
D) taut
apples lacking the expected aroma. [4] But some fruits do
not respond as well to 1-MCP as others 28 did, and 26
some even respond adversely. [5] Furthermore, some A) NO CHANGE
B) there
fruits, particularly those that naturally produce a large
C) its
D) it’s

27
A) NO CHANGE
B) they
C) which
D) who

28
A) NO CHANGE
B) do,
C) have,
D) will,

Unauthorized copying or reuse of any part of this page is illegal. 24 CO NTI N U E


2 2
amount of ethylene, do not respond as well to 1-MCP 29

...............................................................................................................................................................................................................................................................................................................
treatment. [6] Take Bartlett 29 pears, for instance, unless A) NO CHANGE
B) pears, for instance:
they are treated with exactly the right amount of 1-MCP
C) pears for instance,
at exactly the right time, they will remain hard and green
D) pears. For instance,
until they rot, and consumers who experience this will be
unlikely to purchase them again. 30 30
To make this paragraph most logical, sentence 4
should be placed
A) where it is now.
B) after sentence 1.
C) after sentence 2.
D) after sentence 5.

Unauthorized copying or reuse of any part of this page is illegal. 25 CO NTI N U E


2 2
Finally, researchers have found that 1-MCP actually 31

...............................................................................................................................................................................................................................................................................................................
increases susceptibility to some pathologies in certain Which choice offers an accurate interpretation of
the data in the graph?
apple varieties. For example, Empire apples are prone to a
A) NO CHANGE
condition that causes the flesh of the apple to turn brown. B) slightly more browning than
Traditionally, apple producers have dealt with this C) twice as much browning as
problem by leaving the apples in the open air for three D) substantially less browning than

weeks before storing them in a controlled atmosphere


32
with tightly regulated temperature, humidity, and carbon
Which choice offers an accurate interpretation of
dioxide levels. As the graph shows, the flesh of untreated the data in the graph?
Empire apples that are first stored in the open air A) NO CHANGE
B) roughly half of their flesh turns brown,
undergoes 31 roughly five percent less browning than
regardless of whether the apples are first stored
the flesh of untreated Empire apples that are immediately in the open air.
C) their flesh browns when they are put directly
put into storage in a controlled environment. However,
into a controlled atmosphere but not when they
when Empire apples are treated with 1-MCP, 32 their are first stored in the open air.
D) their flesh turns brown when they are first stored
flesh turns brown when the apples are first stored in the
in the open air, though not as quickly as the
open air, though not under other conditions. Although apple flesh in an untreated group does.

Unauthorized copying or reuse of any part of this page is illegal. 26 CO NTI N U E


2 2

...............................................................................................................................................................................................................................................................................................................
researchers continue to search for the right combination 33
of factors that will keep fruits fresh and attractive, 33 the The writer wants a conclusion that conveys how the
shortcomings of 1-MCP presented in the passage
problem may be that consumers are overly concerned affect the actions of people in the fruit industry.
Which choice best accomplishes this goal?
with superficial qualities rather than the actual freshness
A) NO CHANGE
of the fruit.
B) many of the improvements to fruit quality they
have discovered so far have required trade-offs
Results of Treatment to Control in other properties of the fruit.
Browning of Empire Apples C) for now many fruit sellers must weigh the
relative values of aroma, color, and freshness
60 untreated 1-MCP
Percentage of flesh browning

when deciding whether to use 1-MCP.


50 D) it must be acknowledged that 1-MCP, despite
some inadequacies, has enabled the fruit
40
industry to ship and store fruit in ways that
30 were impossible before.

20
10
0
apples placed apples placed in
immediately controlled atmosphere
in controlled after three weeks
atmosphere in open air
Adapted from Hannah J. James, Jacqueline F. Nock, and Chris B. Watkins,
“The Failure of Postharvest Treatments to Control Firm Flesh Browning in
Empire Apples.” ©2010 by The New York State Horticultural Society.

Unauthorized copying or reuse of any part of this page is illegal. 27 CO NTI N U E


2 2
Questions 34-44 are based on the following passage.

...............................................................................................................................................................................................................................................................................................................
34
A) NO CHANGE
More than One Way to Dress a Cat B) Gothic. Works
C) Gothic; these works
From Michelangelo’s David to Vincent van Gogh’s
D) Gothic, works
series of self-portraits to Grant Wood’s iconic image of
a farming couple in American 34 Gothic. These works by
35
human artists have favored representations of members A) NO CHANGE
of their own species to those of other species. Indeed, B) Coolidge—
when we think about animals depicted in well-known C) Coolidge;
D) Coolidge
works of art, the image of dogs playing
poker—popularized in a series of paintings by American
36
artist C. M. 35 Coolidge, may be the first and only one A) NO CHANGE
that comes to mind. Yet some of the earliest known B) portraying
works of art, including paintings and drawings tens of C) portray
D) has portrayed
thousands of years old found on cave walls in Spain and
France, 36 portrays animals. Nor has artistic homage to
37
our fellow creatures entirely died out in the millennia
The writer wants to link the first paragraph with the
since, 37 despite the many years that have passed ideas that follow. Which choice best accomplishes
this goal?
between then and now.
A) NO CHANGE
B) with special attention being paid to domestic
animals such as cats.
C) even though most paintings in museums are of
people, not animals.
D) as the example of one museum in Russia shows.

Unauthorized copying or reuse of any part of this page is illegal. 28 CO NTI N U E


2 2
[1] The State Hermitage Museum in St. Petersburg,

...............................................................................................................................................................................................................................................................................................................
38
one of Russia’s greatest art museums, has long had a A) NO CHANGE
B) scaring
productive partnership with a much loved animal: the
C) scare
cat. [2] For centuries, cats have guarded this famous
D) have scared
museum, ridding it of mice, rats, and other rodents that
could damage the art, not to mention 38 scared off 39
visitors. [3] Peter the Great introduced the first cat to the To make this paragraph most logical, sentence 5
should be placed
Hermitage in the early eighteenth century. [4] Later
A) where it is now.
Catherine the Great declared the cats to be official
B) after sentence 1.
guardians of the galleries. [5] Continuing the tradition, C) after sentence 3.
Peter’s daughter Elizaveta introduced the best and D) after sentence 6.

strongest cats in Russia to the Hermitage. [6] Today, the


museum holds a yearly festival honoring these faithful
workers. 39

Unauthorized copying or reuse of any part of this page is illegal. 29 CO NTI N U E


2 2

...............................................................................................................................................................................................................................................................................................................
These cats are so cherished by the museum that 40
officials recently 40 decreed original paintings to be A) NO CHANGE
B) commissioned
made of six of them. In each, a cat is depicted upright in a
C) forced
humanlike pose and clothed in imperial-era Russian
D) licensed
attire. The person chosen for this 41 task, digital artist,
Eldar Zakirov painted the cats in the style traditionally 41
used by portrait artists, in so doing 42 presenting the A) NO CHANGE
B) task, digital artist, Eldar Zakirov,
cats as noble individuals worthy of respect. One portrait,
C) task digital artist Eldar Zakirov,
The Hermitage Court Chamber Herald Cat, includes an
D) task, digital artist Eldar Zakirov,

42
Which choice most effectively sets up the examples
that follow?
A) NO CHANGE
B) managing to capture unique characteristics of
each cat.
C) commenting on the absurdity of dressing up cats
in royal robes.
D) indicating that the cats were very talented mouse
catchers.

Unauthorized copying or reuse of any part of this page is illegal. 30 CO NTI N U E


2 2
aristocratic tilt of feline ears as well as a stately sweep of

.....................................................................................................................................................................................................................................................
43
tail emerging from the stiff scarlet and gold of royal court At this point, the writer is considering adding the
following sentence.
dress. The wise, thoughtful green eyes of the subject of
The museum occupies six historic buildings,
The Hermitage Court Outrunner Cat mimic those of a including the Winter Palace, a former residence
trusted royal advisor. 43 Some may find it peculiar to of Russian emperors.
Should the writer make this addition here?
observe cats portrayed in formal court poses, but these
A) Yes, because it shows the link between
felines, by 44 mastering the art of killing mice and rats, Peter the Great and the cat paintings.
are benefactors of the museum as important as any B) Yes, because it helps explain why Russian art
celebrates animals.
human.
C) No, because it fails to indicate why the
Winter Palace became an art museum.
D) No, because it provides background information
that is irrelevant to the paragraph.

44
A) NO CHANGE
B) acting as the lead predator in the museum’s
ecosystem,
C) hunting down and killing all the mice and rats
one by one,
D) protecting the museum’s priceless artworks from
destructive rodents,

STOP
If you finish before time is called, you may check your work on this section only.
Do not turn to any other section.

Unauthorized copying or reuse of any part of this page is illegal. 31


SAT Practice Test #3: Worksheets

ANSWER KEY

Reading Test Answers Writing and Language Test Answers

1 B 12 C 23 A 34 D 45 D 1 A 12 A 23 A 34 D
2 C 13 D 24 B 35 C 46 C 2 B 13 C 24 D 35 B
3 A 14 B 25 C 36 A 47 B 3 C 14 D 25 B 36 C
4 A 15 B 26 C 37 D 48 B 4 C 15 B 26 A 37 D
5 C 16 C 27 B 38 C 49 A 5 A 16 C 27 D 38 C
6 A 17 B 28 B 39 A 50 B 6 B 17 C 28 B 39 C
7 A 18 B 29 D 40 D 51 D 7 A 18 B 29 B 40 B
8 B 19 A 30 D 41 A 52 D 8 D 19 D 30 B 41 D
9 B 20 A 31 B 42 C 9 C 20 C 31 D 42 A
10 D 21 D 32 C 43 C 10 C 21 D 32 B 43 D

11 A 22 A 33 C 44 D 11 B 22 A 33 C 44 D

READING TEST
WRITING AND
RAW SCORE
LANGUAGE TEST
(NUMBER OF
RAW SCORE
CORRECT ANSWERS)
(NUMBER OF
CORRECT ANSWERS)

Math Test Math Test


No Calculator Answers Calculator Answers

1 C 11 B 1 C 11 B 21 C 31 4 or 5

2 D 12 A 2 B 12 D
22 B 32 58.6
3 D 13 B 3 C 13 D 23 C 33 9

4 B 14 A 4 C 14 A
24 D 34 5/8 or 0.625
5 C 15 D 5 B 15 A 25 D 35 50

6 C 16 1 or 2
6 A 16 B
26 C 36 750

7 C 17 2
7 D 17 B
27 C 37 7

8 A 18 105
8 C 18 B
28 D 38 60

9 A 19 370
9 B 19 C
29 A
10 A 20 3/5 or 0.6 10 D 20 B
30 A

MATH TEST MATH TEST


NO CALCULATOR CALCULATOR
RAW SCORE RAW SCORE
(NUMBER OF (NUMBER OF
CORRECT ANSWERS) CORRECT ANSWERS)

SAT Practice Test #3 Created 8/4/2015 6


SAT Practice Test #4
®

IMPORTANT REMINDERS

A No. 2 pencil is required for the test. Sharing any questions with anyone
Do not use a mechanical pencil or pen. is a violation of Test Security
and Fairness policies and may result
in your scores being canceled.

This cover is representative of what you’ll see on test day.

THIS TEST BOOK MUST NOT BE TAKEN FROM THE ROOM. UNAUTHORIZED
REPRODUCTION OR USE OF ANY PART OF THIS TEST BOOK IS PROHIBITED.
© 2015 The College Board. College Board, SAT, and the acorn logo are registered trademarks of the College Board.
2 2
Writing and Language Test
35 M I NU TES, 4 4 QUESTIONS

Turn to Section 2 of your answer sheet to answer the questions in this section.

DIRECTIONS

Each passage below is accompanied by a number of questions. For some questions, you
will consider how the passage might be revised to improve the expression of ideas. For
other questions, you will consider how the passage might be edited to correct errors in
sentence structure, usage, or punctuation. A passage or a question may be accompanied by
one or more graphics (such as a table or graph) that you will consider as you make revising
and editing decisions.

Some questions will direct you to an underlined portion of a passage. Other questions will
direct you to a location in a passage or ask you to think about the passage as a whole.

After reading each passage, choose the answer to each question that most effectively
improves the quality of writing in the passage or that makes the passage conform to the
conventions of standard written English. Many questions include a “NO CHANGE” option.
Choose that option if you think the best choice is to leave the relevant portion of the
passage as it is.

Questions 1-11 are based on the following passage.


...................................................................................................................................

1
A) NO CHANGE
Ghost Mural B) which he accordingly titled
In 1932 the well-known Mexican muralist David C) accordingly he titled it
D) it was titled accordingly
Alfaro Siqueiros was commissioned to paint a mural on
the second-story exterior wall of a historic building in
2
downtown Los Angeles. Siqueiros was asked to celebrate A) NO CHANGE
tropical America in his work, 1 he accordingly titled it B) However,
“América Tropical.” He painted the mural’s first two C) Although,
D) Moreover,
sections, featuring images of a tropical rainforest and a
Maya pyramid, during the day. 2 Also, to avoid

Unauthorized copying or reuse of any part of this page is illegal. 16 CO NTI N U E


2 2

...............................................................................................................................................................................................................................................................................................................
scrutiny, Siqueiros painted the final section of the mural, 3
the 3 centerpiece at night. A) NO CHANGE
B) centerpiece,
4 The reason for Siqueiros’s secrecy became clear
C) centerpiece;
when the mural was 5 confided. The centerpiece of the D) centerpiece—
work was dominated by images of native people being
oppressed and 6 including an eagle symbolizing the 4
United States. Siqueiros’s political message did not please Which choice best connects the sentence with the
previous paragraph?
the wealthy citizens who had commissioned his work.
A) NO CHANGE
They eventually ordered the mural to be literally B) All three sections of the mural were on display
whitewashed, or painted over with white paint. C) The community turned out in large numbers

However, by the 1970s, the white paint had begun to D) Siqueiros was informed of people’s reactions

fade, and the bright colors of the mural were beginning to


5
show through. At the same time, a social and civil rights
A) NO CHANGE
movement for Mexican Americans was working to raise B) promulgated.
awareness of Mexican American cultural identity. Artists C) imparted.
associated with 7 this began to rediscover and promote D) unveiled.

the work of the Mexican muralists, particularly Siqueiros.


6
To them, “América Tropical” was an example of how art
A) NO CHANGE
in public spaces could be used to celebrate Mexican B) included
American heritage while at the same time making a C) includes
political statement. Inspired by Siqueiros and the other D) had included

muralists, this new generation of artists strove to emulate


7
the old mural masters.
A) NO CHANGE
B) it
C) them
D) this movement

Unauthorized copying or reuse of any part of this page is illegal. 17 CO NTI N U E


2 2
8 The result was an explosion of mural painting

...............................................................................................................................................................................................................................................................................................................
8
that spread throughout California and the southwestern Which choice most effectively combines the
underlined sentences?
United States in the 1970s. It was the Chicano mural
A) The result was an explosion, the Chicano mural
movement. Hundreds of large, colorful new murals movement, of mural painting that spread
throughout California and the southwestern
depicting elements of Mexican American life and history United States in the 1970s.
appeared during this period, some in designated cultural B) The result was the Chicano mural movement, an
explosion of mural painting that spread
locations but many more in abandoned lots, on unused throughout California and the southwestern
buildings, or 9 painted on infrastructure such as United States in the 1970s.
C) The explosion of mural painting that spread
highways and bridges. Many of these murals can still be throughout California and the southwestern
seen today, although some have not been well United States in the 1970s was the resulting
Chicano mural movement.
maintained. D) An explosion of mural painting resulted and it
spread throughout California and the
southwestern United States in the 1970s; it was
the Chicano mural movement.

9
A) NO CHANGE
B) they were painted on
C) on
D) DELETE the underlined portion.

Unauthorized copying or reuse of any part of this page is illegal. 18 CO NTI N U E


2 2
Fortunately, a new group of artists has discovered the

...............................................................................................................................................................................................................................................................................................................
10
murals, and efforts are underway to clean, restore, and Which choice most effectively sets up the
information that follows?
repaint them. Once again, Siqueiros’s “América Tropical”
A) NO CHANGE
is 10 leading the way. After a lengthy and complex B) being cleaned and restored.
restoration process, this powerful work is now a tourist C) at risk of destruction.
attraction, complete with a visitor center and a rooftop D) awaiting its moment of appreciation.

viewing platform. 11 Advocates hope that Siqueiros’s


11
mural will once more serve as an inspiration, this time
At this point, the writer is considering adding the
inspiring viewers to save and restore an important following sentence.
cultural and artistic legacy. When it was painted in 1932, Siqueiros’s mural
was considered offensive, but now it is
acclaimed.
Should the writer make this addition here?
A) Yes, because it provides historical context for the
changes discussed in the passage.
B) Yes, because it provides a useful reminder of
how people once viewed Siqueiros’s work.
C) No, because it unnecessarily repeats information
from earlier in the passage.
D) No, because it makes a claim about Siqueiros’s
work that is not supported by the passage.

Unauthorized copying or reuse of any part of this page is illegal. 19 CO NTI N U E


2 2
Questions 12-22 are based on the following passage.

...............................................................................................................................................................................................................................................................................................................
12
A) NO CHANGE
The Hype of Healthier Organic Food B) the purchase of
C) purchasing
Some people buy organic food because they believe
D) DELETE the underlined portion.
organically grown crops are more nutritious and safer for
consumption than 12 the people who purchase their
13
conventionally grown counterparts, which are usually A) NO CHANGE
produced with pesticides and synthetic fertilizers. In the B) these consumers spend
name of health, 13 spending $1.60 for every dollar they C) having spent
D) to spend
would have spent on food that is 14 grown in a manner
that is considered conventional. Scientific evidence,
14
15 therefore, suggests that consumers do not reap A) NO CHANGE
significant benefits, in terms of either nutritional value or B) grown with conventional methods, using
pesticides and synthetic fertilizers.
safety, from organic food.
C) conventionally and therefore not organically
grown.
D) conventionally grown.

15
A) NO CHANGE
B) furthermore,
C) however,
D) subsequently,

Unauthorized copying or reuse of any part of this page is illegal. 20 CO NTI N U E


2 2
Although advocates of organic food 16 preserve that

...............................................................................................................................................................................................................................................................................................................
16
organic produce is healthier than conventionally grown A) NO CHANGE
B) carry on
produce because it has more vitamins and minerals, this
C) maintain
assertion is not supported by scientific research. 17 For
D) sustain
instance, one review published in The American Journal
of Clinical Nutrition provided analysis of the results of 17
comparative studies conducted over a span of 50 years; A) NO CHANGE
B) However,
researchers consistently found no evidence that organic
C) In addition,
crops are more nutritious than conventionally grown
D) Likewise,
ones in terms of their vitamin and mineral content. 18
Similarly, Stanford University researchers who examined 18
almost 250 studies comparing the nutritional content of At this point, the writer is considering adding the
following sentence.
different kinds of organic foods with that of their
The United States Department of Agriculture
nonorganic counterparts found very little difference (USDA) reports that organic agricultural
between the two. products are now available in approximately
20,000 markets specializing in natural foods.
Should the writer make this addition here?
A) Yes, because it adds a relevant research finding
from a government agency.
B) Yes, because it supports the passage’s argument
that organic food is less nutritious than
conventionally grown food.
C) No, because it is not relevant to the paragraph’s
discussion of scientific evidence.
D) No, because it introduces a term that has not
been defined in the passage.

Unauthorized copying or reuse of any part of this page is illegal. 21 CO NTI N U E


2 2
Evidence also undermines the claim that organic

...............................................................................................................................................................................................................................................................................................................
19
food is safer to eat. While researchers have found lower A) NO CHANGE
B) is having
levels of pesticide residue in organic produce than in
C) has had
nonorganic produce, the pesticide residue detected in
D) has
conventional produce falls within acceptable safety limits.
According to such organizations as the US 20
Environmental Protection Agency, the minute amounts At this point, the writer wants to further reinforce
the paragraph’s claim about the safety of nonorganic
of residue falling within such limits 19 have no negative food. Which choice most effectively accomplishes
impact on human health. 20 this goal?
A) To be labeled organic, a product must meet
certain standards determined and monitored by
the US Department of Agriculture.
B) Organic food, however, is regulated to eliminate
artificial ingredients that include certain types of
preservatives, sweeteners, colorings, and flavors.
C) Moreover, consumers who are concerned about
ingesting pesticide residue can eliminate much
of it by simply washing or peeling produce
before eating it.
D) In fact, the Environmental Protection Agency
estimates that about one-fifth of the pesticides
used worldwide are applied to crops in the
United States.

Unauthorized copying or reuse of any part of this page is illegal. 22 CO NTI N U E


2 2

...............................................................................................................................................................................................................................................................................................................
Based on scientific evidence, organic food offers 21
neither significant nutritional nor safety benefits for A) NO CHANGE
B) there are
consumers. Proponents of organic food, of course, are
C) there is
quick to add that 21 their are numerous other reasons to D) their is
buy organic 22 food, such as, a desire to protect the
environment from potentially damaging pesticides or a 22
preference for the taste of organically grown foods. A) NO CHANGE
B) food such as:
Research regarding these issues is less conclusive than the
C) food such as,
findings regarding nutritional content and pesticide D) food, such as
residue safety limits. What is clear, though, is this: if a
consumer ’s goal is to buy the healthiest and safest food to
eat, the increased cost of organic food is a waste of
money.

Unauthorized copying or reuse of any part of this page is illegal. 23 CO NTI N U E


2 2
Questions 23-33 are based on the following passage

...............................................................................................................................................................................................................................................................................................................
23
and supplementary material.
The writer wants to convey an attitude of genuine
interest and to avoid the appearance of mockery.
You Are Where You Say Which choice best accomplishes this goal?
A) NO CHANGE
Research on regional variations in English-language
B) galvanizing
use has not only yielded answers to such 23 life-altering C) intriguing
questions as how people in different parts of the D) weird
United States refer to carbonated beverages (“soda”?
“pop”? “coke”?) 24 it also illustrates how technology can 24
A) NO CHANGE
change the very nature of research. While traditional,
B) and also illustrates
human-intensive data collection 25 has all but C) but also illustrates
disappeared in language studies, the explosion of social D) illustrating
media has opened new avenues for investigation.
[1] Perhaps the epitome of traditional methodology 25

is the Dictionary of American Regional English, Which choice most effectively sets up the contrast in
the sentence and is consistent with the information
colloquially known as DARE. [2] Its fifth and final in the rest of the passage?
alphabetical volume—ending with “zydeco”—released in A) NO CHANGE
B) still has an important place
2012, the dictionary represents decades of arduous work.
C) remains the only option
[3] Over a six-year period from 1965 to 1970, university D) yields questionable results
graduate students conducted interviews in more than a
thousand communities across the nation. [4] Their goal
was to determine what names people used for such
everyday objects and concepts as a submarine sandwich

Unauthorized copying or reuse of any part of this page is illegal. 24 CO NTI N U E


2 2
(a “hero” in New York City but a “dagwood” in many

...............................................................................................................................................................................................................................................................................................................
26
parts of Minnesota, Iowa, and Colorado) and a heavy A) NO CHANGE
B) scholars, and these scholars
rainstorm (variously a “gully washer,” “pour-down,” or
C) scholars, but scholars
“stump mover”). [5] The work that dictionary founder
D) scholars, who
Frederic G. Cassidy had expected to be finished by 1976
was not, in fact, completed in his lifetime. [6] The wait 27
did not dampen enthusiasm among 26 scholars. To improve the cohesion and flow of this paragraph,
the writer wants to add the following sentence.
Scholars consider the work a signal achievement in
Data gathering proved to be the quick part of the
linguistics. 27 project.
Not all research into regional English varieties The sentence would most logically be placed after
28 requires such time, effort, and resources, however. A) sentence 2.
B) sentence 3.
Today’s researchers have found that the veritable army of
C) sentence 4.
trained volunteers traveling the country conducting
D) sentence 5.
face-to-face interviews can sometimes be 29 replaced by
another army the vast array of individuals volunteering 28
details about their lives—and, inadvertently, their A) NO CHANGE
language—through social media. Brice Russ of Ohio State B) are requiring
C) have required
University, for example, has employed software to sort
D) require
through postings on one social media 30 cite in search
of particular words and phrases of interest as well as the 29
location from which users are posting. From these data, A) NO CHANGE
B) replaced—by another army,
C) replaced by another army;
D) replaced by another army:

30
A) NO CHANGE
B) site in search of
C) sight in search for
D) cite in search for

Unauthorized copying or reuse of any part of this page is illegal. 25 CO NTI N U E


2 2

...............................................................................................................................................................................................................................................................................................................
he was able, among other things, to confirm regional 31
variations in people’s terms for soft drinks. As the map The writer wants the information in the passage
to correspond as closely as possible with the
shows, “soda” is commonly heard in the middle and information in the map. Given that goal and
western portions of the United States; “pop” is frequently assuming that the rest of the previous sentence
would remain unchanged, in which sequence
used in many southern states; and “coke” is predominant should the three terms for soft drinks be discussed?
in the northeastern and southwest regions but used A) NO CHANGE
B) “pop,” “soda,” “coke”
elsewhere as well. 31 As interesting as Russ’s findings
C) “pop,” “coke,” “soda”
are, though, 32 they’re true value lies in their reminder D) “soda,” “coke,” “pop”
that the Internet is not merely a sophisticated tool for
collecting data but is also 33 itself a rich source of data. 32
A) NO CHANGE
Soft Drink Descriptions by State B) their true value lies in their
Highest Percentage Reported
C) there true value lies in they’re
D) their true value lies in there

33
Which choice most effectively concludes the
sentence and paragraph?
N A) NO CHANGE
B) where we can learn what terms people use
pop to refer to soft drinks.
coke
C) a useful way to stay connected to friends,
soda
family, and colleagues.
Adapted from Jennifer M. Smith, Department of Geography, The D) helpful to researchers.
Pennsylvania State University, with data from www.popvssoda.com

Unauthorized copying or reuse of any part of this page is illegal. 26 CO NTI N U E


2 2
Questions 34-44 are based on the following passage.

...............................................................................................................................................................................................................................................................................................................
34
A) NO CHANGE
Creating Worlds: A Career in Game Design B) has been
C) are
If you love video games and have thought about how
D) was
the games you play might be changed or improved, or if
you’ve imagined creating a video game of your own, you
35
might want to consider a career as a video game designer. A) NO CHANGE
There 34 were a number of steps you can take to B) elements: the settings, characters, and plots that
make each game unique—
determine whether game design is the right field for you
C) elements—the settings, characters, and plots that
and, if it is, to prepare yourself for such a career. make each game unique—
Before making the choice, you should have some D) elements; the settings, characters, and plots that
make each game unique;
sense of what a video game designer does. Every video
game, whether for a console, computer, or mobile device, 36
starts with a concept that originates in the mind of a A) NO CHANGE
designer. The designer envisions the game’s fundamental B) job, however. No
C) job—however, no
35 elements: the settings, characters, and plots that make
D) job however no
each game unique, and is thus a primary creative force
behind a video game. 37
Conceptualizing a game is only the beginning of a At this point, the writer is considering adding the
following sentence.
video game designer’s 36 job, however, no matter how
Successful communication is essential if a
good a concept is, it will never be translated into a video designer’s idea is to become a reality.
game unless it is communicated effectively to all the other Should the writer make this addition here?
members of the video game development team. 37 A A) Yes, because it supports the conclusion drawn in
the following sentence.
designer must generate extensive documentation and
B) Yes, because it illustrates a general principle
discussed in the paragraph.
C) No, because it distracts from the focus of the
paragraph by introducing irrelevant material.
D) No, because it merely reformulates the thought
expressed in the preceding sentence.

Unauthorized copying or reuse of any part of this page is illegal. 27 CO NTI N U E


2 2
38 explain his or her ideas clearly in order to ensure that

...............................................................................................................................................................................................................................................................................................................
38
the programmers, artists, and others on the team all share Which choice results in a sentence that best supports
the point developed in this paragraph?
the same vision. 39 Likewise, anyone considering a
A) NO CHANGE
career as a video game designer must be 40 skilled B) possess a vivid imagination
writers and speakers. In addition, because video game C) assess his or her motivations carefully
development is a collaborative effort and because the D) learn to accept constructive criticism

development of any one game may take months or even


39
years, a designer must be an effective team player as well
A) NO CHANGE
as detail oriented. B) Nevertheless,
[1] A basic understanding of computer programming C) Consequently,
is essential. [2] In fact, many designers 41 initially begin D) However,

their pursuits as programmers. [3] Consider taking some


40
general computer science courses as well as courses in
A) NO CHANGE
artificial intelligence and graphics in order to increase B) a skilled writer and speaker.
your understanding of the technical challenges involved C) skilled both as writers and speakers.
in developing a video game. [4] Courses in psychology D) both skilled writers and speakers.

and human behavior may help you develop 42 emphatic


41
collaboration skills, while courses in the humanities, such
A) NO CHANGE
as in literature and film, should give you the background
B) start to begin their work
necessary to develop effective narrative structures. [5] A C) initiate their progression
D) begin their careers

42
A) NO CHANGE
B) paramount
C) eminent
D) important

Unauthorized copying or reuse of any part of this page is illegal. 28 CO NTI N U E


2 2
designer also needs careful educational preparation.

..............................................................................................................................................................................................................................................................
43
[6] Finally, because a designer should understand the A) NO CHANGE
B) the choice of video game design
business aspects of the video game industry, such as
C) you should choose video game design because it
budgeting and marketing, you may want to consider
D) choosing to design video games
taking some business courses. [7] Although demanding
and deadline driven, 43 video game design can be a 44
lucrative and rewarding field for people who love gaming To make this paragraph most logical, sentence 5
should be
and have prepared themselves with the necessary skills
A) placed where it is now.
and knowledge. 44 B) placed before sentence 1.
C) placed after sentence 3.
D) DELETED from the paragraph.

STOP
If you finish before time is called, you may check your work on this section only.
Do not turn to any other section.

Unauthorized copying or reuse of any part of this page is illegal. 29 CO NTI N U E


SAT Practice Test #4: Worksheets

ANSWER KEY

Reading Test Answers Writing and Language Test Answers

1 C 12 D 23 C 34 A 45 D 1 B 12 D 23 C 34 C
2 D 13 A 24 C 35 D 46 A 2 B 13 B 24 C 35 C
3 D 14 B 25 B 36 B 47 D 3 B 14 D 25 B 36 B
4 C 15 A 26 C 37 D 48 C 4 A 15 C 26 D 37 D
5 A 16 C 27 A 38 D 49 D 5 D 16 C 27 C 38 A
6 A 17 C 28 B 39 D 50 C 6 B 17 A 28 A 39 C
7 B 18 A 29 B 40 A 51 B 7 D 18 C 29 D 40 B
8 D 19 B 30 D 41 B 52 A 8 B 19 A 30 B 41 D
9 D 20 A 31 D 42 C 9 C 20 C 31 C 42 D
10 A 21 D 32 D 43 B 10 A 21 B 32 B 43 A

11 C 22 A 33 D 44 A 11 C 22 D 33 A 44 B

READING TEST
WRITING AND
RAW SCORE
LANGUAGE TEST
(NUMBER OF
RAW SCORE
CORRECT ANSWERS)
(NUMBER OF
CORRECT ANSWERS)

Math Test Math Test


No Calculator Answers Calculator Answers

1 A 11 C 1 B 11 A 21 C 31 1160

2 A 12 C 2 C 12 C
22 B 32 1/2 or 0.5
3 A 13 B 3 C 13 C 23 B 33 4.55
4 B 14 A 4 B 14 D 24 C 34 150

5 C 15 B 5 B 15 B
25 B 35 9/4 or 2.25
6 B 16 9
6 A 16 A 26 C 36 29, 30, 31, 32, 33, or 34

7 D 17 3/5 or 0.6 7 A 17 D
27 D 37 0.72
8 A 18 5
8 D 18 C 28 D 38 134

9 D 19 0 9 B 19 A
29 B
10 D 20 25
10 A 20 C
30 D

MATH TEST MATH TEST


NO CALCULATOR CALCULATOR
RAW SCORE RAW SCORE
(NUMBER OF (NUMBER OF
CORRECT ANSWERS) CORRECT ANSWERS)

SAT Practice Test #4 Created 8/4/2015 6


Practice
Test 5
#

Make time to take the practice test.


It’s one of the best ways to get ready
for the SAT.

After you’ve taken the practice test, score it


right away at sat.org/scoring.

© 2016 The College Board. College Board, SAT, and the acorn logo are registered trademarks of the College Board. K-5MSA04
2 2
Writing and Language Test
35 M I NU TES, 4 4 QUESTIONS

Turn to Section 2 of your answer sheet to answer the questions in this section.

Each passage below is accompanied by a number of questions. For some questions, you
will consider how the passage might be revised to improve the expression of ideas. For
other questions, you will consider how the passage might be edited to correct errors in
sentence structure, usage, or punctuation. A passage or a question may be accompanied by
one or more graphics (such as a table or graph) that you will consider as you make revising
and editing decisions.

Some questions will direct you to an underlined portion of a passage. Other questions will
direct you to a location in a passage or ask you to think about the passage as a whole.

After reading each passage, choose the answer to each question that most effectively
improves the quality of writing in the passage or that makes the passage conform to the
conventions of standard written English. Many questions include a “NO CHANGE” option.
Choose that option if you think the best choice is to leave the relevant portion of the
passage as it is.

Questions 1-11 are based on the following passage.


...................................................................................................................................

1
At this point, the writer is considering adding the
Prehistoric Printing following sentence.
Fossils provide paleontologists with a convenient
Paleontologists are using modern technology to gain
way of estimating the age of the rock in which
a greater understanding of the distant past. With the aid the fossils are found.

of computed tomography (CT) scanning and 3-D Should the writer make this addition here?

printing, researchers are able to create accurate models of A) Yes, because it supports the paragraph’s
argument with an important detail.
prehistoric fossils. 1 These models have expanded B) Yes, because it provides a logical transition from
the preceding sentence.
C) No, because it is not directly related to the main
point of the paragraph.
D) No, because it undermines the main claim of the
paragraph.

Unauthorized copying or reuse of any part of this page is illegal. 22 CO NTI N U E


2 2
researchers’ knowledge of ancient species and 2 swear

...............................................................................................................................................................................................................................................................................................................
2
to advance the field of paleontology in the years to come. A) NO CHANGE
CT scanners use X-rays to map the surface of a fossil B) subscribe
C) vow
in minute detail, recording as many as one million data
D) promise
points to create a digital blueprint. A 3-D printer then
builds a polymer model based on this blueprint, much as
3
a regular computer printer reproduces digital documents The writer is considering deleting the underlined
on paper. 3 Whereas the head of an ordinary computer sentence. Should the sentence be kept or deleted?

printer moves back and forth while printing ink onto A) Kept, because it helps explain why X-rays are
used in CT scanners.
paper, the corresponding part of a 3-D printer moves in B) Kept, because it provides details to illustrate how
multiple dimensions while squirting out thin layers of a 3-D printer works.
C) Deleted, because it contradicts the passage’s
melted polymer plastic. The plastic hardens quickly, information about digital blueprints.
4 it allows the printer to build the layers of the final D) Deleted, because it creates confusion about how
researchers gather data.
model. Compared with older ways of modeling fossils,
scanning and printing in this way is extremely versatile.
4
A) NO CHANGE
B) this
C) which
D) that

Unauthorized copying or reuse of any part of this page is illegal. 23 CO NTI N U E


2 2
[1] One significant benefit of 3-D printing

...............................................................................................................................................................................................................................................................................................................
5
technology is its ability to create scale reproductions of A) NO CHANGE
fossils. [2] But now 3-D scale models can be rearranged B) in order for learning
C) so that one is learning
with ease, which is a huge boon to scientists. [3] A team
D) so to learn
led by Drexel University professor Kenneth Lacovara is
making models of dinosaur bones one-tenth the bones’
6
original sizes 5 in order to learn how they fit together
A) NO CHANGE
when the animals were alive. [4] In the past, such B) it’s
research was limited by the weight and bulk of the fossils C) their
D) there
as well as 6 its preciousness and fragility. [5] In many
cases, scientists had to rearrange bones virtually, using
7
artists’ renderings. 7
To make this paragraph most logical, sentence 2
Because CT scanners can map objects that are should be placed
impossible to excavate, CT scanning and 3-D printing A) where it is now.
can also be used to reproduce fossils that scientists B) before sentence 1.
C) after sentence 4.
cannot observe firsthand. 8 By contrast, researchers
D) after sentence 5.

8
A) NO CHANGE
B) Nonetheless,
C) Besides,
D) For example,

Unauthorized copying or reuse of any part of this page is illegal. 24 CO NTI N U E


2 2
from the National Museum of Brazil 9 has relied on

...............................................................................................................................................................................................................................................................................................................
9
this technique to study a fossilized skeleton that was A) NO CHANGE
discovered protruding from a rock at an old São Paulo B) relied
C) will rely
railroad site. 10 The fossil was too delicate to be
D) is relying
removed from the rock. Because of the fossil’s delicate
nature, the team dug up a block of stone around the fossil
10
and brought it to their lab. With the aid of a CT scanner Which choice most effectively combines the
and a 3-D printer, they were able to produce a resin underlined sentences?

model of the fossil. Examining the model, the researchers A) The fossil could not be removed from the rock
on account of it being too delicate; moreover, the
determined that 11 one had found a new species, a team dug up a block of stone around it and
brought it to their lab.
75-million-year-old crocodile. While not every discovery
B) The team thought the fossil was too delicate to
will be as dramatic as this one, paleontologists anticipate remove from the rock, and their next decision
was to dig up a block of stone around the fossil
further expanding their knowledge of ancient life-forms and bring it to their lab.
as CT scanning and 3-D printing continue to make fossils C) The fossil was too delicate to be removed from
the rock, so the team dug up a block of stone
more accessible. around the fossil and brought it to their lab.
D) In removing the fossil from the rock, the team
found it was too delicate; then they dug up a
block of stone around the fossil and brought it to
their lab.

11
A) NO CHANGE
B) he or she
C) they
D) it

Unauthorized copying or reuse of any part of this page is illegal. 25 CO NTI N U E


2 2
Questions 12-22 are based on the following passage.

...............................................................................................................................................................................................................................................................................................................
12
A) NO CHANGE
Thomas Nast, the Crusading Cartoonist B) Therefore,
“Stop them pictures!” Legend has it that the corrupt C) Furthermore,
D) DELETE the underlined portion.
politician William “Boss” Tweed once used those words
when ordering someone to offer a bribe to Thomas Nast,
13
an artist who had become famous for cartoons that called
Which choice most effectively combines the
for reforms to end corruption. 12 As a result, Tweed’s underlined sentences?
attempt to silence the artist failed, and Nast’s cartoons, A) Powerful political organizations in the 1860s and
the 1870s started taking control of city
published in magazines like Harper’s Weekly, actually governments, and they were known as
“political machines.”
played a key role in bringing Boss Tweed and his cronies
B) Known as “political machines,” in the 1860s and
to justice. the 1870s, political organizations that were
powerful started taking control of city
13 There were powerful political organizations in
governments.
the 1860s and the 1870s. The organizations were known C) City governments were taken control of in the
1860s and the 1870s, and powerful political
as “political machines” and started taking control of city
organizations known as “political machines”
governments. These political machines were able to pack did so.
D) In the 1860s and the 1870s, powerful political
legislatures and courts with hand-picked supporters by
organizations known as “political machines”
purchasing 14 votes, a form of election fraud involving started taking control of city governments.

the exchange of money or favors for votes. Once a


political machine had control of enough important 14
A) NO CHANGE
positions, its members were able to use public funds to
B) votes, being
enrich themselves and their friends. Boss Tweed’s
C) votes, that is
Tammany Hall group, which controlled New York D) votes, which it is
15 City in the 1860s—stole more than $30 million,
15
A) NO CHANGE
B) City in the 1860s,
C) City, in the 1860s,
D) City in the 1860s

Unauthorized copying or reuse of any part of this page is illegal. 26 CO NTI N U E


2 2
the equivalent of more than $365 million today.

...............................................................................................................................................................................................................................................................................................................
16
16 Tweed had been elected to a single two-year term in The writer is considering deleting the underlined
sentence. Should the sentence be kept or deleted?
Congress in 1852. Tammany Hall was so powerful and
A) Kept, because it introduces the quote from the
17 corrupt that, the New York Times, commented New York Times in the next sentence.
“There is absolutely nothing . . . in the city which is B) Kept, because it adds a vital detail about Tweed
that is necessary to understand his power.
beyond the reach of the insatiable gang.”
C) Deleted, because it blurs the focus of the
Given the extent of Tweed’s power, it is remarkable paragraph by introducing loosely related
information.
that a single cartoonist could have played such a
D) Deleted, because it contains information that
significant role in bringing about his downfall. Nast’s undermines the main claim of the passage.
cartoons depicted Tweed as a great big bloated thief. One
of the artist’s most 18 famous images showed Tweed 17
A) NO CHANGE
with a bag of money in place of his 19 head. Another
B) corrupt, that the New York Times commented,
featured Tweed leaning against a ballot box with the
C) corrupt that the New York Times commented,
caption “As long as I count the votes, what are you going D) corrupt that the New York Times, commented
to do about it?” These cartoons were so effective in part
because many of the citizens who supported Tweed were 18
illiterate and thus could not read the newspaper accounts A) NO CHANGE
B) famous and well-known
of his criminal activities. Nast’s cartoons, though, widely
C) famous and commonly known
exposed the public to the injustice of Tweed’s political
D) famous, commonly known
machine.

19
Which choice adds the most relevant supporting
information to the paragraph?
A) head; like many other Nast cartoons, that one
was published in Harper’s Weekly.
B) head; Nast would later illustrate Tweed’s escape
from prison.
C) head, one depiction that omits Tweed’s
signature hat.
D) head, an image that perfectly captured Tweed’s
greedy nature.

Unauthorized copying or reuse of any part of this page is illegal. 27 CO NTI N U E


2 2

...............................................................................................................................................................................................................................................................................................................
Nast’s campaign to bring down Tweed and the 20
Tammany Hall gang was ultimately successful. In the A) NO CHANGE
elections of 1871, the public voted against most of the B) persecuted on
C) persecuted with
Tammany Hall candidates, greatly weakening Tweed’s
D) prosecuted on
power. Eventually, Tweed and his gang were
20 persecuted for a number of charges, including fraud 21
and larceny, and many of them were sent to jail. In 1875 A) NO CHANGE
Tweed escaped from jail and fled to Spain and B) bringing
unwittingly 21 brought about one final 22 pinnacle for C) brings
D) has brought
the power of political cartoons: A Spanish police officer
recognized Tweed from one of Nast’s cartoons.
22
Consequently, Tweed was sent back to jail, and Nast was
A) NO CHANGE
hailed as the man who toppled the great Tammany Hall B) triumph
machine. C) culmination
D) apex

Unauthorized copying or reuse of any part of this page is illegal. 28 CO NTI N U E


2 2
Questions 23-33 are based on the following passage

...............................................................................................................................................................................................................................................................................................................
23
and supplementary material.
A) NO CHANGE
B) its
Rethinking Crowdfunding in the Arts
C) its’
Crowdfunding is a popular way to raise money using D) their
the Internet. The process sounds simple: an artist,
entrepreneur, or other innovator takes his or her ideas 24

straight to the public via a crowdfunding website. The Which choice most effectively combines the
underlined sentences?
innovator creates a video about the project and offers, in
A) With the idea being obviously very attractive, an
exchange for donations, a series of “perks,” from obscure method of photography may be made
available to many at little expense.
acknowledgment on a social media site to a small piece of
B) The idea is obviously very attractive: an obscure
art. Many crowdfunding programs are all-or-nothing; in method of photography may be made available
to many at little expense.
other words, the innovator must garner 100 percent
C) An obscure method of photography may be
funding for the project or the money is refunded to the made available to many at little expense, and the
idea is obviously very attractive.
donors. At 23 it’s best, the system can give creators
D) An obscure method of photography, an idea that
direct access to millions of potential backers. is obviously very attractive, may be made
available to many at little expense.
The home page of one leading crowdfunding site
features a project to manufacture pinhole cameras on a
25
3‑D printer. 24 The idea is obviously very attractive. An
A) NO CHANGE
obscure method of photography may be made available B) Therefore,
to many with little expense. Within weeks, the project C) In effect,
D) As a rule,
was 621 percent funded. In contrast, on the same page, a
small Brooklyn performance venue is attempting to raise
money for its current season. The venue features works of
performance art showcased in a storefront window.
Those who have seen the space consider it vital.
25 However, that group may not be large enough; with
just fourteen days to go in the fund-raising period, the
campaign is only 46 percent funded.

Unauthorized copying or reuse of any part of this page is illegal. 29 CO NTI N U E


2 2
Artists such as these Brooklyn performers find that

...............................................................................................................................................................................................................................................................................................................
26
crowdfunding exacerbates problems that already exist. A) NO CHANGE
26 Work, that is easily understood and appreciated, is B) Work that is easily understood and appreciated
is supported,
supported, while more complex work goes unnoticed. C) Work that is easily understood, and appreciated
27 Time that could be used creating art is spent devising is supported
D) Work—that is easily understood and
clever perks to draw the attention of potential appreciated—is supported,
contributors. 28 In addition, audiences may contain
many “free 29 riders,” they did not make contributions. 27
At this point, the writer is considering adding the
following sentence.
Crowdfunding tends to attract contributors from
a wide variety of professional fields.
Should the writer make this addition here?
A) Yes, because it gives more information about the
people who donate to crowdfunding campaigns.
B) Yes, because it reinforces the writer’s point about
the funding of artistic projects.
C) No, because it fails to take into account project
funding received from public institutions.
D) No, because it blurs the focus of the paragraph
by introducing a poorly integrated piece of
information.

28
A) NO CHANGE
B) Conversely,
C) However,
D) Thus,

29
A) NO CHANGE
B) riders,” not making
C) riders,” who did not make
D) riders” to not make

Unauthorized copying or reuse of any part of this page is illegal. 30 CO NTI N U E


2 2
Ironically, the success of crowdfunding may weaken

...............................................................................................................................................................................................................................................................................................................
30
overall funding for the arts if people begin to feel that A) NO CHANGE
paying for the art 30 loved by them is someone else’s B) they love
C) loved by him or her
responsibility.
D) he or she loves
[1] One innovative playwright has woven the
deficiencies of the system into her crowdfunding model.
31
[2] Though the price for her tickets was higher than that A) NO CHANGE
of tickets for comparable shows, it was still affordable to B) their
most theatergoers—and reflected the real cost of the C) her
D) its
performance. [3] She presented the total cost for
producing her play on a crowdfunding site. [4] Then she
32
divided the total cost by the number of people she
To make this paragraph most logical, sentence 2
expected to attend the performance. [5] The result of the should be placed

calculation was the minimum donor price, and only A) where it is now.
B) after sentence 3.
donors who paid at least the minimum ticket price were
C) after sentence 4.
allowed to attend the performance. [6] By subverting the
D) after sentence 5.
presumption that money used for her project is an
altruistic donation, the playwright showed that 31 our
work has monetary value to those who enjoy it. 32

Unauthorized copying or reuse of any part of this page is illegal. 31 CO NTI N U E


2 2
Crowdfunded Projects on Kickstarter in 2012

games food dance


publishing fashion theater
food dance art
art theater food
theater photography photography
fashion art games
photography games publishing
dance publishing fashion

0 25 50 75 100 0 25 50 75 100 0 25 50 75 100


Money raised Average pledge Success rate (percent
(millions of dollars) (dollars) of fully funded projects)
Adapted from “These Were the Most Successful Projects on Kickstarter Last Year.” ©2013 by The Economist Newspaper Limited.

Question 33 asks about the graphic.


.....................................................................................................................................................................

33
Which choice offers an accurate interpretation of the
data in the graphs?
A) The project category with the lowest amount of
money raised was also the most successfully
funded project category.
B) The project category with the highest average
pledge amount was also the most successfully
funded project category.
C) The project category with the lowest average
pledge amount was also the project category that
raised the most money.
D) The project category with the highest average
pledge amount was also the project category with
the most money raised.

Unauthorized copying or reuse of any part of this page is illegal. 32 CO NTI N U E


2 2
Questions 34-44 are based on the following passage.

...............................................................................................................................................................................................................................................................................................................
34
For the sake of the logic and cohesion of the
Investigative Journalism: An Evolving American paragraph, sentence 3 should be
Tradition A) placed where it is now.
[1] The recent precipitous decline of print journalism B) placed before sentence 1.
as a viable profession has exacerbated long-held concerns C) placed after sentence 1.
D) DELETED from the paragraph.
about the state of investigative reporting in the United
States. [2] Facing lower print circulation and diminished
35
advertising revenue, many major newspapers have
A) NO CHANGE
reduced or eliminated investigative resources. B) reporters:
[3] Newspapers, the traditional nurturing ground for C) reporters,
investigative journalism, have been hit especially hard by D) reporter’s;

the widespread availability of free news online. [4] To


36
survive, investigative journalism must continue to adapt
A) NO CHANGE
to the digital age. 34
B) Undertaken in
It is not difficult to understand why
C) Overtaking
a cash-strapped, understaffed publication might feel D) Taking off from
pressure to cut teams of investigative
35 reporter’s—their work is expensive and 37
time-consuming. 36 Taking on the public interest, A) NO CHANGE
B) business scandals,
investigative journalism involves original, often
C) abuse of government power,
long-form reporting on such topics as 37 illegal
D) DELETE the underlined portion.
activities, street crime, corporate wrongdoing, and
political corruption. An investigative story involves one
or more experienced journalists dedicating their full
energy and the resources of the publisher to a piece for a
prolonged period of time. Expensive legal battles may
ensue. The results of this work, though costly, have

Unauthorized copying or reuse of any part of this page is illegal. 33 CO NTI N U E


2 2
helped keep those in power accountable. The exposure by

...............................................................................................................................................................................................................................................................................................................
38
Washington Post reporters Bob Woodward and Carl At this point, the writer is considering adding the
following sentence.
Bernstein of government misconduct in the Watergate
In 1954, Edward R. Murrow and Fred Friendly
scandal resulted in the resignation of President Richard produced episodes of the CBS television show
Nixon in 1974. More recently, Seymour Hersh, reporting See It Now that contributed to the end of
US senator Joseph McCarthy’s anticommunist
for the New Yorker in 2004, helped publicize the “witch hunts.”
mistreatment of Iraqi prisoners by US personnel at Should the writer make this addition here?
Abu Ghraib during the Iraq War. 38 In these and other A) Yes, because it helps clarify that the passage’s
main focus is on investigations of political
cases, exposure from reporters has served as an corruption.
important 39 blockade to or scolding of malfeasance. B) Yes, because it offers an important counterpoint
to the other cases previously described in the
paragraph.
C) No, because it gives an example that is both
chronologically and substantively out of place in
the paragraph.
D) No, because it provides an example that is
inconsistent with the passage’s definition of
investigative journalism.

39
A) NO CHANGE
B) interference to or condemnation of
C) drag on or reproof of
D) deterrent or rebuke to

Unauthorized copying or reuse of any part of this page is illegal. 34 CO NTI N U E


2 2
While worrisome, the decline of traditional print

...............................................................................................................................................................................................................................................................................................................
40
media 40 could not entail the end of investigative Which choice most effectively suggests that the
“end of investigative journalism” is a real possibility
journalism. 41 Although many newsrooms have but one that can be prevented?
reduced their staff, some still employ investigative A) NO CHANGE
reporters. Nonprofit 42 enterprises such as the B) need

Organized Crime and Corruption Reporting Project have C) will


D) must
begun to fill the void created by staff losses at newspapers
and magazines. Enterprising freelance reporters, newly
41
funded by nonprofits, make extensive use of social media,
Which choice most effectively sets up the examples
in the following sentences?
A) NO CHANGE
B) Investigative journalism also declined between
the 1930s and 1950s, only to be revived in
the 1960s.
C) According to the Pew Research Center, more
people get their national and international news
from the Internet than from newspapers.
D) Indeed, recent years have witnessed innovative
adjustments to changing times.

42
A) NO CHANGE
B) enterprises: such as
C) enterprises such as:
D) enterprises, such as

Unauthorized copying or reuse of any part of this page is illegal. 35 CO NTI N U E


2 2
including blogs and Twitter, to foster a public

.....................................................................................................................................................................................................................................................
43
conversation about key issues. The Help Me Investigate A) NO CHANGE
project, 43 for example, solicited readers to submit tips B) therefore,
C) however,
and information related to ongoing stories to its website.
D) in any case,
Far from marking the end of investigative journalism,
44 cooperation among journalists and ordinary citizens
44
has been facilitated by the advent of the digital age
A) NO CHANGE
through an increase in the number of potential B) the number of potential investigators has
increased since the advent of the digital age
investigators.
owing to the facilitation of cooperation among
journalists and ordinary citizens.
C) the advent of the digital age has increased the
number of potential investigators by facilitating
cooperation among journalists and ordinary
citizens.
D) by facilitating cooperation among journalists
and ordinary citizens the advent of the digital
age has increased the number of potential
investigators.

STOP
If you finish before time is called, you may check your work on this section only.
Do not turn to any other section.

Unauthorized copying or reuse of any part of this page is illegal. 36


SAT Practice Test #5: Worksheets

ANSWER KEY

Reading Test Answers Writing and Language Test Answers

1 D 12 A 23 B 34 B 45 C 1 C 12 D 23 B 34 C
2 C 13 B 24 A 35 A 46 B 2 D 13 D 24 B 35 B
3 C 14 D 25 B 36 B 47 A 3 B 14 A 25 A 36 B
4 A 15 C 26 C 37 C 48 C 4 C 15 B 26 B 37 D
5 C 16 A 27 D 38 A 49 C 5 A 16 C 27 D 38 C
6 A 17 B 28 B 39 B 50 A 6 C 17 C 28 A 39 D
7 D 18 B 29 D 40 D 51 A 7 D 18 A 29 C 40 B
8 B 19 B 30 B 41 D 52 B 8 D 19 D 30 B 41 D
9 B 20 A 31 D 42 C 9 B 20 D 31 C 42 A
10 B 21 D 32 B 43 D 10 C 21 A 32 D 43 A

11 B 22 A 33 C 44 D 11 C 22 B 33 A 44 C

READING TEST
WRITING AND
RAW SCORE
LANGUAGE TEST
(NUMBER OF
RAW SCORE
CORRECT ANSWERS)
(NUMBER OF
CORRECT ANSWERS)

Math Test Math Test


No Calculator Answers Calculator Answers

1 D 11 C 1 D 11 A 21 A 31 1492

2 A 12 D 2 C 12 D
22 C 32 29/3, 9.66, 9.67
3 B 13 B 3 A 13 D 23 D 33 7

4 C 14 C 4 B 14 A
24 B 34 9

5 D 15 D 5 C 15 A
25 D 35 13

6 A 16 4
6 B 16 D
26 B 36 80

7 C 17 6/5, 1.2 7 A 17 D
27 C 37 43, 43.5, 44, 87/2

8 A 18 21/4, 5.25 8 C 18 C
28 C 38 6

9 A 19 2
9 B 19 B
29 B
10 B 20 97
10 A 20 D
30 B

MATH TEST MATH TEST


NO CALCULATOR CALCULATOR
RAW SCORE RAW SCORE
(NUMBER OF (NUMBER OF
CORRECT ANSWERS) CORRECT ANSWERS)

SAT Practice Test #5 Created 7/29/2016 6


Practice
Test 6
#

Make time to take the practice test.


It’s one of the best ways to get ready
for the SAT.

After you’ve taken the practice test, score it


right away at sat.org/scoring.

© 2016 The College Board. College Board, SAT, and the acorn logo are registered trademarks of the College Board. 5MSA05
2 2
Writing and Language Test
35 M I NU TES, 4 4 QUESTIONS

Turn to Section 2 of your answer sheet to answer the questions in this section.

Each passage below is accompanied by a number of questions. For some questions, you
will consider how the passage might be revised to improve the expression of ideas. For
other questions, you will consider how the passage might be edited to correct errors in
sentence structure, usage, or punctuation. A passage or a question may be accompanied by
one or more graphics (such as a table or graph) that you will consider as you make revising
and editing decisions.

Some questions will direct you to an underlined portion of a passage. Other questions will
direct you to a location in a passage or ask you to think about the passage as a whole.

After reading each passage, choose the answer to each question that most effectively
improves the quality of writing in the passage or that makes the passage conform to the
conventions of standard written English. Many questions include a “NO CHANGE” option.
Choose that option if you think the best choice is to leave the relevant portion of the
passage as it is.

Questions 1-11 are based on the following passage.


...................................................................................................................................

1
A) NO CHANGE
A Necessary Resource for Science B) 227. Which is one
In the winter of 1968, scientists David Schindler and C) 227. One
D) 227, one
Gregg Brunskill poured nitrates and phosphates into
Lake 1 227, this is one of the 58 freshwater bodies that
2
compose Canada’s remotely located Experimental Lakes
A) NO CHANGE
Area. Schindler and Brunskill were contaminating the B) destructive, and irresponsible this method
water not out of malice but in the name of research. C) destructive and, irresponsible, this method
While deliberately adding chemical compounds to a lake D) destructive and irresponsible this method,

may seem 2 destructive and irresponsible, this method


of experimenting is sometimes the most effective way to
influence policy and save the environment from even
more damaging pollution.

Unauthorized copying or reuse of any part of this page is illegal. 22 CO NTI N U E


2 2
Schindler and Brunskill were investigating possible

...............................................................................................................................................................................................................................................................................................................
3
causes for the large blooms of blue-green algae, or Which choice most effectively combines the
underlined sentences?
cyanobacteria, that had been affecting bodies of water
A) In addition to being unsightly and odorous,
such as Lake Erie. 3 In addition to being unsightly and these algal blooms cause oxygen depletion: the
result being that it kills fish and other wildlife in
odorous, these algal blooms cause oxygen depletion.
the lakes.
Oxygen depletion kills fish and other wildlife in the lakes. B) In addition to being unsightly and odorous,
these algal blooms cause oxygen depletion; the
Just weeks after the scientists added the nitrates and
algal blooms cause oxygen depletion that kills
phosphates, the water in Lake 227 turned bright fish and other wildlife in the lakes.
C) In addition to being unsightly and odorous,
4 green. It was thick with: the same type of algal
these algal blooms cause oxygen depletion, and
blooms that had plagued Lake Erie. oxygen depletion caused by the algal blooms kills
fish and other wildlife in the lakes.
D) In addition to being unsightly and odorous,
these algal blooms cause oxygen depletion,
which kills fish and other wildlife in the lakes.

4
A) NO CHANGE
B) green: it was thick with
C) green. It was thick with—
D) green, it was thick with

Unauthorized copying or reuse of any part of this page is illegal. 23 CO NTI N U E


2 2
5 One mission of the Experimental Lakes Area is

...............................................................................................................................................................................................................................................................................................................
5
to conduct research that helps people better understand Which choice provides the best transition from the
previous paragraph to this one?
threats to the environment. The scientists divided the
A) NO CHANGE
lake in half by placing a nylon barrier through the
B) The Experimental Lakes Area is located in
narrowest part of its figure-eight shape. In one half of a sparsely inhabited region that experiences few
effects of human and industrial activity.
Lake 226, they added phosphates, nitrates, and a source
C) To isolate the cause of the algae, Schindler and
of carbon; in the other, they added just nitrates 6 and a Brunskill performed another experiment, this
time using Lake 226.
source of carbon was added. Schindler and Brunskill
D) The process by which water becomes enriched
hypothesized that phosphates were responsible for the by dissolved nutrients, such as phosphates, is
called eutrophication.
growth of cyanobacteria. The experiment confirmed their
suspicions when the half of the lake containing the
6
phosphates 7 was teeming with blue-green algae.
A) NO CHANGE
B) and a source of carbon.
C) plus also a source of carbon.
D) but also adding a source of carbon.

7
A) NO CHANGE
B) were teeming
C) are teeming
D) teems

Unauthorized copying or reuse of any part of this page is illegal. 24 CO NTI N U E


2 2
Schindler and Brunskill’s findings were 8 shown

...............................................................................................................................................................................................................................................................................................................
8
off by the journal Science. The research demonstrated a A) NO CHANGE
clear correlation between introducing phosphates and the B) put in the spotlight of
C) published in
growth of blue-green algae. 9 For example, legislators
D) put into
in Canada passed laws banning phosphates in laundry
detergents, which had been entering the water supply. 10 9
A) NO CHANGE
B) Similarly,
C) However,
D) Subsequently,

10
At this point, the writer wants to add a second policy
outcome of the research described. Which choice
best accomplishes this goal?
A) Lake 226 continued to develop blooms of
blue-green algae for eight consecutive years after
the experiment took place.
B) In the United States, many individual states have
also adopted legislation to eliminate, or at least
reduce, phosphorous content in laundry
detergents.
C) In 1974, Schindler initiated a study of the effects
of acid rain, using Lake 223 to examine how
sulfuric acid altered aquatic ecosystems.
D) Aerial photos of the lakes taken before and
during algal blooms helped convey the effects
of phosphates in water to the public.

Unauthorized copying or reuse of any part of this page is illegal. 25 CO NTI N U E


2 2
Experiments like these can help people understand

...............................................................................................................................................................................................................................................................................................................
11
the unintended consequences of using certain household Which choice most effectively anticipates and
addresses a relevant counterargument to the
products. 11 Of course, regulating the use of certain argument in favor of the types of experiments
chemical compounds can be a controversial issue. described in the passage?
A) NO CHANGE
Selectively establishing remote study locations, such as
B) Many companies now offer phosphate-free
the Experimental Lakes Area, can provide scientists with alternatives for household cleaning products.
opportunities to safely conduct controlled research. This C) Obviously, scientists should not be allowed to
randomly perform experiments on just any body
research can generate evidence solid enough to persuade of water.
policy makers to take action in favor of protecting the D) Phosphates are sometimes used in agricultural
fertilizers, in addition to being used in cleaning
larger environment. products.

Unauthorized copying or reuse of any part of this page is illegal. 26 CO NTI N U E


2 2
Questions 12-22 are based on the following passage.

...............................................................................................................................................................................................................................................................................................................
12
A) NO CHANGE
A Little to the Left, but Not Too Much! B) stage’s of its’
Italy’s Tower of Pisa has been leaning southward C) stage’s of it’s
D) stages of its
since the initial 12 stages of it’s construction over
800 years ago. 13 Indeed, if the tower’s construction had
13
not taken two centuries and involved significant breaks
A) NO CHANGE
due to war and civil unrest, which allowed the ground B) Therefore,
beneath the tower to settle, the tower would likely have C) Nevertheless,
collapsed before it was completed. D) However,

Unauthorized copying or reuse of any part of this page is illegal. 27 CO NTI N U E


2 2

...............................................................................................................................................................................................................................................................................................................
Luckily, the tower survived, and its tilt has made it an 14
Italian 14 icon, it attracts visitors from all over who flock A) NO CHANGE
to Pisa to see one of the greatest architectural B) icon, attracting
C) icon, its attracting
15 weirdnesses in the world. 16 By the late twentieth
D) icon; attracting
century, the angle of the tower’s tilt had reached an
astonishing 5.5 degrees; in 17 1990, Italy’s government 15
closed the tower to visitors and appointed a committee to A) NO CHANGE
find a way to save it. B) deviations
C) oddities
D) abnormalities

16
At this point, the writer is considering adding the
following sentence.
Unfortunately, the tower’s tilt has steadily
increased over the centuries, placing the
structure in danger of collapse.
Should the writer make this addition here?
A) Yes, because it provides an important
restatement of the main claim in the previous
sentence.
B) Yes, because it establishes an important shift in
emphasis in the paragraph’s discussion about the
tower’s tilt.
C) No, because it interrupts the paragraph’s
discussion with irrelevant information.
D) No, because it repeats information that is already
presented in the first paragraph.

17
A) NO CHANGE
B) 1990, Italy’s government, closed
C) 1990 Italy’s government, closed,
D) 1990: Italy’s government closed

Unauthorized copying or reuse of any part of this page is illegal. 28 CO NTI N U E


2 2
The committee was charged with saving the tower

...............................................................................................................................................................................................................................................................................................................
18
without ruining its aesthetic, 18 which no one had yet Which choice best supports the main point of the
paragraph?
managed to achieve. The committee’s first attempt to
A) NO CHANGE
reduce the angle of the tower’s tilt—placing 600 tons of
B) although not everyone on the committee agreed
iron ingots (molded pieces of metal) on the tower’s north completely about what that aesthetic was.
C) which meant somehow preserving the tower’s tilt
side to create a counterweight—was derided because the
while preventing that tilt from increasing and
bulky weights ruined the tower’s appearance. The toppling the tower.
D) which included the pristine white marble finish
attempt at a less visible solution—sinking anchors into
that has come to be widely associated with the
the ground below the tower—almost caused the tower to tower’s beauty.

fall.

Unauthorized copying or reuse of any part of this page is illegal. 29 CO NTI N U E


2 2
[1] Enter committee member John Burland, 19 he is

...............................................................................................................................................................................................................................................................................................................
19
a geotechnical engineer from England who saved A) NO CHANGE
London’s clock tower Big Ben from collapse. [2] Burland B) Burland is
C) his being
began a years-long process of drilling out small amounts
D) DELETE the underlined portion.
of soil from under the tower 20 that took several years to
complete and then monitoring the tower’s resulting 20
movement. [3] Twice daily, Burland evaluated these A) NO CHANGE
movements and made recommendations as to how much B) —taking several years to complete—
soil should be removed in the next drilling. [4] By 2001, C) that took him several years to complete
D) DELETE the underlined portion.
almost 77 tons of soil had been removed, and the tower’s
tilt had decreased by over 1.5 degrees; the ugly iron
21
weights were removed, and the tower was reopened to
A) NO CHANGE
visitors. [5] Burland 21 advocated using soil extraction: B) advocated to use
removing small amounts of soil from under the tower’s C) advocated the using of
north side, opposite its tilt, to enable gravity to straighten D) advocating to use

the tower. 22
22
The tower’s tilt has not increased since, and the
To make this paragraph most logical, sentence 5
committee is confident that the tower will be safe for should be
another 200 years. Burland is now working on a more A) placed after sentence 1.
permanent solution for keeping the tower upright, but he B) placed after sentence 2.
C) placed after sentence 3.
is adamant that the tower never be completely
D) DELETED from the paragraph.
straightened. In an interview with PBS’s Nova, Burland
explained that it is very important “that we don’t really
change the character of the monument. That would be
quite wrong and quite inappropriate.”

Unauthorized copying or reuse of any part of this page is illegal. 30 CO NTI N U E


2 2
Questions 23-33 are based on the following passage

...............................................................................................................................................................................................................................................................................................................
23
and supplementary material.
Which choice is the best introduction to the
paragraph?
The Physician Assistant Will See You Now A) NO CHANGE
23 The term “paramedics” refers to health care B) For many Americans, finding a physician is
likely to become a growing challenge.
workers who provide routine and clinical services. While
C) Getting treatment for an illness usually requires
the pressures of an aging population, insurance reforms, seeing either a general practitioner or a
specialist.
and health epidemics have increased demand for care, the
D) Worldwide the costs of health care are increasing
supply of physicians is not expected to 24 keep pace. at an alarming rate.
The Association of American Medical Colleges predicts a
shortage of over 90,000 physicians by 2020; by 2025, that 24
A) NO CHANGE
number could climb to more than 130,000. In some parts
B) maintain the tempo.
of the country, shortages are already a sad fact of life. A
C) get in line.
2009 report by the Bureau of Health Professions notes D) move along.
that although a fifth of the US population lives in rural
areas, less than a tenth of US physicians serves that 25
population. Because a traditionalist response to the A) NO CHANGE
B) bolstering
crisis— 25 amping up medical-college enrollments and
C) arousing
expanding physician training programs—is too slow and
D) revving up
costly to address the near-term problem, alternatives are
being explored. One promising avenue has been greater
reliance on physician assistants (PAs).

Unauthorized copying or reuse of any part of this page is illegal. 31 CO NTI N U E


2 2
26 By virtue of 27 there medical training, PAs can

...............................................................................................................................................................................................................................................................................................................
26
perform many of the jobs traditionally done by doctors, At this point, the writer is considering adding the
following sentence.
including treating chronic and acute conditions,
Several factors argue in favor of such an
performing minor 28 surgeries: and prescribing some expanded role.
medications. However, although well 29 compensated Should the writer make this addition here?
earning in 2012 a median annual salary of $90,930, PAs A) Yes, because it introduces a counterargument for
balance.
cost health care providers less than do the physicians who
B) Yes, because it frames the points that the
paragraph will examine.
C) No, because it does not specify the education
required to be a PA.
D) No, because it presents information that is only
tangential to the main argument.

27
A) NO CHANGE
B) they’re
C) their
D) his or her

28
A) NO CHANGE
B) surgeries; and
C) surgeries, and,
D) surgeries, and

29
A) NO CHANGE
B) compensated (earning in 2012 a median annual
salary of $90,930),
C) compensated, earning in 2012 a median annual
salary of $90,930
D) compensated: earning in 2012 a median annual
salary of $90,930,

Unauthorized copying or reuse of any part of this page is illegal. 32 CO NTI N U E


2 2
might otherwise undertake these tasks. Moreover, the

...............................................................................................................................................................................................................................................................................................................
30
training period for PAs is markedly shorter than A) NO CHANGE
30 those for physicians—two to three years versus the B) that compared with
C) that for
seven to eleven required for physicians.
D) DELETE the underlined portion.
Physician assistants already offer vital primary care
in many locations. Some 90,000 PAs were employed 31
nationwide in 2012. Over and above their value in A) NO CHANGE
partially compensating for the general physician shortage B) Thus,
has been their extraordinary contribution to rural health C) Despite this,
D) On the other hand,
care. A recent review of the scholarly literature by
Texas researchers found that PAs lend cost-efficient,
widely appreciated services in underserved areas.
31 In addition, rural-based PAs often provide a broader
spectrum of such services than do their urban and
suburban counterparts, possibly as a consequence of the
limited pool of rural-based physicians.

Unauthorized copying or reuse of any part of this page is illegal. 33 CO NTI N U E


2 2

...............................................................................................................................................................................................................................................................................................................
Increasingly, PAs and other such medical 32
practitioners have become a critical complement to At this point, the writer is considering adding the
following sentence.
physicians. A 2013 RAND Corporation report estimates
In fact, according to the data presented in the
that while the number of primary care physicians will table, physician assistants will likely outnumber
physicians by 2025.
increase slowly from 2010 to 2025, the number of
Should the writer make this addition here?
physician assistants and nurse-practitioners in primary
A) Yes, because it provides additional support for
care will grow at much faster rates. 32 Both by merit and the main point of the paragraph.
from necessity, PAs are likely to greet more 33 patience B) Yes, because it addresses a possible
counterargument to the writer’s main claim.
than ever before.
C) No, because it is not an accurate interpretation
of the data.
Supply of Physicians, Physician Assistants,
and Nurse-Practitioners in Primary Care D) No, because it introduces irrelevant information
Clinical Practice in 2010 and 2025 that interrupts the flow of the passage.
2010 2025 (predicted)
Percent Percent 33
Provider type Number Number
of total of total
Physicians 210,000 71 216,000 60 A) NO CHANGE
Physician B) patience, than
30,000 10 42,000 12
assistants C) patients then
Nurse-
56,000 19 103,000 28 D) patients than
practitioners
Total 296,000 100 361,000 100
Adapted from David I. Auerbach et al., “Nurse-Managed Health Centers
and Patient-Centered Medical Homes Could Mitigate Expected Primary
Care Physician Shortage.” ©2013 by Project HOPE: The People-to-People
Health Foundation, Inc.

Unauthorized copying or reuse of any part of this page is illegal. 34 CO NTI N U E


2 2
Questions 34-44 are based on the following passage.

...............................................................................................................................................................................................................................................................................................................
34
Which choice most effectively combines the
Gold into Silver: The “Reverse Alchemy” of Superhero underlined sentences?
Comics History A) In an effort to make their characters and stories
34 Popular film franchises are often “rebooted” in fresh and relevant for new audiences, popular
film franchises, which are often “rebooted,” are
an effort to make their characters and stories fresh and similar to superhero comic books, which are
periodically reworked to try to increase their
relevant for new audiences. Superhero comic books are appeal to contemporary readers.
periodically reworked to try to increase their appeal to B) Just as popular film franchises are often
“rebooted” in an effort to make their characters
contemporary readers. This practice is almost as and stories fresh and relevant for new audiences,
35 elderly as the medium itself and has in large part superhero comic books are periodically
reworked to try to increase their appeal to
established the “ages” that compose comic book history. contemporary readers.
The shift from the Golden to the Silver Age is probably C) Superhero comic books are periodically
reworked to try to increase their appeal to
the most successful 36 example: of publishers contemporary readers, while popular film
franchises are often “rebooted” in an effort to
responding to changing times and tastes. make their characters and stories fresh and
relevant for new audiences.
D) Superhero comic books are much like popular
film franchises in being often “rebooted” in an
effort to make their characters and stories fresh
and relevant for new audiences and periodically
reworked to try to increase their appeal to
contemporary readers.

35
A) NO CHANGE
B) old
C) mature
D) geriatric

36
A) NO CHANGE
B) example, of publishers
C) example of publishers,
D) example of publishers

Unauthorized copying or reuse of any part of this page is illegal. 35 CO NTI N U E


2 2
The start of the first (“Golden”) age of comic books is

...............................................................................................................................................................................................................................................................................................................
37
often dated to 1938 with the debut of Superman in Action Which choice is most consistent with the previous
examples in the sentence?
Comics #1. Besides beginning the age, Superman in many
A) NO CHANGE
respects defined it, becoming the model on which many
B) hold down a regular job as a newspaper reporter.
later superheroes were based. His characterization, as C) wear a bright blue costume with a flowing
red cape.
established in Superman #1 (1939), was relatively simple.
D) live in the big city of Metropolis instead of the
He could “hurdle skyscrapers” and “leap an eighth of a small town where he grew up.
mile”; “run faster than a streamline train”; withstand
anything less than a “bursting shell”; and 37 lift a car
over his head. Sent to Earth from the “doomed planet”
Krypton, he was raised by human foster parents, whose
love helped infuse him with an unapologetic desire to
“benefit mankind.” Admirable but aloof, the Golden Age
Superman was arguably more paragon than character, a
problem only partially solved by giving him a human
alter ego. Other Golden Age superheroes were similarly
archetypal: Batman was a crime-fighting millionaire,
Wonder Woman a warrior princess from a mythical
island.

Unauthorized copying or reuse of any part of this page is illegal. 36 CO NTI N U E


2 2
By contrast, the second (“Silver”) age of comics was

...............................................................................................................................................................................................................................................................................................................
38
marked by characters that, though somewhat simplistic Which choice most effectively sets up the main idea
of the following two sentences?
by today’s standards, 38 were provided with origin
A) NO CHANGE
stories often involving scientific experiments gone wrong.
B) reflected the increasing conservatism of the
In addition to super villains, the new, soon-to-be-iconic United States in the 1950s.
C) engaged in bizarre adventures frequently
characters of the 39 age: Spider-Man, the Fantastic Four,
inspired by science fiction.
and the Hulk among them—had to cope with mundane, D) were more “realistic” than their Golden Age
counterparts.
real-life problems, including paying the rent, dealing with
family squabbles, and facing anger, loneliness, and
39
ostracism. Their interior lives were richer and their
A) NO CHANGE
motivations more complex. Although sales remained
B) age;
strong for Golden Age stalwarts Superman and, to a C) age,
lesser extent, Batman, 40 subsequent decades would D) age—

show the enduring appeal of these characters.


40
The writer wants a conclusion to the sentence and
paragraph that logically completes the discussion of
the Silver Age and provides an effective transition
into the next paragraph. Which choice best
accomplishes these goals?
A) NO CHANGE
B) the distinctions between later stages of comic
book history are less well defined than the one
between the Golden and Silver Ages.
C) readers increasingly gravitated to the upstarts as
the 1960s and the Silver Age drew to a close.
D) these characters themselves underwent
significant changes over the course of the
Silver Age.

Unauthorized copying or reuse of any part of this page is illegal. 37 CO NTI N U E


2 2
More transformations would take place in the

...........................................................................................................................................................................................................................................................
41
medium as the Silver Age gave way to the Bronze and A) NO CHANGE
Modern (and possibly Postmodern) Ages. Such efforts B) would have yielded
C) were yielding
41 have yielded diminishing returns, as even the
D) will yield
complete relaunch of DC 42 Comics’ superhero’s, line in
2011 has failed to arrest the steep two-decade decline of 42
comic book sales. For both commercial and, arguably, A) NO CHANGE
creative reasons, 43 then, no transition was more B) Comic’s superhero’s
successful than 44 those from the Golden to Silver Age. C) Comics superhero’s
D) Comics’ superhero

43
A) NO CHANGE
B) however,
C) nevertheless,
D) yet,

44
A) NO CHANGE
B) these
C) that
D) DELETE the underlined portion.

STOP
If you finish before time is called, you may check your work on this section only.
Do not turn to any other section.

Unauthorized copying or reuse of any part of this page is illegal. 38


SAT Practice Test #6: Worksheets

ANSWER KEY

Reading Test Answers Writing and Language Test Answers

1 C 12 D 23 A 34 A 45 C 1 D 12 D 23 B 34 B
2 B 13 C 24 D 35 D 46 D 2 A 13 A 24 A 35 B
3 D 14 C 25 C 36 D 47 B 3 D 14 B 25 B 36 D
4 A 15 B 26 A 37 A 48 B 4 B 15 C 26 B 37 A
5 C 16 A 27 D 38 D 49 D 5 C 16 B 27 C 38 D
6 D 17 D 28 A 39 C 50 B 6 B 17 A 28 D 39 D
7 B 18 A 29 A 40 C 51 D 7 A 18 C 29 B 40 C
8 B 19 A 30 B 41 B 52 B 8 C 19 D 30 C 41 A
9 A 20 C 31 B 42 D 9 D 20 D 31 A 42 D
10 D 21 C 32 D 43 C 10 B 21 A 32 C 43 A

11 B 22 B 33 B 44 A 11 C 22 A 33 D 44 C

READING TEST
WRITING AND
RAW SCORE
LANGUAGE TEST
(NUMBER OF
RAW SCORE
CORRECT ANSWERS)
(NUMBER OF
CORRECT ANSWERS)

Math Test Math Test


No Calculator Answers Calculator Answers

1 B 11 B 1 A 11 B 21 D 31 10
2 B 12 D 2 C 12 D 22 B 32 31
3 C 13 A 3 A 13 A 23 C 33 97, 98, 99, 100, 101
4 A 14 A 4 D 14 B 24 D 34 5
5 D 15 D 5 B 15 D 25 B 35 1.25, 5/4
6 A 16 1, 2, 4, 8, 16 6 C 16 B 26 C 36 2.6, 13/5
7 C 17 15/4, 3.75 7 D 17 B 27 A 37 30
8 B 18 30 8 D 18 C 28 A 38 8
9 C 19 3/2, 1.5 9 B 19 C 29 B
10 D 20 1/6, .166, .167 10 B 20 C 30 D

MATH TEST MATH TEST


NO CALCULATOR CALCULATOR
RAW SCORE RAW SCORE
(NUMBER OF (NUMBER OF
CORRECT ANSWERS) CORRECT ANSWERS)

SAT Practice Test #6 Created 7/29/2016 6


Practice
#7
Test 7
Make time to take the practice test.
It’s one of the best ways to get ready
for the SAT.

After you’ve taken the practice test, score it


right away at sat.org/scoring.

© 2016 The College Board. College Board, SAT, and the acorn logo are registered trademarks of the College Board.
K2-5MSA07
2 2
Writing and Language Test
35 M I NU TES, 4 4 QUESTIONS

Turn to Section 2 of your answer sheet to answer the questions in this section.

Each passage below is accompanied by a number of questions. For some questions, you
will consider how the passage might be revised to improve the expression of ideas. For
other questions, you will consider how the passage might be edited to correct errors in
sentence structure, usage, or punctuation. A passage or a question may be accompanied by
one or more graphics (such as a table or graph) that you will consider as you make revising
and editing decisions.

Some questions will direct you to an underlined portion of a passage. Other questions will
direct you to a location in a passage or ask you to think about the passage as a whole.

After reading each passage, choose the answer to each question that most effectively
improves the quality of writing in the passage or that makes the passage conform to the
conventions of standard written English. Many questions include a “NO CHANGE” option.
Choose that option if you think the best choice is to leave the relevant portion of the
passage as it is.

Questions 1-11 are based on the following passage.


...................................................................................................................................

1
A) NO CHANGE
NASA: A Space Program with Down-to-Earth Benefits B) oftentimes
The National Aeronautics and Space Administration C) repeatedly
D) DELETE the underlined portion.
(NASA) is a US government agency whose budget is
frequently 1 many times contested. Many people think
2
of NASA’s programs as trivial. In truth, the agency has a
A) NO CHANGE
widespread positive 2 effect on society by serving as a
B) affect on
catalyst for innovation and scientific understanding, C) effect to
D) affects on

Unauthorized copying or reuse of any part of this page is illegal. 20 CO NTI N U E


2 2

...............................................................................................................................................................................................................................................................................................................
3 to create jobs, and showing humanity its place within 3
the universe. A) NO CHANGE
In 1958, the program’s first year, very few people B) creating jobs,
C) for job creation,
believed that it was even possible for a manned spacecraft
D) the creation of jobs,
to leave the atmosphere and orbit Earth. But by initiating
and collaborating on projects such as the Apollo Moon 4
missions, the space shuttle program, the Hubble Space A) NO CHANGE
4 Telescope, and unmanned planetary exploration, B) Telescope; and

NASA has continually challenged its scientists and C) Telescope and;


D) Telescope and,
engineers to do things that were previously thought
impossible. All along, these NASA projects have
5
5 greatly increased international cooperation. A short
Which choice most effectively sets up the list of
list of inventions 6 elaborated by NASA includes examples that follows in the next sentence?

communications satellites, invisible braces, and cordless A) NO CHANGE


B) garnered national publicity for the agency.
tools. All these inventions 7 spawns new industries,
C) generated a steady stream of new technology.
and with those industries, jobs. NASA also sponsors the D) made a lot of money for the agency.
Small Business Innovation Research and Small Business
Technology Transfer programs, which are specifically 6
designed to support technological development in the A) NO CHANGE
private sector. B) evolved
C) developed
D) progressed

7
A) NO CHANGE
B) spawned
C) has spawned
D) spawning

Unauthorized copying or reuse of any part of this page is illegal. 21 CO NTI N U E


2 2

...............................................................................................................................................................................................................................................................................................................
[1] A report by the Space Foundation estimated that 8
NASA contributed $180 billion to the economy in 2005. A) NO CHANGE
[2] More than 60 percent of the contribution 8 coming B) which came
C) to come
from commercial goods and services created by
D) came
companies using space-related technology. [3] This
translates as excellent returns from an agency that 9
received approximately 17.7 billion in tax dollars in 2014. To make this paragraph most logical, sentence 1
[4] This investment by taxpayers enhances not only the should be placed
A) where it is now.
national economy but also the United States’
B) after sentence 2.
competitiveness in the international market.
C) after sentence 3.
[5] Moreover, the benefits of NASA funding extend D) after sentence 4.
beyond the purely economic, as astrophysicist Neil
deGrasse Tyson indicated in his testimony before the US
Senate: “For . . . a penny on a dollar—we can transform
the country from a sullen, dispirited nation, weary of
economic struggle, to one where it has reclaimed its
twentieth-century birthright to dream of tomorrow.” 9

Unauthorized copying or reuse of any part of this page is illegal. 22 CO NTI N U E


2 2
Tyson’s expansive vision for the agency hints at

...............................................................................................................................................................................................................................................................................................................
10
another mission of NASA’s, illuminated in this At this point, the writer is considering adding the
following sentence.
observation by Apollo 14 astronaut Edgar Mitchell: “You
In addition, NASA has facilities in
develop an instant global consciousness, a people Washington, DC, Florida, Texas, California,
orientation, an intense dissatisfaction with the state of the and other states.
Should the writer make this addition here?
world, and a compulsion to do something about it.”
A) Yes, because it serves as a counterargument to
10 With world population topping seven billion,
the quotation from astrophysicist Neil
humanity is in need of some perspective. 11 Therefore, deGrasse Tyson.

we should continue to support NASA not only for B) Yes, because it reinforces the passage’s point
about the importance of NASA’s work.
practical reasons but also because it is a necessary vehicle C) No, because it undermines the passage’s claim
for increasing our awareness of how we can fulfill our about the economic benefits of NASA’s work.
D) No, because it blurs the paragraph’s focus by
responsibilities to the planet and each other.
introducing information that does not support
the paragraph’s claim about the importance of
NASA’s work.

11
A) NO CHANGE
B) Instead,
C) For example,
D) However,

Unauthorized copying or reuse of any part of this page is illegal. 23 CO NTI N U E


2 2
Questions 12-22 are based on the following passage

...............................................................................................................................................................................................................................................................................................................
12
and supplementary material.
A) NO CHANGE
B) also new practices,
Professional Development: A Shared Responsibility C) in addition to practices,
New theories, 12 new practices too, and D) practices,
technologies are transforming the twenty-first-century
workplace at lightning speed. To perform their jobs 13
successfully in this dynamic environment, workers in A) NO CHANGE
B) fields
many 13 fields—from social services to manufacturing,
C) fields,
must continually acquire relevant knowledge and update
D) fields;
key skills. This practice of continued education, also
known as professional development, benefits not only 14
employees but also their employers. 14 Accordingly, A) NO CHANGE
meaningful professional development is a shared B) Nevertheless,
responsibility: it is the responsibility of employers to C) Regardless,
D) Similarly,
provide useful programs, and it is also the responsibility
of employees to take advantage of the opportunities
15
offered to them.
Which choice best establishes the argument that
Critics of employer-provided professional follows?
development argue that employees 15 might consider a A) NO CHANGE
B) should lean heavily on their employers.
popular career path. If employees find themselves falling
C) must be in charge of their own careers.
behind in the workplace, these critics 16 contend. Then
D) will be ready for changes in the job market.
it is the duty of those employees to identify, and even pay

16
A) NO CHANGE
B) contend; then
C) contend then
D) contend, then

Unauthorized copying or reuse of any part of this page is illegal. 24 CO NTI N U E


2 2
for, appropriate resources to 17 show them how and

...............................................................................................................................................................................................................................................................................................................
17
why they are falling behind and what they should do A) NO CHANGE
about it. This argument ignores research pointing to B) address their deficiencies.
C) deal with their flaws and shortcomings.
high employee turnover and training of new staff as
D) allow them to meet their employers’ needs in
significant costs plaguing employers in many fields. terms of the knowledge they are supposed
to have.
Forward-thinking employers recognize the importance of
investing in the employees they have rather than hiring
18
new staff when the skills of current workers 18 get old
A) NO CHANGE
and worn out.
B) are no good anymore.
C) become obsolete.
D) have lost their charm.

Unauthorized copying or reuse of any part of this page is illegal. 25 CO NTI N U E


2 2
The most common forms of professional

...............................................................................................................................................................................................................................................................................................................
19
development provided to employees 19 includes A) NO CHANGE
coaching, mentoring, technical assistance, and B) include
C) including
workshops. Some employers utilize several approaches
D) has included
simultaneously, developing a framework that suits the
particular needs of their employees. 20 Around the same
20
time, the figure illustrates a simple yet comprehensive
A) NO CHANGE
professional-development model created for special B) Besides that,
education personnel. As the figure suggests, 21 receiving C) Nevertheless,
D) DELETE the underlined portion and begin the
coaching and consultation is the overarching framework,
sentence with a capital letter.
while the opportunity to belong to professional networks
and participate in activities such as foundation and 21
skill-building workshops is relatively unimportant. Which choice makes the writer’s description of the
figure most accurate?
Professional-Development Framework A) NO CHANGE
B) participation in foundation and skill-building
professional workshops is the overarching framework within
networks which staff receive coaching and consultation as
well as the opportunity to belong to a
coaching and professional network.
consultation C) membership in a professional network is the
overarching framework within which staff
receive coaching and consultation as well as the
foundation opportunity to attend foundation and
and skill-building skill-building workshops.
workshops D) receiving coaching and consultation is the
overarching framework within which staff have
the opportunity to belong to a professional
Adapted from Northern Suburban Special Education District,
network as well as attend foundation and
“Professional Development Framework.” ©2014 by Northern Suburban skill-building workshops.
Special Education Program.

Unauthorized copying or reuse of any part of this page is illegal. 26 CO NTI N U E


2 2
A recent trend in professional development that has

...............................................................................................................................................................................................................................................................................................................
22
provided advantages to both employers and employees is A) NO CHANGE
online instruction. From an employer perspective, the B) identify:
C) identify
first and perhaps most obvious advantage is the lower
D) identify—
cost of online professional development compared with
that of in-person workshops and training. Employers can
also 22 identify, which employees have successfully
completed instructional modules and which need to be
offered additional training. For employees, online
professional development provides the opportunity to
receive instruction at their own pace and interact with
other professionals online. This exciting trend has the
potential to make the shared responsibility of
professional development less burdensome for both
employers and employees.

Unauthorized copying or reuse of any part of this page is illegal. 27 CO NTI N U E


2 2
Questions 23-33 are based on the following passage.

...............................................................................................................................................................................................................................................................................................................
23
A) NO CHANGE
The Evolution of Slow Food B) for example,
In 1986, McDonald’s caused a stir in Italy when it C) however,
D) in fact,
opened a restaurant next to Rome’s historic Spanish
Steps. Young, on-the-go eaters were thrilled;
24
23 specifically, those who prized regional foods and
A) NO CHANGE
Italy’s convivial culture built on cooking and long meals
B) life; a
feared that the restaurant signaled the death of a way of C) life: a
life. To counter the rise of fast food and fast 24 life, a D) life. A

cohort of chefs, journalists, and sociologists spearheaded


a Slow Food movement, declaring loyalty to unhurried 25
At this point, the writer is considering adding the
enjoyment. 25
following sentence.
From its beginning, the movement 26 had opposed The group’s philosophy was connected to the
the standardization of taste that fast food chains promote. tale of the hare and the tortoise, in which the
tortoise wins the race.
For example, a McDonald’s hamburger made in Boston
Should the writer make this addition here?
tastes more or less the same as one made in Beijing. This
A) Yes, because it explains the primary belief that
consistency is made possible by industrial mass led to the development of the Slow Food
movement.
production. Slow Food supporters, by contrast, back
B) Yes, because it reinforces a claim that the writer
methods of growing and preparing food based on makes earlier in the paragraph.
regional culinary traditions. When produced using C) No, because it blurs the paragraph’s focus by
introducing a new idea that is not clearly
traditional methods, goat cheese made in France tastes explained.
different from goat cheese made in Vermont. A goat D) No, because it distracts from the paragraph’s
emphasis on the Slow Food movement’s origins
and beliefs.

26
A) NO CHANGE
B) opposes
C) will oppose
D) has opposed

Unauthorized copying or reuse of any part of this page is illegal. 28 CO NTI N U E


2 2
ingests the vegetation particular to the meadow in which

...............................................................................................................................................................................................................................................................................................................
27
it grazes, which, along with other environmental A) NO CHANGE
27 factors such as altitude and weather shapes the B) factors, such as altitude and weather,
C) factors such as, altitude and weather,
cheese’s taste and texture. If all foods were produced
D) factors, such as altitude and weather
under the industrial model, 28 we would have meals that
are not very flavorful.
28
During 29 their early years, the movement also Which choice most effectively supports the central
focused on the value of 30 spending lots of time with point of the paragraph?

friends and family during long meals. It emphasized A) NO CHANGE


B) the public would not be interested in learning
the importance of preserving these “easygoing, slow about traditional foods.
C) people would not be able to determine how a
particular food was made.
D) consumers would lose this diversity of flavors.

29
A) NO CHANGE
B) there
C) its
D) it’s

30
A) NO CHANGE
B) leisurely meals with friends and family.
C) eating slowly and in the company of loved ones
such as friends and family.
D) joining friends as well as family for
time-consuming meals.

Unauthorized copying or reuse of any part of this page is illegal. 29 CO NTI N U E


2 2
pleasures.” As the movement grew beyond Italy’s

...............................................................................................................................................................................................................................................................................................................
31
borders—today Slow Food International boasts more A) NO CHANGE
than 100,000 members in 150 countries—this emphasis B) portrayed
C) drew
on pleasure 31 pictured criticism for being elitist. Critics
D) sketched
have also asked if growing food using traditional
methods, as opposed to mass production, 32 can
32
adequately and affordably feed the world? Given the
A) NO CHANGE
hectic pace of modern life, who among us has the time B) adequately and affordably can feed the world?
and resources for elaborate meals? Such questions, in C) can adequately and affordably feed the world.
D) adequately and affordably can feed the world.
addition to environmental concerns, are at the heart of
perennial debates about food production.
33
Over time, Slow Food has broadened its mission to
A) NO CHANGE
focus on food that is good, clean, and fair for all.
B) Nonetheless,
Members assert that food should be flavorful, carrying C) To these ends,
the properties of a particular region; it should be raised D) By the same token,

using environmentally sustainable practices that preserve


biodiversity; and it should be accessible to all without
exploiting the labors of those who produced it. 33 In
short, Slow Food runs programs that support small-scale
producers in marketing regional foods in a world where
food corporations threaten to drive them out of the
marketplace and homogenize food choices.

Unauthorized copying or reuse of any part of this page is illegal. 30 CO NTI N U E


2 2
Questions 34-44 are based on the following passage.

...............................................................................................................................................................................................................................................................................................................
34
A) NO CHANGE
Was the Hoax a Hoax? B) headline in the New York Times, declared
For an hour on the evening of October 30, 1938, C) headline, in the New York Times declared,
D) headline, in the New York Times, declared
Orson Welles and other performers from the Mercury
Theatre flooded the airwaves with alarming “news
35
bulletins” about a Martian invasion supposedly occurring
The writer wants to add a supporting detail to
in Grover’s Mill, New Jersey. They were performing a indicate that the story was widely reported. Which
choice best accomplishes this goal?
radio play adapted from The War of the Worlds,
A) NO CHANGE
a science fiction novel by H. G. Wells. The next day, a
B) Other newspapers also ran stories claiming that
front-page 34 headline in the New York Times declared, the broadcast had incited mass hysteria.
“Radio Listeners in Panic, Taking War Drama as Fact.” C) In 2013, many newspapers and magazines
featured articles about the seventy‑fifth
35 The Times article claimed that people had fled their anniversary of the broadcast.
homes and that police stations had been swamped with D) The Times was then and is now one of the
United States’ most popular news sources.
calls. This version of events persisted, and the legend
became that Welles’s broadcast had as many as twelve 36
million people 36 who feared that Martians had invaded A) NO CHANGE
Earth. B) that feared
C) fearing
Recently, however, scholars have questioned the
D) to fear
accuracy of this legend, suggesting the degree of public
hysteria has been grossly exaggerated. The authors of an
37
article published in October 2013 go 37 so far to assign
A) NO CHANGE
blame for the distortion to the newspaper industry. B) as far
C) as far and
D) so far as

Unauthorized copying or reuse of any part of this page is illegal. 31 CO NTI N U E


2 2
38 At this time, Jefferson Pooley and Michael Socolow, 38

...............................................................................................................................................................................................................................................................................................................
both professors of communication studies, argue that the A) NO CHANGE
newspaper industry sought to discredit the newly B) On one hand,
C) In the article,
emerging technology of radio, which was cutting into
D) Next,
newspapers’ 39 profits. The newspaper industry tried to
do this by portraying the new medium as irresponsible. 39
[1] Proof of ulterior motives is scarce, Which choice most effectively combines the
40 consequently weakening Pooley and Socolow’s sentences at the underlined portion?
A) profits, which is what the newspaper industry
argument. [2] For instance, the C. E. Hooper ratings
tried to do when it portrayed
indicate that a mere 2 percent of households had tuned in B) profits, by which the newspaper industry
portrayed
to the broadcast. [3] Pooley and Socolow also call into
C) profits and tried to do this by portraying
question the validity of an oft-cited report that was based
D) profits, by portraying
on a survey conducted six weeks after the broadcast.
[4] Just because some people found the broadcast 40
unsettling, the authors contend, doesn’t mean they Which choice best establishes the main idea of the
paragraph?
believed it and reacted with real terror. [5] According to
A) NO CHANGE
this report, one million people indicated that they had
B) but evidence does suggest that reports of panic
been “frightened” by the broadcast. [6] Ratings, however, have been overblown.
reveal that 41 far fewer than a million people had been C) yet Pooley and Socolow maintain that the
newspaper industry intentionally distorted the
story.
D) making it difficult to determine what really
happened in 1938.

41
A) NO CHANGE
B) many less than
C) much less then
D) much fewer then

Unauthorized copying or reuse of any part of this page is illegal. 32 CO NTI N U E


2 2
listening to the broadcast. [7] Furthermore, Pooley and

.....................................................................................................................................................................................................................................................
42
Socolow note that this survey “conflated being To make this paragraph most logical, sentence 4
should be placed
‘frightened,’ ‘disturbed,’ or ‘excited’ by the program with
A) where it is now.
being ‘panicked.’” 42
B) after sentence 2.
Pooley and Socolow describe a more likely scenario: C) after sentence 5.
most people who heard the broadcast understood they D) after sentence 7.

were listening to a piece of fiction, but 43 some being


influenced by the sensationalized news coverage 43

afterward, later “remembered” being more afraid than A) NO CHANGE


B) some, they were
they had been. The researchers also suggest that,
C) some,
44 not unlike people who got caught up in the D) some
excitement of the story when reading about it in the
newspaper, the American public may have been willing to 44
embrace the legend because of its appeal to the Which choice most effectively signals the
comparison the writer is making between the two
imagination. groups mentioned?
A) NO CHANGE
B) unlike
C) not like
D) different from

STOP
If you finish before time is called, you may check your work on this section only.
Do not turn to any other section.

Unauthorized copying or reuse of any part of this page is illegal. 33


SAT Practice Test #7: Worksheets

ANSWER KEY

Reading Test Answers Writing and Language Test Answers

1 D 12 A 23 A 34 C 45 C 1 D 12 D 23 C 34 A
2 A 13 A 24 D 35 B 46 A 2 A 13 C 24 A 35 B
3 A 14 D 25 C 36 B 47 C 3 B 14 A 25 C 36 C
4 B 15 D 26 A 37 C 48 A 4 A 15 C 26 D 37 D
5 D 16 C 27 C 38 A 49 A 5 C 16 D 27 B 38 C
6 B 17 D 28 B 39 C 50 B 6 C 17 B 28 D 39 D
7 C 18 B 29 C 40 A 51 A 7 B 18 C 29 C 40 B
8 B 19 D 30 D 41 A 52 D 8 D 19 B 30 B 41 A
9 D 20 C 31 B 42 C 9 A 20 D 31 C 42 D
10 D 21 B 32 D 43 D 10 D 21 C 32 C 43 C

11 D 22 C 33 B 44 D 11 A 22 C 33 C 44 A

READING TEST WRITING AND


RAW SCORE LANGUAGE TEST
(NUMBER OF RAW SCORE
CORRECT ANSWERS) (NUMBER OF
CORRECT ANSWERS)

Math Test Math Test


No Calculator Answers Calculator Answers

1 C 11 D 1 B 11 A 21 C 31 195
2 C 12 B 2 A 12 B 22 B 32 2/5, .4
3 B 13 D 3 C 13 B 23 C 33 30
4 D 14 A 4 D 14 D 24 A 34 5/18, .277, .278
5 A 15 D 5 C 15 B 25 B 35 0, 3
6 C 16 8 6 C 16 D 26 B 36 6
7 B 17 30 7 A 17 A 27 A 37 2.4, 12/5
8 C 18 4 8 C 18 B 28 C 38 5/7, .714
9 B 19 8 9 A 19 A 29 D
10 B 20 6632 10 D 20 C 30 B

MATH TEST MATH TEST


NO CALCULATOR CALCULATOR
RAW SCORE RAW SCORE
(NUMBER OF (NUMBER OF
CORRECT ANSWERS) CORRECT ANSWERS)

SAT Practice Test #7 Created 12/2/2016 6


Practice
Test 8
#

Make time to take the practice test.


It’s one of the best ways to get ready
for the SAT.

After you’ve taken the practice test, score it


right away at sat.org/scoring.

© 2016 The College Board. College Board, SAT, and the acorn logo are registered trademarks of the College Board.
2 2
Writing and Language Test
35 M I NU TES, 4 4 QUESTIONS

Turn to Section 2 of your answer sheet to answer the questions in this section.

Each passage below is accompanied by a number of questions. For some questions, you
will consider how the passage might be revised to improve the expression of ideas. For
other questions, you will consider how the passage might be edited to correct errors in
sentence structure, usage, or punctuation. A passage or a question may be accompanied by
one or more graphics (such as a table or graph) that you will consider as you make revising
and editing decisions.

Some questions will direct you to an underlined portion of a passage. Other questions will
direct you to a location in a passage or ask you to think about the passage as a whole.

After reading each passage, choose the answer to each question that most effectively
improves the quality of writing in the passage or that makes the passage conform to the
conventions of standard written English. Many questions include a “NO CHANGE” option.
Choose that option if you think the best choice is to leave the relevant portion of the
passage as it is.

Questions 1-11 are based on the following passage


...................................................................................................................................

1
and supplementary material.
A) NO CHANGE
B) However,
Compost: Don’t Waste This Waste C) Furthermore,
Over the past generation, people in many parts of the D) For example,
United States have become accustomed to dividing their
household waste products into different categories for
recycling. 1 Regardless, paper may go in one container,
glass and aluminum in another, regular garbage in a
third. Recently, some US cities have added a new
category: compost, organic matter such as food scraps
and yard debris. Like paper or glass recycling,
composting demands a certain amount of effort from the

Unauthorized copying or reuse of any part of this page is illegal. 18 CO NTI N U E


2 2
public in order to be successful. But the inconveniences

...............................................................................................................................................................................................................................................................................................................
2
of composting are far outweighed by its benefits. Which choice best maintains the style and tone of the
passage?
Most people think of banana peels, eggshells, and
A) NO CHANGE
dead leaves as “waste,” but compost is actually a valuable
B) eliminating
resource with multiple practical uses. When utilized as a C) ousting
garden fertilizer, compost provides nutrients to soil and D) closing the door on

improves plant growth while deterring or killing pests


and preventing some plant diseases. It also enhances soil 3

texture, encouraging healthy roots and minimizing or A) NO CHANGE


B) savings increase
2 annihilating the need for chemical fertilizers. Better
C) increases savings
than soil at holding moisture, compost minimizes water D) also it increases savings
waste and storm runoff, 3 it increases savings on
watering costs, and helps reduce erosion on 4
embankments near bodies of water. In large A) NO CHANGE
4 quantities, which one would expect to see when it is B) quantities (which
C) quantities which
collected for an entire municipality), compost can be
D) quantities; (which
converted into a natural gas that can be used as fuel for
transportation or heating and cooling systems.

Unauthorized copying or reuse of any part of this page is illegal. 19 CO NTI N U E


2 2

...............................................................................................................................................................................................................................................................................................................
In spite of all compost’s potential uses, however, 5
most of this so-called waste is wasted. According to the The writer wants to include information from the
graph that is consistent with the description of
Environmental Protection Agency (EPA), over compost in the passage. Which choice most
5 13 million tons of metal ended up in US landfills in effectively accomplishes this goal?
A) NO CHANGE
2009, along with over 13 million tons of yard debris.
B) 6 million tons of rubber and leather
Remarkably, 6 less glass was discarded in landfills in
C) 10 million tons of textiles
that year than any other substance, including plastics or D) 33 million tons of food waste
paper. Even 7 worse, then the squandering of this
useful resource is the fact that compost in landfills cannot 6
break down due to the lack of necessary air and moisture. The writer wants to support the paragraph’s main
idea with accurate, relevant information from the
graph. Which choice most effectively accomplishes
this goal?
A) NO CHANGE
B) more metal
C) more food waste
D) more yard waste

7
A) NO CHANGE
B) worse than
C) worse then
D) worse, than

Unauthorized copying or reuse of any part of this page is illegal. 20 CO NTI N U E


2 2
As a result, organic material that is sent to landfills

...............................................................................................................................................................................................................................................................................................................
8
8 contribute to the release of methane, a very A) NO CHANGE
9 potent greenhouse gas. B) are contributing
C) contributes
EPA Estimates of Municipal Solid D) have contributed
Waste Discarded in US Landfills in 2009
35
(in millions of tons)

30 9
Amount of waste

25
A) NO CHANGE
20
15 B) sturdy
10 C) influential
5 D) commanding
0
te s r ls d te s ss r d
was astic pape eta woo was xtile gla othe r an er
l m e e h
fo
od p ya
rd t ubb leat
r
Type of waste

Adapted from Food Waste Disposal. ©n.d. by Food Waste Disposal, LLC.

Unauthorized copying or reuse of any part of this page is illegal. 21 CO NTI N U E


2 2

...............................................................................................................................................................................................................................................................................................................
10 While composting can sometimes lead to 10
accidental pollution through the release of methane gas, Which choice provides the most effective transition
from the previous paragraph?
cities such as San Francisco and Seattle have instituted
A) NO CHANGE
mandatory composting laws requiring individuals and B) Though government regulations vary,
businesses to use separate bins for compostable waste. C) Armed with these facts,
This strict approach may not work everywhere. However, D) Mindful of this setback,

given the clear benefits of composting and the


environmental costs of not composting, all municipalities 11

should encourage their residents either to create their A) NO CHANGE


B) nor
own compost piles for use in backyard gardens 11 or to
C) but
dispose of compostable materials in bins for collection. D) and

Unauthorized copying or reuse of any part of this page is illegal. 22 CO NTI N U E


2 2
Questions 12-22 are based on the following passage.

...............................................................................................................................................................................................................................................................................................................
12
Which choice most effectively combines the
A Lion’s Share of Luck sentences at the underlined portion?

It’s the beginning of February, and as they do every A) red,


B) red; in addition, red is
year, thousands of people line H Street, the heart of
C) red; in other words, red is
Chinatown in Washington, DC. The crowd has gathered D) red, the color; that is
to celebrate Lunar New Year. The street is a sea of
12 red. Red is the traditional Chinese color of luck and 13
happiness. Buildings are 13 draped with festive, red, A) NO CHANGE
banners, and garlands. Lampposts are strung with B) draped, with festive red banners,
C) draped with festive red banners—
crimson paper lanterns, which bob in the crisp winter
D) draped with festive red banners
breeze. The eager spectators await the highlight of the
New Year parade: the lion dance.
14
Experts agree that the lion dance originated in the Which choice most effectively completes the
Han dynasty (206 BCE–220 CE); however, there is little explanation of a possible origin of the lion dance?

agreement about the dance’s original purpose. Some A) NO CHANGE


B) the evil spirit was called Nian.
evidence suggests that the earliest version of the dance
C) villagers dressed in lion costumes to scare the
was an attempt to ward off an evil spirit; 14 lions are spirit away.
obviously very fierce. Another theory is that an emperor, D) the precise location of the village remains lost to
history.
upon waking from a dream about a lion, hired an artist to

Unauthorized copying or reuse of any part of this page is illegal. 23 CO NTI N U E


2 2

...............................................................................................................................................................................................................................................................................................................
choreograph the dance. 15 The current function of the 15
dance is celebration. Which choice most effectively concludes the
paragraph?
The lion dance requires the strength, grace, and
A) NO CHANGE
coordination of two dancers, 16 both of whom are B) It turns out that the origins of the lion dance are
almost completely hidden by the elaborate bamboo and irrelevant.
C) Whatever its origins, today the lion dance is a
papier-mâché lion costume that they maneuver. One joyous spectacle, a celebration of the promise of
person operates the lion’s head as the other guides the the New Year.
D) Things are different these days, of course.
torso and tail. Many of the moves in the dance, such as
jumps, rolls, and kicks, are similar to 17 martial arts and
16
acrobatics. The dancers must be synchronized with the
A) NO CHANGE
music accompanying the dance—drums, cymbals, and
B) of which both
gongs that supply the lion’s roar—as well as with each C) both of them
other. D) both

17
A) NO CHANGE
B) the disciplines of martial arts and acrobatics.
C) martial artists and acrobats.
D) those in martial arts and acrobatics.

Unauthorized copying or reuse of any part of this page is illegal. 24 CO NTI N U E


2 2
[1] While there are many regional variations of the

...............................................................................................................................................................................................................................................................................................................
18
lion dance costume, all make extensive use of symbols Which choice provides information that is most
consistent in style and content with the information
and colors. [2] The lion’s head is often adorned with a about the symbolism of the tortoise?
phoenix 18 (a mythical bird) or a tortoise (for A) NO CHANGE
longevity). [3] Green lions encourage friendliness. B) (for new beginnings)
[4] Golden and red lions represent liveliness and bravery, C) (from Chinese mythology)
D) (for symbolic reasons)
respectively. [5] Their older counterparts, yellow and
white lions, dance more slowly and deliberately. [6] In
19
some variations, lions of different colors are different
To make this paragraph most logical, sentence 5
ages, and they move accordingly. [7] Black lions are the should be placed

youngest; therefore, they dance quickly and playfully. A) where it is now.


B) after sentence 1.
[8] The appearance of the lions varies, but their message
C) after sentence 3.
is consistent: Happy New Year. 19
D) after sentence 7.

Unauthorized copying or reuse of any part of this page is illegal. 25 CO NTI N U E


2 2

...............................................................................................................................................................................................................................................................................................................
As the parade winds its way through Chinatown, the 20
music crescendos, and the lion dance reaches 20 it’s A) NO CHANGE
climax with the “plucking of the greens.” Approaching a B) its
C) there
doorway in which dangles a red envelope filled with
D) their
green paper money, the 21 lion’s teeth snare the
envelope. It then chews up the bills and spits out the
21
22 money-filled envelope instead of chewing it up. The
A) NO CHANGE
crowd cheers for the lion dancers and for the prosperity B) lion snares the envelope with its teeth.
and good fortune their dance foretells. C) envelope is snared by the lion with its teeth.
D) teeth of the lion snare the envelope.

22
A) NO CHANGE
B) envelope that had been dangling from the
doorway.
C) envelope that had the money in it.
D) envelope.

Unauthorized copying or reuse of any part of this page is illegal. 26 CO NTI N U E


2 2
Questions 23-33 are based on the following passage.

...............................................................................................................................................................................................................................................................................................................
23
Which choice best fits with the tone of the rest of the
Court Reporting: Humans v. Machines passage?

Court reporters for years have been the record A) NO CHANGE


B) super-rigorous
keepers of the court, taking 23 scrupulous notes during
C) spot-on
24 hearings; depositions, and other legal proceedings. D) intense
Despite the increasing use of digital recording
technologies, court reporters still play a vital role in 24
A) NO CHANGE
B) hearings; depositions;
C) hearings, depositions,
D) hearings, depositions;

Unauthorized copying or reuse of any part of this page is illegal. 27 CO NTI N U E


2 2

...............................................................................................................................................................................................................................................................................................................
courtrooms. 25 Although machines can easily make 25
digital audio recordings of court events, they lack the At this point, the writer is considering adding the
following graph.
nuance of human court reporters in providing a precise
Salary Comparison: Court Reporters
record. versus Other Occupations
80
[1] Court reporters record the spoken word in real
70

(thousands of dollars)
time, most commonly using the technique of 60

Median salary
stenography. [2] A stenotype machine allows a person to 50
40
type about 200 words per minute (the speed of speech is
30
about 180 words per minute). [3] The typed words are 20
instantaneously translated onto a computer screen for the 10
judge to view, and the transcript is used later by people 0
legal court all US
who want to review the case, such as journalists and occupations reporters occupations
Occupations
lawyers. [4] Digital audio recording is becoming
Adapted from Bureau of Labor Statistics, US Department of Labor,
increasingly popular in courtrooms across the United Occupational Outlook Handbook, 2014–15 Edition.

States, with six states using solely audio recordings for Should the writer make this addition here?
A) Yes, because it supports the claim that court
reporting is an important part of a trial.
B) Yes, because it offers a relevant counterpoint to
the argument that the use of digital recorders is
on the rise.
C) No, because it presents information that is not
directly related to the paragraph’s discussion of
the role of court reporters.
D) No, because it does not provide information
about the pay scale for more experienced court
reporters.

Unauthorized copying or reuse of any part of this page is illegal. 28 CO NTI N U E


2 2
general jurisdiction sessions. [5] Proponents of going

...............................................................................................................................................................................................................................................................................................................
26
digital say that technology is the easiest way to get the A) NO CHANGE
most accurate record of the proceedings, as the machine B) subjected to
C) subjected from
records everything faithfully as it occurs and is not
D) subject for
26 subject to human errors such as mishearing or
mistyping. [6] However, with the rise of high-quality
27
recording technology, reliance on court reporters 27 as a
A) NO CHANGE
record keeper is decreasing. 28 B) each as record keepers
C) as record keepers
D) to be a record keeper

28
To make this paragraph most logical, sentence 6
should be placed
A) where it is now.
B) after sentence 1.
C) after sentence 3.
D) after sentence 4.

Unauthorized copying or reuse of any part of this page is illegal. 29 CO NTI N U E


2 2

...............................................................................................................................................................................................................................................................................................................
Champions of court reporting, though, argue the 29
29 opposite. They argue that with the increased reliance Which choice most effectively combines the
sentences at the underlined portion?
on technology, errors actually increase. Because digital
A) opposite, such
systems record 30 indiscriminately; they cannot discern
B) opposite—
important parts of the proceedings from other noises in C) opposite, which is
the courtroom. 31 Despite this, a digital device does D) opposite; their opinion is

indeed record everything, but that includes loud noises,


such as a book dropping, that can make the actual words 30

spoken impossible to hear. A court reporter, however, A) NO CHANGE


B) indiscriminately, they
C) indiscriminately. They
D) indiscriminately, therefore they

31
A) NO CHANGE
B) In other words,
C) Therefore,
D) Consequently,

Unauthorized copying or reuse of any part of this page is illegal. 30 CO NTI N U E


2 2

...............................................................................................................................................................................................................................................................................................................
can distinguish between the words 32 and distinguish 32
between the extrinsic noises that need not be recorded. A) NO CHANGE
Also, if a witness mumbles, a human court reporter can B) also between the
C) and when there are
pause court proceedings to ask the witness to repeat what
D) and the
he or she said. In some cases, digital recording 33 makes
it necessary for the judge to make additional
33
announcements at the beginning of a trial. Increasing use Which choice provides the best supporting example
of technology is “a transition from accurate records to for the main idea of the paragraph?
A) NO CHANGE
adequate records,” says Bob Tate, president of the
B) requires a courtroom monitor to ensure the
Certified Court Reporters Association of New Jersey. equipment is functioning properly.
Despite the apparent benefits of using digital C) leads to changes in the roles and duties of several
members of the courtroom staff.
recording systems in courtrooms, there is still a need for
D) has led to the need for retrial because of
the human touch in legal proceedings. At least for the indistinct testimony from key witnesses.

foreseeable future, machines simply cannot replicate the


invaluable clarification skills and adaptability of human
court reporters.

Unauthorized copying or reuse of any part of this page is illegal. 31 CO NTI N U E


2 2
Questions 34-44 are based on the following passage.

...............................................................................................................................................................................................................................................................................................................
34
A) NO CHANGE
Fire in Space B) however,
On Earth, fire provides light, heat, and comfort. Its C) accordingly,
D) subsequently,
creation, by a process called combustion, requires a
chemical reaction between a fuel source and oxygen. The
35
shape that fire assumes on Earth is a result of
A) NO CHANGE
gravitational influence and the movement of molecules.
B) strove for a method to make their study of
In the microgravity environment of space, 34 moreover, biofuel combustion
C) looked for a method to study biofuel combustion
combustion and the resulting fire behave in
D) sought a method to study combustion of biofuels
fundamentally different ways than they do on
Earth—differences that have important implications for
researchers.
A group of engineering students from the University
of California at San Diego (UCSD), for example, 35 tried
to find a method to make their biofuel combustion study
(fuels derived from once-living material) free of the
drawbacks researchers face on Earth. The standard
method involves burning droplets of fuel, but Earth’s
gravitational influence causes the droplets to lose

Unauthorized copying or reuse of any part of this page is illegal. 32 CO NTI N U E


2 2
spherical symmetry while burning. This 36 deformation

...............................................................................................................................................................................................................................................................................................................
36
results in subtle variations in density that both 37 causes Which choice provides the most precise description
of the phenomenon depicted in the previous
uneven heat flow and limits the size of the droplets that sentence?
can be tested. Specially designed “drop towers” 38 built A) NO CHANGE
for this purpose reduce these problems, but they provide B) alteration
no more than 10 seconds of microgravity, and droplet C) transformation
D) modification
size is still too small to produce accurate models of
combustion rates. 39 The UCSD students understood
37
that these limitations had to be surmounted. As part of
A) NO CHANGE
the program, researchers fly their experiments aboard B) cause uneven heat flow and limit
aircraft that simulate the microgravity environment of C) cause uneven heat flow and limits
space. The aircraft accomplish this feat by flying in D) has caused uneven heat flow and has limited

parabolic paths instead of horizontal ones. On the plane’s


38
ascent, passengers feel twice Earth’s gravitational pull, but
A) NO CHANGE
for brief periods at the peak of the trajectory,
B) intended for this use
C) constructed for this function
D) DELETE the underlined portion.

39
Which choice provides the most effective transition
between ideas in the paragraph?
A) NO CHANGE
B) The UCSD group sought to overcome these
difficulties by participating in NASA’s
Microgravity University program.
C) The engineering group realized that aircraft
might be the tools they were looking for.
D) Thus, for the UCSD group, drop towers were not
an adequate solution.

Unauthorized copying or reuse of any part of this page is illegal. 33 CO NTI N U E


2 2

...............................................................................................................................................................................................................................................................................................................
40 “weightlessness” or microgravity similar to what is 40
experienced in space, is achieved. A) NO CHANGE
These flights allowed the UCSD students to B) “weightlessness” or microgravity, similar to what
is experienced, in space
experience microgravity 41 . Specifically, they C) “weightlessness” or, microgravity, similar to
42 investigated the combustion of biofuel droplets in what is experienced in space
D) “weightlessness,” or microgravity similar to what
microgravity for twice as long as could be accomplished is experienced in space,

41
At this point, the writer is considering adding the
following.
and perform their experiment without traveling
into space
Should the writer make this addition here?
A) Yes, because it elaborates on the advantage the
students gained from the flights.
B) Yes, because it reveals that the students did not
actually go into space, a point that the previous
paragraph does not address.
C) No, because it shifts focus away from the
students’ experiences while on the flights.
D) No, because it restates what has already been said
in the sentence.

42
A) NO CHANGE
B) could investigate
C) were investigating
D) were able to investigate

Unauthorized copying or reuse of any part of this page is illegal. 34 CO NTI N U E


2 2
in drop towers and to perform tests with larger droplets.

...................................................................................................................................................................................................................................................................
43
The larger, 43 spherically symmetric droplets burned Which choice most effectively establishes that the
UCSD students’ approach had solved a problem,
longer and gave the students more reliable data on mentioned earlier in the passage, relating to burning
combustion rates of biofuels because the droplets’ fuel on Earth?

uniform shape reduced the variations in density that A) NO CHANGE


B) combustible
hinder tests performed in normal gravity. The students
C) microgravity-influenced
hope the new data will aid future research by improving D) biofuel-derived
theoretical models of biofuel combustion. Better
combustion-rate models may even lead to the production 44
of more fuel-efficient engines and improved A) NO CHANGE
44 techniques, for fighting fires in space or at future B) techniques for fighting fires, in space or at future
outposts
outposts on the Moon and Mars. C) techniques for fighting fires in space or at future
outposts
D) techniques for fighting fires in space, or at future
outposts,

STOP
If you finish before time is called, you may check your work on this section only.
Do not turn to any other section.

Unauthorized copying or reuse of any part of this page is illegal. 35


SAT Practice Test #8: Worksheets

ANSWER KEY

Reading Test Answers Writing and Language Test Answers

1A 12 D 23 D 34 B 45 C 1 D 12 A 23 A 34 B
2 C 13 D 24 A 35 C 46 A 2 B 13 D 24 C 35 D
3 C 14 A 25 B 36 B 47 D 3 C 14 C 25 C 36 A
4 D 15 D 26 D 37 C 48 B 4 B 15 C 26 A 37 B
5 A 16 B 27 B 38 D 49 B 5 D 16 A 27 C 38 D
6 D 17 C 28 A 39 C 50 B 6 C 17 D 28 C 39 B
7 D 18 B 29 D 40 A 51 C 7 B 18 B 29 B 40 D
8 B 19 C 30 C 41 D 52 C 8 C 19 D 30 B 41 A
9 C 20 A 31 D 42 A 9 A 20 B 31 B 42 D
10 B 21 C 32 B 43 C 10 C 21 B 32 D 43 A

11 B 22 A 33 C 44 A 11 A 22 D 33 D 44 C

READING TEST WRITING AND


RAW SCORE LANGUAGE TEST
(NUMBER OF RAW SCORE
CORRECT ANSWERS) (NUMBER OF
CORRECT ANSWERS)

Math Test – No Calculator Math Test – Calculator


Answers Answers

1 D 11 B 1 A 11 B 21 D 31 102
2 A 12 D 2 C 12 C 22 A 32 2
3 A 13 C 3 A 13 C 23 A 33 30
4 C 14 C 4 C 14 D 24 D 34 25.4, 127/5
5 B 15 D 5 B 15 D 25 A 35 2, 8
6 B 16 3 6 D 16 B 26 C 36 0
7 B 17 32 7 C 17 A 27 B 37 576
8 D 18 1.5, 3/2 8 B 18 C 28 D 38 .8, 4/5
9 A 19 8 9 D 19 A 29 B
10 C 20 144 10 C 20 B 30 B

MATH TEST – MATH TEST –

NO CALCULATOR CALCULATOR

RAW SCORE RAW SCORE

(NUMBER OF (NUMBER OF
CORRECT ANSWERS) CORRECT ANSWERS)

SAT Practice Test #8 6


2 2
Writing and Language Test
35 M I NU TES, 4 4 QUESTIONS

Turn to Section 2 of your answer sheet to answer the questions in this section.

Each passage below is accompanied by a number of questions. For some questions, you
will consider how the passage might be revised to improve the expression of ideas. For
other questions, you will consider how the passage might be edited to correct errors in
sentence structure, usage, or punctuation. A passage or a question may be accompanied by
one or more graphics (such as a table or graph) that you will consider as you make revising
and editing decisions.

Some questions will direct you to an underlined portion of a passage. Other questions will
direct you to a location in a passage or ask you to think about the passage as a whole.

After reading each passage, choose the answer to each question that most effectively
improves the quality of writing in the passage or that makes the passage conform to the
conventions of standard written English. Many questions include a “NO CHANGE” option.
Choose that option if you think the best choice is to leave the relevant portion of the
passage as it is.

Questions 1-11 are based on the following passage. 1


...................................................................................................................................

A) NO CHANGE
Survival in the Hostile Environment of NW Rota-1 B) watch
[1] Sixty miles north of Guam and more than C) observe to see
D) visually watch
1,700 feet under the ocean’s surface is the summit of
NW Rota-1, an undersea volcano discovered in 2003.
[2] Surprisingly, the volcano appears to have been
continuously active; it even grew 130 feet in height
between 2006 and 2009. [3] Yet despite the hostile
environment created by the constant volcanic activity, life
is thriving there. [4] Special adaptations are the key to
survival. [5] At that depth, water pressure suppresses the
explosive force of the volcano’s eruptions, allowing
scientists to 1 watch and observe them up close via

Unauthorized copying or reuse of any part of this page is illegal. CO N T I N UE


Unauthorized copying or reuse of any part of this page is illegal. 18 CO NTI N U E
442
2 2
remotely operated vehicles. 2 2

...............................................................................................................................................................................................................................................................................................................
NW Rota-1 is far below the ocean’s photic zone To make the paragraph most logical, sentence 5
should be placed
where sunlight drives photosynthesis; 3 nevertheless,
A) where it is now.
bacteria supporting a unique food web have adapted to
B) after sentence 1.
this perpetually dark environment. The bacteria have C) after sentence 2.
evolved to use hydrogen sulfide instead of sunlight for D) after sentence 3.
the energy that drives their metabolic processes, and
hydrothermal venting is the source of the chemical soup 3

necessary to support 4 him or her. Seawater seeping A) NO CHANGE


B) afterward,
into fissures in the ocean floor is heated by underlying
C) furthermore,
magma, and the heat drives chemical reactions that D) similarly,
remove oxygen, sulfates, 5 and remove other chemicals
from the water. Once the superheated water (up to 750°F) 4
rises through vents in the ocean floor, additional A) NO CHANGE
reactions cause minerals and compounds to precipitate B) one.
C) them.
onto the seafloor, where bacteria feed on them.
D) it.

5
A) NO CHANGE
B) it also removes
C) also removing
D) and

Unauthorized copying or reuse of any part of this page is illegal. CO N T I N UE


Unauthorized copying or reuse of any part of this page is illegal. 19 CO NTI N U E
443
2 2
Loihi shrimp—originally thought to exist only 6

...............................................................................................................................................................................................................................................................................................................
around an undersea volcano near 6 Hawaii, survive by A) NO CHANGE
using tiny, shear-like claws to harvest rapidly growing B) Hawaii;
C) Hawaii—
bacterial filaments covering rocks near NW Rota-1’s
D) Hawaii
hydrothermal vents. The Loihi shrimp spend most of
their time grazing on the bacteria and evading another,
7
previously unknown, species of shrimp. Shrimp of that Which choice most effectively sets up the
species also graze on bacterial filaments as juveniles, information in the next sentence?

7 resulting from their ability to cope with the noxious A) NO CHANGE


B) but their adaptations are not yet fully understood
environment around the volcano. They feed on the Loihi by the scientific community.
shrimp and other organisms that are overcome by the C) thriving in an unusual ecosystem that also
includes crabs, limpets, and barnacles.
toxic plumes of volcanic gas and ash.
D) but as adults, their claws are large enough for the
shrimp to be predators.

Unauthorized copying or reuse of any part of this page is illegal. CO N T I N UE


Unauthorized copying or reuse of any part of this page is illegal. 20 CO NTI N U E
444
2 2
During an underwater eruption, steam quickly 8

...............................................................................................................................................................................................................................................................................................................
8 condenses. The steam leaves only carbon dioxide Which choice most effectively combines the
sentences at the underlined portion?
bubbles and droplets of molten sulfur. This means that
A) condenses and leaves
the water near NW Rota-1 is more acidic than 9 that of
B) condenses, having to leave
stomach acid, presenting yet another challenge to C) condenses, thereafter leaving
life-forms living nearby. As the carbon dioxide level in D) condenses, and then, after this, it leaves

Earth’s atmosphere rises, the 10 worlds’ ocean’s absorb


more carbon 11 dioxide. Organisms flourishing near the 9

volcano may help biologists understand how life adjusts A) NO CHANGE


B) those of stomach
to very acidic conditions. In addition, NW Rota-1 is a
C) the acid from stomach
natural laboratory where scientists can study conditions D) stomach
that may be similar to those that gave rise to life on Earth
and perhaps even other worlds. 10
A) NO CHANGE
B) world’s oceans’
C) world’s oceans
D) worlds oceans

11
The writer is considering revising the underlined
portion to the following.
dioxide, which increases their acidity.
Should the writer make this revision here?
A) Yes, because it explains the relevance of this
sentence to the point made in the paragraph.
B) Yes, because it helps the reader understand
why organisms near NW Rota-1 evolved the way
they did.
C) No, because it merely repeats information
provided earlier in the passage without
contributing to the paragraph’s main idea.
D) No, because it interrupts discussion of oceanic
life-forms with an irrelevant detail.

Unauthorized copying or reuse of any part of this page is illegal. CO N T I N UE


Unauthorized copying or reuse of any part of this page is illegal. 21 CO NTI N U E
445
2 2
Questions 12-22 are based on the following passage 12

...............................................................................................................................................................................................................................................................................................................
and supplementary material.
A) NO CHANGE
B) ridership, and while
Free Public Transportation
C) ridership,
City planners, concerned about vehicle traffic D) ridership;
clogging their cities’ roadways, are trying to find ways to
get people out of their cars and onto buses and trains. 13
One radical proposal some planners have considered is to Which choice is the most effective version of the
underlined portion?
make public transportation free to passengers. While
A) NO CHANGE
fare-free policies do increase 12 ridership, but they have B) more people use public transportation if they do
not been found to be an effective way to address traffic not have to pay a fare.
C) if people do not have to pay a fare, more of those
problems. Moreover, these policies may result in serious people use public transportation.
budget shortfalls. D) using public transportation is done by more
people when they do not have to pay a fare.
Not surprisingly, 13 public transportation is used
by more people when people do not have to pay a fare.
14
According to a report by the Center for Urban
A) NO CHANGE
Transportation Research, public transit systems that
B) declined:
abolish fares typically see a short-term increase in C) declined,
ridership of about 50 percent. However, this increase D) declined. As

does not necessarily correlate with a decrease in car


traffic. Evidence suggests that when buses and subways 15
A) NO CHANGE
are free, people often take bus and train trips they would
B) that
not have taken otherwise while still using their cars
C) one
nearly as much as they did before. In 2013 Tallinn, D) the policy
Estonia, instituted fare-free rides for city residents
(becoming the largest city in the world to do so), but car
use in Tallinn has only slightly 14 declined; as a 2014
study by the KTH Royal Institute of Technology in
Sweden found that car traffic in Tallinn was down less
than 3 percent since 15 it was enacted.

Unauthorized copying or reuse of any part of this page is illegal. CO N T I N UE


Unauthorized copying or reuse of any part of this page is illegal. 22 CO NTI N U E
446
2 2
Instituting a fare-free system 16 can also have a 16

...............................................................................................................................................................................................................................................................................................................
devastating effect on a city’s transportation budget. All Which choice best introduces the paragraph?
public transportation systems are subsidized by the A) NO CHANGE

government to some extent, but large systems gain a B) also requires planners to make careful
considerations about changes in service.
substantial portion of their operating revenue from fares. C) might also have a negative impact on the
Since systems that go fare-free see increases in ridership, environment as more service is added.
D) also has the drawback of increasing crowding on
they often must operate more buses and trains and hire public transportation.
more drivers and other personnel at the same time that
they are losing a key source of funding. Advocates of 17
fare-free policies claim that the costs of these policies are A) NO CHANGE
largely offset by various 17 savings, however, a recent B) savings,
C) savings, but
study comparing projected results of fare-free policies in
D) savings; and
different cities found this outlook to be 18 way too
sunny. For example, in San Francisco, CA, fare-free
18
A) NO CHANGE
B) looking too much on the bright side.
C) pretty upbeat.
D) overly optimistic.

Unauthorized copying or reuse of any part of this page is illegal. CO N T I N UE


Unauthorized copying or reuse of any part of this page is illegal. 23 CO NTI N U E
447
2 2
public transit was projected to save $8.4 million per year 19

...............................................................................................................................................................................................................................................................................................................
in fare collection costs 19 but create a deficit of $72 Which choice provides an accurate interpretation of
the chart?
million per year in lost fares, on top of capital
A) NO CHANGE
investments in new equipment and infrastructure. 20
B) and save an additional $112 million from lost
fares,
Projected Yearly Savings and Costs of Implementing a C) but result in a total increase of $184 million per
Fare-Free Policy year in operating costs,
Savings D) and save $72 million per year in costs related to
Total
from Cost of adding service,
Transit Cost in additional
eliminating adding
agency lost fares operating
fare service
costs
collection 20
Lane
Transit, $100,000 to $5 not The writer is considering adding the following
$5 million sentence based on information from the chart.
Eugene, $500,000 million provided
OR By contrast, Lane Transit in Eugene, OR, would
Muni, San lose only $5 million in fares if it instituted a
$112 $72 $184
Francisco, $8.4 million fare-free system.
million million* million
CA
Public Should the writer make this addition here?
Transit, $30 $30.9 A) Yes, because it proves how little money Eugene
not provided $900,000
Hamilton, million million would lose under a fare-free system compared
Canada with San Francisco.
*plus $512 million in capital investments B) Yes, because it reinforces the claim made by
advocates of fare-free policies mentioned earlier
Adapted from Transportation Research Board, “Implementation and in the paragraph.
Outcomes of Fare-Free Transit Systems.” ©2012 by Transportation
Research Board. C) No, because it does not support the argument
that fare-free systems cause a substantial loss for
governments.
D) No, because it contradicts a point about fare
collection made earlier in the paragraph.

Unauthorized copying or reuse of any part of this page is illegal. CO N T I N UE


Unauthorized copying or reuse of any part of this page is illegal. 24 CO NTI N U E
448
2 2
This is not to say that fare-free public transportation 21

...............................................................................................................................................................................................................................................................................................................
is always a bad idea. Some college towns and resort A) NO CHANGE
communities embrace the model because buses can go B) do not have
C) did not have
faster when drivers 21 would not have had to collect
D) will not have
fares. For large cities looking to reduce automobile traffic,
though, 22 research about Tallinn, Estonia, could be
22
instructive. Which choice provides the best conclusion to the
passage?
A) NO CHANGE
B) subways will prove to be more important than
buses.
C) public transportation should be cheaper but
not free.
D) fare-free public transportation is not the answer.

Unauthorized copying or reuse of any part of this page is illegal. CO N T I N UE


Unauthorized copying or reuse of any part of this page is illegal. 25 CO NTI N U E
449
2 2
Questions 23-33 are based on the following passage. 23

...............................................................................................................................................................................................................................................................................................................
A) NO CHANGE
Wet Plate Photography: An Old Technique Makes a B) it
New Splash
C) one
[1] Upon the arrival of the digital camera, D) he or she
professional photographers harrumphed that 23 they
produced ugly, low-resolution images. [2] Yet eventually 24
the vast majority of them traded film for megapixels. The writer plans to add the following sentence to this
paragraph.
[3] The latest digital cameras take pictures so crisp that
Why wouldn’t they?
the images in them appear to be die-cut. [4] Even today’s
To make the paragraph most logical, the sentence
humblest smartphones snap bright, sharp photos. [5] A should be placed
few contemporary photographers, however, have A) after sentence 1.

embraced an anachronistic method that was state-of- B) after sentence 2.


C) after sentence 4.
the-art technology when it was invented in 1851: wet
D) after sentence 5.
plate photography. 24

Unauthorized copying or reuse of any part of this page is illegal. CO N T I N UE


Unauthorized copying or reuse of any part of this page is illegal. 26 CO NTI N U E
450
2 2
Wet plate photographers essentially create their own 25

...............................................................................................................................................................................................................................................................................................................
film. The process can be dangerous, given that it requires At this point, the writer is considering adding the
following sentence.
the use of several volatile chemicals. 25 To take a wet
It’s also labor-intensive, involving several
plate photograph, photographers usually first arrange or intricate steps.
pose 26 it’s subjects before mixing collodion (a viscous, Should the writer make this addition here?
light-sensitive chemical solution) with bromide, iodide, A) Yes, because it serves as an effective transition by
reiterating the main idea of the previous
or chloride and applying the mixture to a clean, polished
paragraph.
glass plate. Dried collodion is unusable, 27 so once the B) Yes, because it sets up the paragraph’s outline of
photo is snapped with a massive, tripod-mounted the process of wet plate photography.
C) No, because it blurs the paragraph’s focus on the
dangers involved in wet plate photography.
D) No, because it provides an opinion in a
paragraph that is focused on facts.

26
A) NO CHANGE
B) its
C) there
D) their

27
A) NO CHANGE
B) but
C) and
D) for

Unauthorized copying or reuse of any part of this page is illegal. CO N T I N UE


Unauthorized copying or reuse of any part of this page is illegal. 27 CO NTI N U E
451
2 2
camera, the photographer has 28 nominal minutes to 28

...............................................................................................................................................................................................................................................................................................................
develop it, using more chemicals. When the image The writer wants to emphasize how quickly wet plate
photographers have to work. Which choice most
appears in the negative, water is used to stop the process. effectively accomplishes this goal?
A chemical “fix bath” turns the negative image into a A) NO CHANGE
positive one. The photo is then immersed in water and B) a few

warmed. 29 In conclusion, it is coated with lavender C) a matter of


D) mere
30 oil to give it (a protective finish).

29
A) NO CHANGE
B) Finally,
C) Thus,
D) Nevertheless,

30
A) NO CHANGE
B) oil—to give it a protective finish.
C) oil, to give it, a protective finish.
D) oil to give it a protective finish.

Unauthorized copying or reuse of any part of this page is illegal. CO N T I N UE


Unauthorized copying or reuse of any part of this page is illegal. 28 CO NTI N U E
452
2 2
Wet plate photos are marvelously fine-grained and 31

...............................................................................................................................................................................................................................................................................................................
detailed, and they seem to glow with an ethereal silvery A) NO CHANGE
light. One misstep or a speck of dust on the glass plate, B) will have swirled
C) have swirled
though, and flaws appear. Smudges resembling oyster
D) swirled
shells 31 swirl around the photos’ edges. Sunbursts or
streaks emerge where collodion pools unevenly. Since the
32
film requires long exposures, moving subjects blur. 32 A
A) NO CHANGE
shifting arm or leg might even disappear because of the B) An arm or a leg, shifting during the long
exposure time required by wet plate
lengthy exposure time required. The exposure time
photography, might even disappear.
required explains why people in wet plate photographs C) A wet plate photographer’s subject’s arm or leg
might even disappear during this long exposure
often look dour: it’s hard to hold a smile for that long.
time.
Prominent among contemporary wet plate D) A shifting arm or leg might even disappear.
photographers is Joni Sternbach, whose work centers,
appropriately, on water and people’s relationship to it. 33
Sternbach’s photo series Ocean Details, Sea/Sky, and The writer wants to highlight the contrast between
Sternbach’s techniques and the people Sternbach
SurfLand depict surging surfs, roiling waves, and the photographs. Which choice most effectively
accomplishes this goal?
surfers who ride them. 33 Her subjects could be
A) NO CHANGE
nineteenth-century wave riders, if not for the modern
B) The subjects of her photos could be ordinary
board shorts and bikinis they wear. Sternbach people,
characterizes wet plate photography as “one part C) It would be hard to tell her subjects are surfers,
D) They would appear to come from all walks of
photography, one part performance art, and one part life,
three-ring circus,” a worthwhile endeavor because it
produces the unique, haunting images she seeks. “When I
look at a digital print,” she says, “it might be gorgeous
and smooth, but it’s on a piece of paper and it’s one of
many.”

Unauthorized copying or reuse of any part of this page is illegal. CO N T I N UE


Unauthorized copying or reuse of any part of this page is illegal. 29 CO NTI N U E
453
2 2
Questions 34-44 are based on the following passage. 34

...............................................................................................................................................................................................................................................................................................................
The writer is considering revising the underlined
Digging Up Cities portion to the following.

In 2010, as a construction crew began to tear up cities by excavating artifacts that have
accumulated over centuries of land development.
sidewalks in New York City’s South Street Seaport to
Should the writer make this revision here?
replace a water pipe, Alyssa Loorya and her team watched
A) Yes, because it helps set up the rest of the passage
eagerly, picks and brushes in hand. Loorya, an urban by explaining what urban archaeologists do.

archaeologist, studies the history of 34 cities. Any New B) Yes, because it identifies the characteristics that
make particular cities worthy of archaeological
York City construction project using municipal funds study.
35 are required to consider whether historical artifacts C) No, because it does not give enough detail about
the kinds of artifacts that urban archaeologists
will be affected during construction, and if that typically find.
possibility 36 exists or is possible, an urban D) No, because it does not explain how excavation
benefits the study of a city’s history.
archaeologist must be consulted. Since the South Street
Seaport area was a bustling commercial center for early
35
colonists, Loorya anticipated that a rich history lay
A) NO CHANGE
beneath the pavement. “It’s our job to document and B) have been
recover that history before it’s lost,” she said. C) is
D) were

36
A) NO CHANGE
B) exists potentially,
C) exists, it is necessary that
D) exists,

Unauthorized copying or reuse of any part of this page is illegal. CO N T I N UE


Unauthorized copying or reuse of any part of this page is illegal. 30 CO NTI N U E
454
2 2
As the work continued, 37 therefore, the team 37

...............................................................................................................................................................................................................................................................................................................
faced obstacles. Fieldwork in a city has to be done A) NO CHANGE
intermittently: the construction crew had to proceed one B) though,
C) meanwhile,
block at a time to avoid interrupting traffic, and the
D) similarly,
archaeology team’s work was periodically 38 halted—by
stormy weather and the discovery of toxic materials
38
underground. Moreover, as archaeologists underground
A) NO CHANGE
attempted to relay information to those at the surface, B) halted;
they had to contend with the noise of construction C) halted,
D) halted
vehicles, car horns, and 39 pedestrians’ noise on the
busy New York City streets.
39
A) NO CHANGE
B) the noise of pedestrians
C) pedestrians
D) that of pedestrians

Unauthorized copying or reuse of any part of this page is illegal. CO N T I N UE


Unauthorized copying or reuse of any part of this page is illegal. 31 CO NTI N U E
455
2 2
Despite these setbacks, Loorya and her team 40

...............................................................................................................................................................................................................................................................................................................
eventually began to uncover some interesting artifacts. In A) NO CHANGE
2012, the team discovered a foundation wall, a network of B) site, among these were
C) site, including
wooden pipes, and several well bases dating to the
D) site; including
eighteenth century. In August 2013, the archaeologists
discovered thousands of objects in a single fifteen-foot
41
stretch that was likely a garbage disposal 40 site. Which choice most effectively sets up the example
Including buttons from Revolutionary War uniforms, discussed in the following sentence?

clay pipes, and an imported mineral water bottle from A) NO CHANGE


B) the numerous rituals associated with hospitality
Germany.
C) public utility planning and infrastructure
As they cleaned and catalogued the artifacts, the development
archaeologists took stock of their findings. The team’s D) how major construction projects were financed

discoveries provided a snapshot of 41 the various kinds


42
of construction materials that were used in the eighteenth
Which choice best maintains the style and tone of the
century. Colonial-era New Yorkers went to great lengths passage?
to secure fresh drinking water, Loorya noted, given the A) NO CHANGE
42 effort involved in laying wooden pipes to bring in B) blood, sweat, and tears
fresh water from surrounding areas, digging very deep C) hassle
D) feats of strength and fortitude
wells, brewing alcohol to mask the water’s salty taste, and
even importing bottled water.

Unauthorized copying or reuse of any part of this page is illegal. CO N T I N UE


Unauthorized copying or reuse of any part of this page is illegal. 32 CO NTI N U E
456
2 2
Through such discoveries, 43 they tell the story of a 43

..............................................................................................................................................................................................................................................................
city’s history in a new way. 44 “One of my favorite A) NO CHANGE
things is putting together someone’s life,” Loorya said. B) we
C) colonial-era New Yorkers
D) urban archaeologists

44
The writer wants to conclude the passage with a
quotation from Loorya that illustrates the broad
impact of her team’s work. Which choice most
effectively accomplishes this goal?
A) NO CHANGE
B) “New York City construction has a lot of stops
and starts,”
C) “Finding the bits and pieces that were actually
used by the people in the past makes New York
City’s history real,”
D) “We call our archaeological technique
‘monitoring,’ and we work hand-in-hand with
the contractors and are a part of their team,”

STOP
If you finish before time is called, you may check your work on this section only.
Do not turn to any other section.

Unauthorized copying or reuse of any part of this page is illegal. CO N T I N UE


Unauthorized copying or reuse of any part of this page is illegal. 33
457
SAT Practice Test
Worksheet: Answer Key
Reading Test Writing and Language Test Math Test – Math Test –
Answers Answers No Calculator Calculator
Answers Answers
QUESTION #

QUESTION #

QUESTION #

QUESTION #

QUESTION #

QUESTION #
MARK YOUR

MARK YOUR

MARK YOUR

MARK YOUR

MARK YOUR

MARK YOUR
ANSWERS

ANSWERS

ANSWERS

ANSWERS

ANSWERS

ANSWERS
CORRECT

CORRECT

CORRECT

CORRECT

CORRECT

CORRECT

CORRECT

CORRECT

CORRECT

CORRECT

CORRECT

CORRECT
1 D 27 A 1 B 23 B 1 B 1 B
2 D
2 B 28 D 2 B 24 B 2 A
3 B
3 B 29 D 3 A 25 B 3 D
4 A
4 A 30 C 4 C 26 D 4 A 5 D
5 C 31 B 5 D 27 A 5 C 6 A
7 C
6 A 32 D 6 C 28 D 6 B
8 A
7 C 33 A 7 D 29 B 7 D
9 D
8 C 34 B 8 A 30 D 8 C 10 D
9 B 35 B 9 D 31 A 9 B 11 A

10 C 36 A 10 C 32 D 10 C 12 D
13 C
11 D 37 A 11 A 33 A 11 B
14 B
12 D 38 C 12 C 34 A 12 D 15 D
13 D 39 D 13 B 35 C 13 A 16 B
14 B 40 C 14 B 36 D 14 B 17 C
18 B
15 B 41 A 15 D 37 B 15 B
19 B
16 C 42 C 16 A 38 D 16 360 20 A
17 A 43 C 17 C 39 C 17 2 21 D
18 A 44 D 18 D 40 C 18 8 22 B
23 B
19 C 45 A 19 C 41 C 3/4
19 24 C
or .75
20 D 46 D 20 C 42 A 25 C
5/2
21 A 47 C 21 B 43 D 20 26 D
or 2.5
22 B 48 A 22 D 44 C 27 A
28 C
23 D 49 A
29 A
24 A 50 B
30 D
25 C 51 C 31 6
26 B 52 D 32 2
33 8

34 9

35 15
36 3/2 or 1.5
37 1.3
38 3

READING TEST WRITING AND LANGUAGE TEST MATH TEST – MATH TEST –
RAW SCORE RAW SCORE NO CALCULATOR CALCULATOR
(Total # of Correct Answers) (Total # of Correct Answers) RAW SCORE RAW SCORE
(Total # of (Total # of
Correct Answers) Correct Answers)

Scoring Your SAT Practice Test #9 7


2 2
Writing and Language Test
35 M I NU TES, 4 4 QUESTIONS

Turn to Section 2 of your answer sheet to answer the questions in this section.

Each passage below is accompanied by a number of questions. For some questions, you
will consider how the passage might be revised to improve the expression of ideas. For
other questions, you will consider how the passage might be edited to correct errors in
sentence structure, usage, or punctuation. A passage or a question may be accompanied by
one or more graphics (such as a table or graph) that you will consider as you make revising
and editing decisions.

Some questions will direct you to an underlined portion of a passage. Other questions will
direct you to a location in a passage or ask you to think about the passage as a whole.

After reading each passage, choose the answer to each question that most effectively
improves the quality of writing in the passage or that makes the passage conform to the
conventions of standard written English. Many questions include a “NO CHANGE” option.
Choose that option if you think the best choice is to leave the relevant portion of the
passage as it is.

Questions 1-11 are based on the following passage. 1


...................................................................................................................................

A) NO CHANGE
How a Cat in a Hat Changed Children’s Education B) and with
In a 1954 Life magazine article, author John Hersey C) and also
D) and competing with
expressed concern that children in the United States were
disengaged from learning how to read. Among other
problems, Hersey noted, the reading material available to
grade-schoolers had a hard time competing with
television, radio, 1 and other media for children’s
attention. One solution he proposed was to make

Unauthorized copying or reuse of any part of this page is illegal. CO N T I N UE


Unauthorized copying or reuse of any part of this page is illegal. 20 CO NTI N U E
316
2 2
children’s books more 2 interesting, since “an 2

...............................................................................................................................................................................................................................................................................................................
individual’s sense of wholeness . . . follows, and cannot The writer wants to include a quotation by Hersey
that supports the topic of the passage. Which choice
precede, a sense of accomplishment.” best accomplishes this goal?
The story of The Cat in the Hat’s publication began A) NO CHANGE
when William 3 Spaulding, the director of the B) interesting, since “learning starts with failure; the
first failure is the beginning of education.”
education division at the publishing company
C) interesting because “journalism allows its readers
Houghton Mifflin, read Hersey’s article and had an idea. to witness history; fiction gives its readers an
opportunity to live it.”
Spaulding agreed that there was a need for appealing
D) interesting with “drawings like those of the
books for beginning 4 readers. He thought he knew wonderfully imaginative geniuses among
children’s illustrators.”
who should write one. He arranged to have dinner with
Theodor Geisel, who wrote and illustrated children’s
3
books under the name “Dr. Seuss,” and issued him a
A) NO CHANGE
challenge: “Write me a story that first graders can’t put B) Spaulding the director
down!” C) Spaulding, the director,
D) Spaulding—the director

4
Which choice most effectively combines the
sentences at the underlined portion?
A) readers, and he
B) readers—namely, he
C) readers; and Spaulding
D) readers, and meanwhile he

Unauthorized copying or reuse of any part of this page is illegal. CO N T I N UE


Unauthorized copying or reuse of any part of this page is illegal. 21 CO NTI N U E
317
2 2
Having 5 known Spaulding for many years and 5

...............................................................................................................................................................................................................................................................................................................
having maintained a professional relationship with him, Which choice best supports the information that
follows in the sentence?
Geisel was an experienced writer and illustrator.
A) NO CHANGE
6 However, this new project presented him with an
B) acquired a reputation for perfectionism and for
obstacle. Spaulding told Geisel to write his entire book setting high standards for his work,

using a restricted vocabulary from an elementary school C) been interested in politics before breaking into
the genre of children’s literature,
list of 348 words. Geisel started two stories, only to D) published nine children’s books and having
abandon them when he found that he needed to use received three nominations for the prestigious
Caldecott Medal,
words that were not on the list. On the verge of giving up,
7 Geisel’s story finally hit upon an image that became 6
its basis: a cat wearing a battered stovepipe hat. His main A) NO CHANGE
character established, Geisel commenced the difficult task B) For example,
of writing a book with a limited vocabulary. 8 At the C) Furthermore,
D) At any rate,
end of a duration nine months long, The Cat in the Hat
was complete.
7
A) NO CHANGE
B) an image that Geisel finally hit upon became the
basis of his story:
C) Geisel finally hit upon the image that became the
basis for his story:
D) the story was finally based on an image that
Geisel hit upon:

8
A) NO CHANGE
B) After thirty-six weeks—or nine months—had
passed,
C) After a length of nine months had elapsed,
D) Nine months later,

Unauthorized copying or reuse of any part of this page is illegal. CO N T I N UE


Unauthorized copying or reuse of any part of this page is illegal. 22 CO NTI N U E
318
2 2
The book was a hit. Children were entertained by its 9

...............................................................................................................................................................................................................................................................................................................
plot about the antics of a mischievous cat and 9 is A) NO CHANGE
captivated by its eye-catching illustrations and B) was
C) has been
memorable rhythms and rhymes. Its sales inspired
D) DELETE the underlined portion.
another publishing company, Random House, to
establish a series for early readers called Beginner Books,
10
which featured works by Geisel and other writers, and
A) NO CHANGE
other publishers quickly followed suit. In the years that B) followed; many
10 followed. Many talented writers and illustrators of C) followed, many
D) followed—many
children’s books imitated Geisel’s formula of restricted
vocabulary and whimsical artwork. But perhaps the best
11
proof of The Cat in the Hat’s success is not its influence
The writer wants a conclusion that restates the
on other books but its 11 limited vocabulary and main themes of the passage. Which choice best
accomplishes this goal?
appealing word choices.
A) NO CHANGE
B) impressive worldwide sales that continue to
remain high to this day.
C) enduring ability to delight children and engage
them in learning how to read.
D) important role in the history of illustration in
the twentieth century.

Unauthorized copying or reuse of any part of this page is illegal. CO N T I N UE


Unauthorized copying or reuse of any part of this page is illegal. 23 CO NTI N U E
319
2 2
Questions 12-22 are based on the following passage. 12

...............................................................................................................................................................................................................................................................................................................
A) NO CHANGE
Keep Student Volunteering Voluntary B) to pick up litter,
A growing number of public schools in the C) litter collection,
D) picking up litter,
United States require students to complete community
service hours to graduate. Such volunteering, be it
13
helping at a local animal shelter, 12 when they pick up
The writer wants a transition from the previous
litter, or working at a health-care facility, has obvious paragraph that highlights the criticism of compulsory
volunteering mentioned in the previous paragraph.
benefits for the community it serves and teaches students
Which choice best accomplishes this goal?
important life skills. But critics say that making A) NO CHANGE
volunteerism compulsory misses the point of the act. B) Whatever the work may be,
13 By its very definition, volunteer work is done C) For many students,
D) Fortunately for communities in need,
willingly. By requiring students to do community service
in order to graduate, school 14 officials’ are taking away
14
students’ choice to give up their time for nonprofit
A) NO CHANGE
activities, making volunteerism less meaningful and
B) officials are taking away students
pleasurable. According to a psychological concept called C) officials are taking away student’s
the reactance theory, the loss of freedom in choosing an D) officials are taking away students’

activity can cause a negative reaction. For instance,


instead of focusing on the good they are doing, students
may become resentful of the demands that compulsory
volunteering places on their schedules.

Unauthorized copying or reuse of any part of this page is illegal. CO N T I N UE


Unauthorized copying or reuse of any part of this page is illegal. 24 CO NTI N U E
320
2 2
Proponents of compulsory 15 volunteering who are 15

...............................................................................................................................................................................................................................................................................................................
in favor of it point out that it allows young people to A) NO CHANGE
garner the benefits that volunteering offers. Students who B) volunteering, advocating it,
C) volunteering
volunteer report increased self-esteem, better
D) volunteering and its advocates
relationship-building skills, and 16 increasingly busy
schedules. Some studies have also found that students
16
who do community service are more likely to volunteer Which choice provides a supporting example that is
as adults, and thus 17 effect society positively over the most similar to the examples already in the sentence?

course of many years. A) NO CHANGE


B) a closer connection with their community.
C) less time spent engaging in social activities.
D) little increase in academic achievement.

17
A) NO CHANGE
B) affect
C) effecting
D) affects

Unauthorized copying or reuse of any part of this page is illegal. CO N T I N UE


Unauthorized copying or reuse of any part of this page is illegal. 25 CO NTI N U E
321
2 2
However, most research looks at students who 18

...............................................................................................................................................................................................................................................................................................................
volunteer in general, not making a distinction between A) NO CHANGE
students who are required to volunteer by their schools B) coercive
C) forcible
and those who volunteer willingly. One recent study by
D) imperative
Sara E. Helms, assistant professor of economics at
Samford University in Birmingham, Alabama, did focus
19
specifically on 18 mandatory volunteering. She found
A) NO CHANGE
that students who were required to volunteer rushed to B) school; they then,
complete their service hours in early high 19 school, C) school. They, then
D) school; they then
they then did significantly less regular volunteer work in
the twelfth grade 20 than the service hours of those not
20
required to volunteer. Helms concluded that compulsory
A) NO CHANGE
volunteering does not necessarily create lifelong
B) than did students who were
volunteers. C) than hours worked by students
D) compared with students

Unauthorized copying or reuse of any part of this page is illegal. CO N T I N UE


Unauthorized copying or reuse of any part of this page is illegal. 26 CO NTI N U E
322
2 2
Instead of requiring students to volunteer, schools 21

...............................................................................................................................................................................................................................................................................................................
21 have to recognize that not all students are equally Which choice most effectively sets up the point made
in the next sentence?
well suited to the same activities. Many studies show that
A) NO CHANGE
when schools simply tell students about opportunities for
B) should allow students to spend their time
community service and connect them with organizations participating in athletics and other
extracurricular activities.
that need help, more students volunteer of their own
C) should focus on offering arrangements that
free will. 22 make volunteering an easy and attractive choice.
D) are advised to recognize the limits of their ability
to influence their students.

22
The writer wants a conclusion that states the main
claim of the passage. Which choice best accomplishes
this goal?
A) It is imperative that schools do their part to find
volunteers for the many worthwhile
organizations in the United States.
B) Schools that do this will produce more engaged,
enthusiastic volunteers than schools that require
volunteer work.
C) Studies in the fields of psychology and
economics have revolutionized researchers’
understanding of volunteerism.
D) It is important that students choose charitable
work that suits their interests and values.

Unauthorized copying or reuse of any part of this page is illegal. CO N T I N UE


Unauthorized copying or reuse of any part of this page is illegal. 27 CO NTI N U E
323
2 2
Questions 23-33 are based on the following passage 23

...............................................................................................................................................................................................................................................................................................................
and supplementary material.
A) NO CHANGE
B) will long be believing
Marsupials Lend a Hand to Science
C) have long believed
Marsupials (mammals that carry their young in a D) long believe
pouch) are a curiosity among biologists because they lack
a corpus callosum, the collection of nerve fibers 24
connecting the two hemispheres of the brain. In most A) NO CHANGE
B) and favor the use of one hand over the other,
other mammals, the left hemisphere of the brain controls
C) one hand over the other that could be chosen,
the right side of the body, the right hemisphere controls
D) one hand on a regular basis,
the left, and the corpus callosum allows communication
between the hemispheres. Scientists 23 are long 25
believing that this structure enables complex tasks by A) NO CHANGE
sequestering skilled movement to a single hemisphere B) trait,
without sacrificing coordination between both sides of C) trait;
D) trait:
the body; this sequestration would explain handedness,
the tendency to consistently prefer 24 one hand over the
26
other, in humans. However, a recent finding of
A) NO CHANGE
handedness in marsupials suggests that a 25 trait other B) correlates with
than the presence of a corpus callosum 26 links as C) correlates from
handedness: bipedalism. D) links on

Unauthorized copying or reuse of any part of this page is illegal. CO N T I N UE


Unauthorized copying or reuse of any part of this page is illegal. 28 CO NTI N U E
324
2 2
Researchers at Saint Petersburg State University and 27

...............................................................................................................................................................................................................................................................................................................
the University of Tasmania observed marsupials walking Which choice accurately reflects the information in
the graph?
on either two legs (bipeds) or four (quadrupeds) and
A) NO CHANGE
performing tasks such as bringing food to their mouths.
B) scores of 0 or less indicated a left-forelimb
The scientists employed a mean handedness index; preference and positive scores indicated a lack of
forelimb preference.
27 negative scores indicated a left-forelimb preference
C) positive scores indicated a lack of forelimb
and positive scores indicated a right-forelimb preference. preference and negative scores indicated a
right-forelimb preference.
While eating, the eastern gray kangaroo, red-necked
D) positive scores indicated a left-forelimb
wallaby, red 28 kangaroo and, brush-tailed bettong, all preference and negative scores indicated a
right-forelimb preference.
bipedal marsupials, preferred using their left forelimb, as
revealed by 29 positive mean handedness index values
28
less than 0.2 for all four species. These results suggest
A) NO CHANGE
handedness among these animals. B) kangaroo, and
C) kangaroo; and
Mean Handedness Index
Scores of One-Handed Feeding D) kangaroo—and,
in Bipedal Marsupials
0.8
left-forelimb preference 29
0.6
Which choice most accurately reflects the data in the
Mean handedness index

0.4 graph?
A) NO CHANGE
0.2
B) positive mean handedness index values greater
0 than 0.6

–0.2 C) positive mean handedness index values between


right-forelimb preference 0.4 and 0.6
–0.4 D) mean handedness index values of 0
–0.6
brush- red red- eastern
tailed kangaroo necked gray
bettong wallaby kangaroo
Bipedal marsupial
Adapted from Andrey Giljov et al., “Parallel Emergence of True
Handedness in the Evolution of Marsupials and Placentals.” ©2015 by
Elsevier Ltd.

Unauthorized copying or reuse of any part of this page is illegal. CO N T I N UE


Unauthorized copying or reuse of any part of this page is illegal. 29 CO NTI N U E
325
2 2
30 Having four feet, quadrupedal marsupials in the 30

...............................................................................................................................................................................................................................................................................................................
study did not show a strong preference for the use of one Which choice provides the best transition from the
previous paragraph?
forelimb. For instance, gray short-tailed opossums and
A) NO CHANGE
sugar gliders were assigned mean handedness values very
B) Like most other mammals,
close to zero—they used their right and left forelimbs C) In contrast to their bipedal counterparts,
nearly equally. In effect, the study provided no evidence D) While using their forelimbs for eating,

of handedness among quadrupedal marsupials.

Unauthorized copying or reuse of any part of this page is illegal. CO N T I N UE


Unauthorized copying or reuse of any part of this page is illegal. 30 CO NTI N U E
326
2 2
31 Kangaroos, though, still do not exhibit 31

...............................................................................................................................................................................................................................................................................................................
handedness to the extent that humans do. As the Which choice presents a main claim of the passage?
researchers noted, the quadrupeds typically live in trees A) NO CHANGE

and employ all four limbs in climbing. The bipeds, on the B) For the marsupials in the study, then,
handedness seems to be associated with
other hand, are far less arboreal, leaving their forelimbs bipedalism.

relatively free for tasks in 32 whom handedness may C) There are many things scientists do not
understand about the marsupial brain.
confer an evolutionary advantage. Why the majority of D) Additional studies on this phenomenon will
marsupials studied preferred their left forelimbs while the need to be performed with other mammals.

majority of humans prefer their right remains a mystery,


32
however, 33 as does the mechanism by which, in the
A) NO CHANGE
absence of a corpus callosum, the hemispheres of the
B) which
marsupial brain communicate. C) what
D) whose

33
The writer wants to conclude the passage by recalling
a topic from the first paragraph that requires
additional research. Which choice best accomplishes
this goal?
A) NO CHANGE
B) though researchers should not neglect the sizable
minority of humans who are left handed.
C) and scientists believe that studies like this one
may someday yield insights into the causes of
certain neurological disorders.
D) and an additional study is planned to study
handedness in other animals that stand upright
only some of the time.

Unauthorized copying or reuse of any part of this page is illegal. CO N T I N UE


Unauthorized copying or reuse of any part of this page is illegal. 31 CO NTI N U E
327
2 2
Questions 34-44 are based on the following passage. 34

...............................................................................................................................................................................................................................................................................................................
Which choice provides the most effective transition
An Employee Benefit That Benefits Employers from the previous sentence to the information that
immediately follows in this sentence?
—1—
A) NO CHANGE
According to a 2014 report from the Society for B) In addition to the 2014 report,
Human Resource Management, 54 percent of surveyed C) Although these levels are impressive,
companies provide tuition assistance to employees D) Whether they want to or not,

pursuing an undergraduate degree, and 50 percent do so


35
for employees working toward a graduate degree.
Which choice most effectively establishes the main
34 Despite these findings, more companies should idea of the passage?
consider helping employees pay for education because A) NO CHANGE
doing so helps 35 increase customer satisfaction and B) solve the problem of rising tuition costs
improve the quality of the companies’ business. C) strengthen the US economy
D) attract and retain employees

Unauthorized copying or reuse of any part of this page is illegal. CO N T I N UE


Unauthorized copying or reuse of any part of this page is illegal. 32 CO NTI N U E
328
2 2
—2— 36

...............................................................................................................................................................................................................................................................................................................
Tuition-reimbursement programs signal that A) NO CHANGE
employers offer their 36 workers’ opportunities for B) workers opportunities’
C) workers opportunities
personal and professional development. According to
D) worker’s opportunity’s
professor of management Peter Cappelli, such
opportunities are appealing to highly motivated and
37
disciplined individuals and may attract applicants with
A) NO CHANGE
these desirable qualities. Many in the business B) stressed
community concur. Explaining his company’s decision to C) stressing
D) and he stressed
expand its tuition-assistance program, John Fox, the
director of dealer training at Fiat Chrysler Automobiles
in the United States, 37 who stressed the importance of
drawing skilled employees to Fiat Chrysler’s car
dealerships: “This is a benefit that can surely bring top
talent to our dealers,” he said.

Unauthorized copying or reuse of any part of this page is illegal. CO N T I N UE


Unauthorized copying or reuse of any part of this page is illegal. 33 CO NTI N U E
329
2 2
—3— 38

...............................................................................................................................................................................................................................................................................................................
Paying for tuition also helps businesses retain Which choice most effectively combines the
sentences at the underlined portion?
38 employees. Retaining employees is important not
A) employees, and this retention
only because it ensures a skilled and experienced
B) employees, the retaining of whom
workforce but also because it mitigates the considerable C) employees, which
costs of finding, hiring, and training new workers. D) employees; that
Employees whose tuition is reimbursed often stay with
their employer even after they complete their 39

39 degrees. Because their new qualifications give them A) NO CHANGE


B) degrees: because
opportunities for advancement within the company. The
C) degrees because
career of Valerie Lincoln, an employee at the aerospace D) degrees; because
company United Technologies Corporation 40 (UTC)
is a significant success story for her company’s 40
tuition‑reimbursement program. In eight years at UTC, A) NO CHANGE
Lincoln earned associate and bachelor’s degrees in B) (UTC)—
C) (UTC):
business and advanced from an administrative assistant
D) (UTC),
position to an accounting associate position. This allowed
UTC to retain an employee with a 41 deep knowledge of
41
her industry and years of valuable experience.
A) NO CHANGE
B) hidden
C) large
D) spacious

Unauthorized copying or reuse of any part of this page is illegal. CO N T I N UE


Unauthorized copying or reuse of any part of this page is illegal. 34 CO NTI N U E
330
2 2
—4— 42

..............................................................................................................................................................................................................................................................
Tuition reimbursement can be expensive, and many A) NO CHANGE
companies would find it impractical to pay for multiple B) minimizing costs associated with employees’
coursework
degrees for all employees. Businesses have succeeded in C) being effective at keeping down costs
42 minimizing and keeping down costs and ensuring D) keeping down costs
the relevance of employees’ coursework by offering fixed
amounts of reimbursement each year and stipulating 43
which subjects workers can study. Even with these A) NO CHANGE
B) diverted
methods, tuition reimbursement may not be appropriate
C) in diverting
in all cases, especially if classes are likely 43 to divert
D) diversions for
employees’ time and energy from their jobs.

Think about the previous passage as a whole as you


answer question 44.
Question 44 asks about the previous passage as a
44
whole.
The writer wants to insert the following sentence.
Still, since securing an excellent workforce is
crucial to a business’s success, employers should
give serious thought to investing in
reimbursement programs.
To make the passage most logical, the sentence
should be placed immediately after the last sentence
in paragraph
A) 1.
B) 2.
C) 3.
D) 4.

STOP
If you finish before time is called, you may check your work on this section only.
Do not turn to any other section.

Unauthorized copying or reuse of any part of this page is illegal. CO N T I N UE


Unauthorized copying or reuse of any part of this page is illegal. 35
331
SAT Practice Test
Worksheet: Answer Key
Reading Test Writing and Language Test Math Test – Math Test –
Answers Answers No Calculator Calculator
Answers Answers
QUESTION #

QUESTION #

QUESTION #

QUESTION #

QUESTION #

QUESTION #
MARK YOUR

MARK YOUR

MARK YOUR

MARK YOUR

MARK YOUR

MARK YOUR
ANSWERS

ANSWERS

ANSWERS

ANSWERS

ANSWERS

ANSWERS
CORRECT

CORRECT

CORRECT

CORRECT

CORRECT

CORRECT

CORRECT

CORRECT

CORRECT

CORRECT

CORRECT

CORRECT
1 A 27 B 1 A 23 C 1 B 1 B
2 A
2 B 28 D 2 D 24 A 2 C
3 B
3 D 29 A 3 A 25 A 3 B
4 C
4 B 30 D 4 A 26 B 4 C 5 C
5 A 31 A 5 D 27 D 5 A 6 D
7 B
6 A 32 B 6 A 28 B 6 A
8 C
7 D 33 C 7 C 29 C 7 D
9 C
8 C 34 B 8 D 30 C 8 C 10 D
9 C 35 B 9 D 31 B 9 C 11 A

10 B 36 D 10 C 32 B 10 D 12 C
13 C
11 D 37 C 11 C 33 A 11 A
14 A
12 A 38 A 12 D 34 C 12 B 15 B
13 A 39 A 13 A 35 D 13 C 16 C
14 B 40 B 14 D 36 C 14 B 17 D
18 C
15 C 41 D 15 C 37 B 15 A
19 D
16 C 42 A 16 B 38 C 16 2200 20 C
17 D 43 A 17 B 39 C 17 5 21 B
18 C 44 B 18 A 40 D 18 1.21 22 D
23 A
19 B 45 D 19 D 41 A 19 2500
24 B
20 B 46 C 20 B 42 D 20 20 25 A
21 D 47 A 21 C 43 A 26 D
22 D 48 B 22 B 44 D 27 A
28 D
23 B 49 C
29 D
24 A 50 A
30 A
25 C 51 C 31 6
26 C 52 D 32 146
33 2500

34 34

35 5/2 or 2.5
36 25/4 or 6.25
37 293
38 9

READING TEST WRITING AND LANGUAGE TEST MATH TEST – MATH TEST –
RAW SCORE RAW SCORE NO CALCULATOR CALCULATOR
(Total # of Correct Answers) (Total # of Correct Answers) RAW SCORE RAW SCORE
(Total # of (Total # of
Correct Answers) Correct Answers)

Scoring Your SAT Practice Test #10 7

Você também pode gostar